ASIPP Diagnosis and Therapy Questions Flashcards Preview

Pain Medicine Board Review MJ > ASIPP Diagnosis and Therapy Questions > Flashcards

Flashcards in ASIPP Diagnosis and Therapy Questions Deck (603)
Loading flashcards...
1
Q
  1. Proposed mechanisms of action for spinal cord
    stimulation include all of the following except:
    A. Segmental antidromic inhibition of A-beta afferents
    B. Blocking of transmission in the spinothalamic tract
    C. Supraspinal pain inhibition
    D. Activation of central inhibition of sympathetic efferent
    neurons
    E. Antidromic activation of C nociceptive afferents
A
  1. Answer: E
    Explanation:
    Reference:
    Krames, Interventional Pain Management,Second Edition;
    Chapter 53 Mechanisms of Action of Spinal Cord
    Stimulation
    A. Segmental activation of large A-beta fi bers within the
    dorsal columns which antidromically inhibit reception of
    small fi ber nociceptive information at the substantia
    gelatinosa of the dorsal horn.
    This was Melzack and Wall’s original hypothesis and is
    consistent with a classic “gate control” theory of spinal
    cord stimulation.
    B. Segmental blockade of neurotransmission in the
    spinothalamic tract.
    This theory is supported by studies that show there is
    inhibition of pain transmission locally within the cord
    during spinal cord stimulation.
    C. Spinal cord stimulation produces changes in
    supraspinal neurons that either modulate supraspinal
    pain transmission or trigger supraspinal descending
    inhibition of the dorsal horn.
    D. Activation of central inhibition of sympathetic efferent
    neurons could affect pain processing. The consistent effect
    of vasodilation supports a sympathetic inhibition effect of
    spinal cord stimulation.
    E. Release of putative neurotransmitters and/or
    neuromodulators. This theory is based on the observation
    that pain relief often outlasts the duration of stimulation
    for minutes, hours and sometimes days.
    Source: Schultz D, Board Review 2004
2
Q
740. For brachial plexus avulsion pain, the long-term outcomes
of DREZ lesioning are approximately:
A. 60-65%
B. 10-15%
C. 1-2%
D. 40-50%
E. 15-25%
A
  1. Answer: A
    Explanation:
    (Raj, Practical Mgmt of Pain, 3rd Ed., page 802)
    Long term relief of brachial plexus avulsion pain with
    DREZ lesioning is 60-65% at 3-7 years. Phantom and
    stump pain success is about 50-60%. Spinal cord injury
    pain is usually not responsive, except for end zone pain
    (segmental) occurring just below the level of injury. 70-
    75% of patients report successful relief with end zone pain.
    Source: Schultz D, Board Review 2004
3
Q
  1. The anterior spinothalamic tract:
    A. Is the primary target of a cordotomy
    B. Primarily conveys proprioceptive afferent fi bers
    C. Primarily conveys small fi ber afferents.
    D. Conveys light touch.
    E. Conveys temperature sensation
A
  1. Answer: D

Source: Feler C, Board Review 2005

4
Q
  1. Which of the following carries the lowest risk of
    complications?
    A. Microvascular decompression
    B. Subtemporal sensory rhizotomy
    C. Selective trigeminal rhizotomy
    D. Open trigeminal (nucleus caudalis) tractotomy
    E. Stereotactic trigeminal tractotomy
A
  1. Answer: A
    Explanation:
    (Raj, Pain Review 2nd Ed., page 311, Raj, Practical Pain
    Mgmt, 3rd Ed. Page
    798, Bonica 3rd ed. Page 2042-2043)
    Microvascular decompression, initially developed by
    Janetta, is a non-destructive and potentially curative
    operation for trigeminal neuralgia. A pulsating aberrant
    vessel loop, e.g., superior cerebellar artery (V3),
    trigeminal vein (V2), or anterior inferior cerebellar artery
    (V1), is felt to be the cause. An interposition felt of Tefl on
    or polyvinyl sponge is placed between the vessel loop and
    the trigeminal nerve. Recall, V1+V2+V3 –> TG –>
    Trigeminal Nerve –> Brainstem. Although, complications
    such as cerebellar or brainstem strokes, CSF leaks,
    meningitis…complication rates are
5
Q
743. Dextrose is added to lidocaine, during a spinal anesthetic.
Which position would most likely result in anesthesia of
the sacral dermatomes?
A. Prone
B. Side-lying
C. Jack Knife
D. Sitting upright
E. Trendelenburg
A
  1. Answer: D
    Explanation:
    (Raj, Practical Management of Pain, 3rd Ed., page 632,
    635)
    Baricity of a local anesthetic is described as the density of a
    local anesthetic solution divided by the density of CSF. The
    density of CSF is 1.001 to 1.005 at 37°C. Local anesthetic
    solutions are characterized relative to CSF as hyperbaric,
    hypobaric, or isobaric. Understanding this density
    relationship allows the anesthesiologist to take advantage
    of the characteristics of the local anesthetic or the position
    of the patient to direct local anesthetic toward the
    dermatomes to be anesthetized. A hyperbaric solution has
    a higher specifi c gravity than CSF, so that it moves to lowlying
    parts of the subarachnoid.
    Although prone may result in blockade of the sacral
    dermatomes, the sitting position would do this most
    effectively, by having the bulk of the local anesthetic dose
    go towards the sacral nerve roots.
    Source: Shah RV, Board Review 2005
6
Q
744. The most widely practiced percutaneous technique for
relief of trigeminal neuralgia is:
A. Glycerol rhizolysis
B. Balloon-catheter decompression
C. Microvascular decompression
D. Radiofrequency thermocoagulation
E. Radiosurgery
A
  1. Answer: D
    Explanation:
    (Raj, Pain Review 2nd Ed, page 311)
    Radiofrequency thermocoagulation of the trigeminal
    ganglion is the most widely practiced percutaneous
    intervention. MVD and radiosurgery are not percutaneous
    methods.
    Source: Schultz D, Board Review 2004
7
Q
745. What percentage of patients is affl icted by
glossopharyngeal neuralgia compared to trigeminal
neuralgia?
A. 0.1-0.5%
B. 1-2%
C. 10-15%
D. 30-40%
E. 10-20%
A
  1. Answer: B
    Explanation:
    (Raj, Pain Review 2nd Ed., page 312)
    Glossopharyngeal neuralgia affl icts 1-1.3% as many
    individuals as trigeminal neuralgia. The lancinating,
    paroxysmal qualities are similar but the pain is located at
    the base of the tongue, throat, and deep in the ear. Triggers
    include chewing and swallowing. GPN may rarely be
    associated with syncope or bradycardia.
    Source: Schultz D, Board Review 2004
8
Q
746. Rotator cuff tear is diagnosed by:
A. Plain radiographs
B. MRI
C. MR arthrography
D. CT
E. Sonography
A
  1. Answer: B
    Explanation:
    MRI provides the greatest imaging resolution and
    complete evaluation in the setting of shoulder pain.
    Although a rotator cuff tear may be diagnosed with
    ultrasound or MR arthrography, diagnostic MRI remains
    the best modality in MRI compatible patients
    Source: Bieneman B, Board Review 2005
9
Q
747. As a part of a psychological evaluation, a clinical interview
includes all of the following EXCEPT:
A. History
B. Financial and legal information
C. General medical status
D. Psychosocial information
E. Pain tolerance testing
A
  1. Answer: E

Source: Janata JW, Board Review 2005

10
Q
  1. While performing a right lumbar sympathetic
    radiofrequency lesioning, at L3, the patient complains
    of pain in the right groin. What is the likely etiology of
    this pain?
    A. Lesioning of the ilioinguinal nerve
    B. Psoas spasm
    C. Lesioning of the iliohypogastric nerve
    D. Lesioning of the genitofemoral nerve
    E. Quadratus spasm
A
  1. Answer: D

Source: Day MR, Board Review 2005

11
Q
  1. Which of the following is a true statement regarding a
    thoracic sympathetic block?
    A. Can only be performed with the patient in the prone
    position.
    B. Can be performed bilaterally at the same visit.
    C. Pneumothorax is not a concern.
    D. Blocks Kuntz’s fi bers
    E. Not effective for treating thoracic visceral pain.
A
  1. Answer: D

Source: Day MR, Board Review 2005

12
Q
  1. Which of the following factors infl uence the spread of
    local anesthetic in the subarachnoid space the most?
    A. baricity
    B. barbotage
    C. anesthetic dose
    D. injection level
    E. injection speed
A
  1. Answer: A
    Explanation:
    (Raj, Practical Mgmt of Pain 3rd Ed., page 635)
    Factors affecting the spread of local anesthetic include (1)
    baricity of the local anesthetic, (2) position of the patient
    after injection, (3) level of injection, (4) speed of injection,
    (5)dose and volume of the local anesthetic used, and (6) a
    technique known as barbotage. Of these, the two with the
    greatest infl uence are the baricity of the local anesthetic
    and the position of the patient.
    Source: Shah RV, Board Review 2005
13
Q
751. The somatoform condition with the lowest incidence is:
A. Factitious disorder
B. Hypochondriasis
C. Conversion disorder
D. Somatization disorder
E. Malingering
A
  1. Answer: C

Source: Janata JW, Board Review 2005

14
Q
  1. The following is (are) true regarding Deep Brain
    Stimulation (DBS):
    A. Deep brain stimulation is an effective method for controlling
    back pain.
    B. It is FDA approved for painful conditions.
    C. It is FDA approved for spasticity
    D. It is FDA approved for some movement disorders
    E. Two of the above
A
  1. Answer: D

Source: Feler C, Board Review 2005

15
Q
  1. Which of the following is true?
    A. “Radiculopathy” requires no neurologic defi cit
    B. Any patient who has had prior lumbar spine surgery
    who later presents with low back pain should be considered
    to have FBSS.
    C. A refl ex change alone is suffi cient to diagnose a
    radiculopathy
    D. Discogenic pain typically radiates into the affected dermatome
    E. Two of the above
A
  1. Answer: C

Source: Feler C, Board Review 2005

16
Q
  1. The following is not true:
    A. MRI evidence of degenerative disc disease is necessary
    for consideration of spinal instrumentation
    B. A patient complaining of mechanical back pain who
    has no evidence of instability is properly selected for
    decompression of a nerve root without stabilization.
    C. SCS is an FDA approved therapy for many indications
    D. The L4 disc is usually the level of disease in a L5
    radiculopathy
    E. An absent ankle refl ex is indicative of a S1
    radiculopathy
A
  1. Answer: A

Source: Feler C, Board Review 2005

17
Q
755. Subarachnoid hemorrhage is best diagnosed by what
test?
A. Enhanced CT
B. Unenhanced CT
C. Enhanced MRI
D. Unenhanced MRI
E. Skull radiographs
A
755. Answer: B
Explanation:
Noncontrast CT brain is the appropriate exam for any
acute neurologic abnormality
Source: Bieneman B, Board Review 2005
18
Q
756. Which of the following opioids should be avoided in a
patient with renal disease?
A. Meperidine
B. Sufentanil
C. Morphine
D. Hydrocodone
E. Hydromorphone
A
  1. Answer: A

Source: Day MR, Board Review 2005

19
Q
757. Thalamic lesioning is usually used to treat:
A. Shooting, allodynic pain
B. Deafferentation pain
C. Burning, dysesthetic pain
D. Peripheral nociceptive pain
E. Radicular pain
A
  1. Answer: A
    Explanation:
    (Raj, Pain Review 2nd Ed., page 309)
    Thalamotomy is useful for intermittent, shooting,
    hyperpathic or allodynic pain. Thalamotomy may not be
    useful for burning, dysesthetic, central, or deafferentation
    pain. Thalamotomy is not useful for peripheral nociceptive
    pain. One of the most effective targets is the inferior
    posteromedial thalamus.
    Source: Schultz D, Board Review 2004
20
Q
  1. A conversion disorder is:
    A. Intentionally produced or feigned
    B. Limited to pain
    C. Suggestive of a neurological or general medical condition
    D. Explainable by the effects of a substance.
    E. Unrelated to functional impairment
A
  1. Answer: C

Source: Janata JW, Board Review 2005

21
Q
759. Munchhausen syndrome is an type of:
A. Hypochondriacal presentation
B. Conversion disorder
C. Somatization disorder
D. Personality disorder
E. Factitious disorder
A
  1. Answer: E

Source: Janata JW, Board Review 2005

22
Q
  1. Which of the following is true about the stellate
    ganglion?
    A. Everybody has one
    B. It is located at C6
    C. It is formed by the fusion of the inferior cervical and fi rst
    thoracic ganglion.
    D. It is bordered anteriorly by the vertebral artery.
    E. Blockade of the ganglion reliably causes a sympathectomy
    of the ipsilateral upper extremity.
A
  1. Answer: C

Source: Day MR, Board Review 2005

23
Q
761. Somatization disorder criteria include all of the following
EXCEPT:
A. A history of many physical complaints
B. Onset after age 30
C. Four pain symptoms
D. Two gastrointestinal symptoms
E. One sexual symptom
A
  1. Answer: B

Source: Janata JW, Board Review 2005

24
Q
  1. The presence of pain behavior in chronic pain
    presentations:
    A. is an indication of psychopathology
    B. is abnormal in chronic pain populations
    C. indicates that pain is “all in the patient’s head”
    D. indicates the absence of true pathophysiology
    E. is a normal adaptation to an abnormal set of circumstances
A
  1. Answer: E

Source: Janata JW, Board Review 2005

25
Q
  1. With respect to cortical stimulation:
    A. It is FDA approved for the treatment of atypical facial
    pain.
    B. There is suffi cient evidence based medicine to recommend
    the procedure for patients with Anesthesia Dolorosa
    of the face.
    C. The procedure is easy to do for leg pain
    D. Complication rates for the procedure are reasonabable
    E. B and D are true
A
  1. Answer: D

Source: Feler C, Board Review 2005

26
Q
764. In taking the pain history, what factors are critical to
assess?
A. Temporal features
B. Expectational values of the patient
C. Educational features of the patient
D. All of the above
E. None of the above
A
  1. Answer: A
    Explanation:
    The pain history should include temporal, provocative,
    alleviative, and causative (initiative) parameters/ factors of
    a particular patent’s pain.
    Expectational values and educational features of the
    patient may contribute to pain intensity, duration,
    expression,and amenability to treatment, but are not
    constituents of the pain history.
    Source: Giordano J, Board Review 2005
27
Q
  1. Of the following statements pertaining to post lumbar
    puncture headaches, which is least accurate? Choose one:
    A. Age, female gender, body mass index, and history of recurrent
    headaches are major risk factors for PLPH.
    B. 80% of the cases of PLPH occur within 48 hours of the
    procedure.
    C. 30 cc’s of CSF taken will not induce a headache by volume
    loss and will be replaced within 90 minutes given
    that CSF is produced at a rate of 1cc/3 minutes or approximately
    500 cc’s a day.
    D. 30 cc’s of CSF taken will likely induce a headache by
    volume loss but will be replaced within 270 minutes
    given that CSF is produced at a rate of 1cc/9 minutes or
    approximately 150 cc’s a day.
    E. Lying prone for 1 hour and drinking 24 ounces of water
    after a LP has been shown to decrease the incidence of
    PLPH.
A
  1. Answer: D

Source: Goodwin J, Board Review 2005

28
Q
766. Personality disorders are easily diagnosed by:
A. Careful history-taking
B. Clinical interview
C. Observing waiting room behavior
D. Utilizing the Symptom Checklist – 90
E. None of the above
A
  1. Answer: C

Source: Janata J, Board Review 2006

29
Q

767.The presence of pain behavior in chronic pain
presentations:
A. indicates that pain is “all in the patient’s head”
B. is a normal adaptation to an abnormal set of circumstances
C. is an indication of psychopathology
D. is abnormal in chronic pain populations
E. indicates the absence of true pathophysiology

A
  1. Answer: C

Source: Janata J, Board Review 2006

30
Q
768. Münchhausen syndrome is a type of:
A. Somatization disorder
B. Factitious disorder
C. Conversion disorder
D. Hypochondriacal presentation
E. Personality disorder
A
  1. Answer: B

Source: Janata J, Board Review 2006

31
Q
769. The somatoform condition with the lowest incidence is:
A. Malingering
B. Somatization disorder
C. Factitious disorder
D. Conversion disorder
E. Hypochondriasis
A
  1. Answer: D

Source: Janata J, Board Review 2006

32
Q
770. Diagnostic of which plexus nerve should be performed
for the testicular pain?
A. Splanchnic nerves
B. Lumbar sympathetic nerves
C. Hypogastric plexus
D. Genitofemoral nerve
E. Ganglion impar
A
  1. Answer: A

Source: Day MR, Board Review 2005

33
Q
771. Tietze’s syndrome is defi ned as unilateral costochondritis
of what rib level/s?
A. 1st and 2nd
B. 2nd and 3rd
C. 3rd and 4th
D. 2nd only
E. 3rd only
A
  1. Answer: B

Source: Day MR, Board Review 2006

34
Q
  1. Which statement regarding occipital nerve stimulation
    is true:
    A. The electrode is placed transversely in the subcutaneous
    tissue plane overlying C1-2.
    B. The technique is contra-indicated in patients who have
    undergone posterior cervical spine surgery.
    C. Paresthesias are typically felt in the ipsilateral occiput
    and down the ipsilateral arm
    D. Bilateral occipital leads are contraindicated
    E. A stimulation trial is not necessary prior to implant
A
  1. Answer: A
    Explanation:
    With occipital nerve stimulation, the lead is placed
    transversely in the subcutaneous tissue plane overlying
    C1-2, Unilateral or bilateral leads can be placed depending
    on the patients’ pain pattern. Paresthesias are typically felt
    in the occiput and sometimes in the posterior neck and
    shoulder but do not radiate down the arm.The technique
    is extraspinal so it can be used in patients’ who have
    undergone previous posterior cervical spine surgery. As
    with all neurostimulation, a successful trial of stimulation
    is a necessary prerequisite for implant.
    Reference:
    Heavner, Interventional Pain Management, Second
    Edition; Chapter 57 Peripheral Nerve Stimulation:
    Current Concepts
    Source: Schultz D, Board Review 2004
35
Q
  1. Regarding meningitis, which of the following best
    suggests meningeal irritation? Choose one:
    A. Bilateral Horner’s syndrome
    B. Inability to stay awake
    C. A stiff neck coupled with Kernig’s and Brudzinski’s
    signs
    D. Inability to fall asleep because of headache-induced
    nausea
    E. Opisthotonus
A
  1. Answer: C

Source: Goodwin J, Board Review 2005

36
Q
774. Self-effi cacy is synonymous with:
A. Exclusive reliance on pain interventions
B. External locus of control
C. Internal locus of control
D. Social support
E. Euthymia
A
  1. Answer: C

Source: Janata JW, Board Review 2005

37
Q
  1. Malingering involves production of false or exaggerated
    symptoms that are:
    A. Intentionally produced
    B. Unconsciously motivated
    C. Symptomatic of a psychotic process
    D. Easily detectable on exam
    E. Associated with family history of depression
A
  1. Answer: A

Source: Janata JW, Board Review 2005

38
Q
776. Exclusion criteria for group therapy include all of the
following EXCEPT:
A. Severe depression
B. Pain behavior
C. Signifi cant personality disorders
D. Capacity for violence
E. Signifi cant history of noncompliance
A
  1. Answer: B

Source: Janata JW, Board Review 2005

39
Q
  1. After heat radiofrequency lesioning of the right
    sphenopalatine ganglion, the patient complains of right
    upper tooth numbness. What is the likely explanation?
    A. The greater palatine nerve was lesioned as well
    B. The lesser palatine nerve was lesioned as well
    C. The Vidian nerve was lesioned as well
    D. The maxillary nerve was lesioned as well
    E. The mandibular nerve was lesioned as well
A
  1. Answer: D

Source: Day MR, Board Review 2005

40
Q
778. All of the following neurosurgical procedures for pain
relief have historically been used for the treatment of
psychiatric conditions except:
A. Cingulotomy
B. Anterior capsulotomy
C. Leucotomy
D. Hypothalamotomy
E. Subtemporal sensory rhizotomy
A
  1. Answer: E
    Explanation:
    (Raj, Pain Review 2nd Ed., pages 309-311)
    Cingulotomy, anterior capsulotomy (anterior limb of
    internal capsule), leucotomy (pre-frontal lobotomy), and
    hypothalamotomy have been used for intractable cancer
    pain in multiple sites and for psychiatric disorders, such as
    obsessive compulsive disorders. Hypothalamotomy, in
    fact, may be benefi cial if there is a strong emotional
    component to the pain. Subtemporal sensory rhizotomy
    was the main operation performed for trigeminal
    neuralgia before the 1950s’. Unfortunately, recurrence
    rates ranged from 5-20% and there was a high incidence of
    complications: anesthesia dolorosa, dyesthesias, keratitis.
    Source: Schultz D, Board Review 2004
41
Q
779. Which heating method is contraindicated in patients
with spinal cord stimulation?
A. Diathermy
B. Hydrotherapy
C. Heat Lamps
D. Paraffi n
E. Hot packs
A
779. Answer: A
Explanation:
Diathermy is contraindicated in patients with spinal cord
stimulation
Source: Shah RV, Board Review 2005
42
Q
  1. Proper patient positioning for a subarachnoid phenol
    block is:
    A. Painful side up with no tilt
    B. Painful side down with no tilt
    C. Painful side up with the patient tilted anteriorly 45°.
    D. Painful side down with the patient tilted posteriorly 45°.
    E. Painful side down with the patient tilted anteriorly 45°.
A
  1. Answer: C

Source: Day MR, Board Review 2005

43
Q
  1. In which of the following types of patients would
    you expect the best results following a surgical
    sympathectomy?
    A. Failure of response to sympathetic blocks
    B. Raynaud’s syndrome
    C. Diabetic peripheral neuropathy
    D. Phantom limb pain
    E. Spinal cord injury end zone pain
A
  1. Answer: B
    Explanation:
    (Raj, Practical Management of Pain, page 803; Raj, Pain
    Review 2nd Ed. Page 314).
    All surgical sympathectomy should be prognosticated by a
    series of sympathetic blocks which unequivocally give a
    positive response. That being said, all of the above
    disorders may have a sympathetically maintained
    component. However, a painful vasospastic disorder such
    asRaynaud’s or complex regional pain syndrome would do
    the best. Central and chronic peripheral pain syndromes
    are less predictable.
    Source: Schultz D, Board Review 2004
44
Q
  1. What is true about Tuffi er’s line?
    A. It represents a horizontal line connecting the superiormost
    aspects of the palpable iliac crests
    B. It can be identifi ed, by using the inferior poles of the
    scapulae as landmarks
    C. It can be helpful in performing cervical epidural anesthesia
    D. It is an imaginary line connecting the C7 and L5 spinous
    processes
    E. It represents needle trajectory during the performance of
    a spinal anesthetic
A
  1. Answer: A
    Explanation:
    (Raj, Practical Mgmt of Pain, 3rd Ed., page 634)
    Spinal anesthesia is usually instituted with a needle
    inserted at an easily palpable interspace below L2.
    Depending on the patient’s individual anatomical features,
    the second, third, or fourth lumbar interspace may be
    selected. After the most prominent point of the iliac crests
    is located, an imaginary line is drawn between them
    (Tuffi er’s line), which usually crosses the L4 spinous
    process or the L4-L5 interspace.
    Source: Shah RV, Board Review 2005
45
Q
  1. The following are necessary of successful spinal cord
    stimulation implant except:
    A. Paresthesia sensation that overlaps region of pain
    B. Comfortable paresthesia
    C. Pain relief at low amplitudes
    D. Intact cognitive abilities of the patient recipient
    E. Absence of stimulation in nonpainful targets
A
  1. Answer: C
    Explanation:
    Reference:
    Bedder, Interventional Pain Management, Second Edition;
    Chapter 55 Implantation Techniques for Spinal Cord
    Stimulation
    Successful SCS implant requires a comfortable paresthesia
    sensation that overlaps the area of pain. This should be
    demonstrated in a trial of stimulation with the selected
    device prior to a decision to implant. The patient should
    have reasonably intact cognition because spinal cord
    stimulation requires the patient to turn on and off the
    device, keep track of the external programmer and be able
    to manage a certain amount of technology. Many patients
    with successfully implanted stimulators have paresthesia
    sensation in areas outside the pain target region. This is
    not necessarily a problem. Pain relief at low amplitudes is
    desirable but not required since an RF system with an
    external battery source can be used in these cases.
    Source: Schultz D, Board Review 2004
46
Q
784. A far left lateral disc bulge at the L4-5 level will likely
affect which nerve root?
A. The left L5 nerve root
B. The left L4 nerve root
C. The left L4 and L5 nerve roots
D. The bilateral L4 nerve roots
E. The bilateral L4 nerve roots
A
  1. Answer: B

Source: Bieneman B, Board Review 2005

47
Q
  1. Which of the following regarding pontine spinothalamic
    tractotomy is true?
    A. A more caudal level of analgesia may be obtained compared
    to a high cervical cordotomy
    B. There is minimal risk of obstructive sleep apnea compared
    to high cervical cordotomy
    C. Pancoast tumor-related pain is not an indication
    D. One theoretical advantage over mesencephalotomy is
    that the neospinothalamic and paleospinothalamic
    fi bers are in closer proximity
    E. Oculomotor disturbances are common
A
  1. Answer: D
    Explanation:
    (Raj, Pain Review 2nd Ed., pages 310; Bonica 3rd Ed.,
    pages 2048-9 & 2052-4;
    Raj, Practical Mgmt of Pain 3rd Ed., pg 795)
    Pontine spinothalamic tractotomy produces a more rostral
    level of analgesia compared to a high cervical cordotomy.
    Hence, it was introduced for neck and shoulder pain. C1-2
    cordotomy at was among the most useful procedures for
    unilateral cancer pain below the C5 dermatome. Both
    pontine spinothalamic tractotomy and high cervical
    cordotomy carry a similar risk of paralyzing the automatic
    phase of respiration and thus, causing sleep apnea. Neck
    and shoulder pain due to cancer, e.g., Pancoast tumor, are
    indications for pontine spinothalamic tractotomy. The
    neospinothalamic (spatial and temporal aspects of painful
    stimuli and more laterally located) and paleospinothalamic
    (affective and motivational aspects of painful stimuli and
    more medially located)are closer together at the pontine as
    compared to the midbrain (mesencephalotomy) level.
    Oculomotor complications are common at the midbrain
    level due to the proximity of the oculomotor nucleus and
    medial longitudinal fasciculus. However,
    mesencephalectomy does not carry a risk of sleep apnea.
    Source: Schultz D, Board Review 2004
48
Q
  1. For a patient presenting with a left facial droop and right
    upper extremity paresis, the most likely site of the lesion
    is:
    A. Brainstem
    B. Right parietal lobe
    C. Origin of the left facial nerve and left motor cortex
    D. Anterior bundle of the corpus collosum
    E. Left frontal lobe
A
  1. Answer: A

Source: Goodwin J, Board Review 2005

49
Q
787. Splenic laceration is best diagnosed by which of the
following tests?
A. Ultrasound
B. Plain radiographs
C. CT
D. MR
E. Sulfur colloid scan
A
  1. Answer: C
    Explanation:
    CT is the appropriate imaging modality in acute
    abdominal emergencies and is preferred with IV contrast
    for abdominal trauma
    Source: Bieneman B, Board Review 2005
50
Q
  1. Which of the following statements about nystagmus are true? Choose one:
    A. Attenuating nystagmus may be a medication side effect
    B. Immediate-onset nystagmus usually implicates inner ear
    pathology
    C. Delayed-onset nystagmus is a common cerebellar disease
    fi nding
    D. Nystagmus generally precedes and therefore heralds a neuromuscular junction disorder such as myesthenia
    gravis, so one should initiate a search for a small cell
    carcinoma of the lung.
    E. A, B and C are correct
A
  1. Answer: A

Source: Goodwin J, Board Review 2005

51
Q
789. Referred pain from pericarditis is felt where?
A. Left shoulder
B. Right shoulder
C. Left upper quadrant of the abdomen
D. Right upper quadrant of the abdomen
E. Mid epigastrium
A
  1. Answer: A

Source: Day MR, Board Review 2006

52
Q
  1. The 2004 International Headache Society’s revised
    criteria for chronic tension- type headache (TTH),
    requires a frequency over time consistent with which
    one of the following:
    A. 15 or more days per month over a minimum of 3
    months
    B. No more than 6 headache-free days in a 3 month period
    C. No fewer than 2 days involvement per week for a minimum
    of 3 months
    D. Between 1 and 14 days per month over a 3 month period
    E. An average of 60 days per year for at least 60 months (5
    years
A
  1. Answer: A

Source: Goodwin J, Board Review 2005

53
Q
  1. Current Perception Threshold testing:
    A. Can evaluate small nerve fi bers impossible to assess on
    standard EMG/NCS’s
    B. Is of minimal value in the blind because visual perception
    of stimuli is key to accuracy
    C. Is very expensive to perform and therefore not widely
    available
    D. Is of little value in assessing pain and temperature
    thresholds because the nerve fi bers are too small
    E. Is of greatest value where axonal versus demyelinating
    neuropathies need clarifi cation
A
  1. Answer: A

Source: Goodwin J, Board Review 2005

54
Q
  1. Regarding plexopathies, which of the following is true?
    A. Both the H-refl ex and F-waves may be prolonged
    B. The H-refl ex and F-waves are usually normal
    C. Fibrillations in paraspinal muscles do not rule in a
    plexopathy, but are suggestive of it
    D. Current perception threshold testing is less expensive
    and more sensitive than EMG
    E. Loss of an F-wave is meaningless because the action
    potential is impeded by a proximal lesion and therefore
    cannot be assessed with accuracy
A
  1. Answer: A

Source: Goodwin J, Board Review 2005

55
Q
  1. The initial imaging modality recommended for evaluation
    of traumatic odontoid (dens) fracture is?
    A. X-ray tomograms of the odontoid process
    B. Plain radiographs with lateral and open mouth views
    C. CT scan with axial and coronal recontructions
    D. T1-weighted MRI sagittal and coronal views
    E. Triple phase bone scan to identify fracture line
A
  1. Answer: B

Source: Bieneman B, Board Review 2005

56
Q
794. Nociception of the pancreas is mediated through which
splanchnic nerves?
A. T5-9
B. T10-11
C. T12
D. T8-11
E. T10-12
A
  1. Answer: A
    Explanation:
    Reference: Raj and Patt. Chapter 11. Visceral Pain. In:
    Pain Medicine: A Comprehensive review, 2nd Edition. Raj,
    Mosby, 2003, page 101.
    Source: Day MR, Board Review 2005
57
Q
  1. Which of the following does not characterize pelvic
    congestion syndrome?
    A. There is no identifi able pathologic condition
    B. Pain is dull and achy
    C. Complaints of suprapubic pain
    D. May have psychosomatic features such as headaches and
    urinary symptoms
    E. Common in post-menopausal women
A
  1. Answer: E
    Explanation:
    Reference: Raj and Pott. Chapter 11. Visceral Pain. In Pain
    Medicine: AComprehensive Review, 2nd Edition, Raj,
    Mosby, 2003, page 102
    Source: Day MR, Board Review 2005
58
Q
796. All of the following are complications of a celiac/
splanchnic block except:
A. Constipation
B. Hypotension
C. Paraplegia
D. Pneumothorax
E. Vascular injury
A
  1. Answer: A
    Explanation:
    Reference: Raj and Patt. Chapter 11. Visceral Pain. In:
    Pain Medicine: AComprehensive Review, 2nd Edition, Raj,
    Mosby, 2003, page 105
    Source: Day MR, Board Review 2005
59
Q
  1. True statements regarding Sacroiliac joint dysfunction
    include all of the following except:
    A. Pain radiating to hip, back, and thigh
    B. Pain worsened by twisting movements
    C. Straight leg raising may be positive
    D. Pain worsened by sitting on the contralateral ischeal
    tuberosity
    E. May cause hamstring spasm
A
  1. Answer: D
    Explanation:
    Reference: Raj. Chapter 43. Thoracoabdominal Pain. In:
    Practical Management of Pain.
    3rd Edition. Raj et al, Mosby, 2000. page 627.
    Source: Day MR, Board Review 2005
60
Q
  1. When present, which of the following refl exes or signs
    best localizes an upper motor lesion to a level above the
    cervical spinal cord? Choose one:
    A. Brisk jaw jerk
    B. Babinski sign (upgoing toe)
    C. Hoffman sign
    D. Loss of the superfi cial abdominal refl exes
    E. Clonus of one or both ankles
A
  1. Answer: A

Source: Goodwin J, Board Review 2005

61
Q
799. The most appropriate imaging modality for acute
headache is
A. Magnetic resonance imaging
B. Computed tomography
C. Magnetic resonance angiography
D. Intravenous angiography
E. Duplex scanning
A
  1. Answer: B

Source: Bieneman B, Board Review 2005

62
Q
800. What percentage of community-dwelling elderly suffer
from chronic pain?
A. 10-20%
B. 20-25%
C. 25-50%
D. 50-60%
E. > 60%
A
  1. Answer: C

Source: Day MR, Board Review 2006

63
Q
  1. Classic features of a syrinx include:
    A. Dissociated sensory loss
    B. Long tract signs below the level of the lesion
    C. Bowel or bladder dysfunction
    D. All of the above
    E. None of the above
A
  1. Answer: D

Source: Wirght PD, Board Review 2004

64
Q
802. Shoulder shrug tests which nerve?
A. Vagus
B. CN X
C. CN XII
D. Accessory Nerve
E. Phrenic Nerve
A
  1. Answer: D

Source: Wirght PD, Board Review 2004

65
Q
803. The corneal refl ex tests the trigeminal nerve and:
A. Vagus nerve
B. Spinal accessory nerve
C. Facial nerve
D. Oculomotor nerve
E. None of the above
A
  1. Answer: C

Source: Wirght PD, Board Review 2004

66
Q
  1. The true statements about electromyography and nerve
    conduction velocity:
    A. Electromyographic changes occur within 24 h of neural
    injury
    B. Testing of neural conduction velocity is more sensitive
    than electromyography in the early stages of neural
    injury
    C. Increased motor potential in muscle groups occurs with
    neural injury
    D. Increased neural conduction velocity occurs with neural
    injury
    E. Changes in neural conduction velocity take weeks to
    become apparent after neural injury
A
  1. Answer: B
    Explanation:
    Reference: Tollison, p 326.
    If neural injury is suspected, electromyography and testing
    of neural conduction velocity can provide information as
    to the extent and location of injury.
    With neural injury, a decrease in motor potential in
    muscle groups and slowed conduction velocities occur.
    Neural conduction velocities are decreased quickly after
    neural injury.
    Electromyographic changes may take weeks to occur.
    Therefore, testing of neural conduction velocity is more
    sensitive than electromyography in the early stages of
    neural injury.
    Source: Kahn and Desio
67
Q
  1. Spine myelography:
    A. Contrast is instilled into the epidural space
    B. Contrast is instilled into the subdural space
    C. Contrast is instilled into the subarachnoid space
    D. Myelography is not used for patients with MR contraindications
    E. Myelography is preferable to MR for pregnant patients
A
  1. Answer: C

Source: Bieneman B, Board Review 2005

68
Q
  1. In studying the interaction of psychological stress and
    DNA repair, suppressed DNA repair was found in persons
    with
    A. Chronic stress more often than those with acute stress
    B. Low distress over an extended period of time
    C. A diagnosis of major depression
    D. Increased anxiety and depression from bereavement
    E. Recently diagnosed cancer
A
  1. Answer: C
    Explanation:
    (Baum, pp 194-198.)
    ·In studying peripheral blood lymphocytes (PBLs) from
    patients with a major depression,it was found that they
    had poorer DNA repair (PBLs exposed to radiation
    damage) than lymphocytes obtained from nondepressed
    or low-distressed persons. When patients with a diagnosis
    of major depression were divided into low and highdistress
    subgroups, the PBLs from the high-distress
    subgroup had poorer DNA repair than the PBLs from the
    low-distress subgroup.
    ·While acute stress is immunosuppressive, chronic stress
    over time is associated with adaptation and can even
    enhance immunity.
    ·It is hypothesized that the impact of psychosocial stress
    (distress) on DNA repair could increase cancer risk. While
    the impact of psychosocial stress and DNA repair on the
    initiation of cancer has not been demonstrated, stressinduced
    suppression of the immune system, and
    enhancement of the immune system, has been shown to
    affect the growth and progression of neoplasms.
    ·The increased anxiety and depression from bereavement
    does produce suppressed lymphocyte proliferative
    response to mitogen stimulation 2 to 6 weeks after the
    death of a spouse.
    Source: Ebert 2004
69
Q
  1. A 67-year-old man with lung cancer presents with
    metacarpophalangeal joint pain. On physical
    examination, there is pain on moving his fi ngers and a
    spongy sensation when palpating the proximal aspects of
    the fi ngernails. CHOOSE ACCURATE DIAGNOSIS:
    A. Refl ex sympathetic dystrophy
    B. Ankylosing spondylitis
    C. Reiter syndrome
    D. Hypertrophic osteoarthropathy
    E. Charcot joint
A
  1. Answer: D
    Explanation:
    (Goldman, 21/e, p 1558.)
    Hypertrophic osteoarthropathy
    is nail clubbing accompanied by a symmetrical
    polyarthritis involving the large joints and occasionally
    the metacarpophalangeal joints. Hypertrophic
    osteoarthropathy may be seen secondary to malignancy,
    endocarditis, vasculitis, and other pulmonary and cardiac
    diseases. Ankylosing spondylitis (AS) is a chronic and
    progressive infl ammatory disease, seen mostly in men in
    their thirties, that most commonly affects the spinal,
    sacroiliac, and hip joints. It may go undiagnosed for many
    years, and bilateral hip pain due to sacroiliac involvement
    may be clinically undetectable. It is strongly associated
    with HLA-B27. Examination of the spine usually reveals
    limitation in movement; patients in advanced stages may have a characteristic bent-over posture. Patients with AS
    may present with an acute nongranulomatous uveitis and
    limited chest expansion due to involvement of the
    costovertebral joints. The Schober test is positive in AS
    (with the patient erect, marks are made 5 cm below and 10
    cm above the lumbosacral junction between the posterior
    superior iliac spines; the patient bends, marks are
    measured, and if the distance between the two marks
    increases by less than 4 cm there is spinal immobility). The
    pathogenesis of refl ex sympathetic dystrophy is unknown.
    The presentation may be seen after peripheral limb injury;
    early symptoms include pain in the limb and edema. This
    disorder may lead to contractures. Charcot joint is a
    complication of peripheral neuropathy seen in diabetic
    patients. Repetitive minor trauma to the foot causes
    deformities, which may lead to skin breakdown, erythema,
    edema, and callus formation.
70
Q
  1. Depletion of which neurotransmitter in the substania
    nigra is associated with Parkinson’s disease?
    A. Acetylcholine
    B. Epinephrine
    C. Calcitonin gene-related peptide
    D. Dopamine
    E. Substance P
A
  1. Answer: D

Source: Day MR, Board Review 2006

71
Q
  1. A 42-year-old woman (5 ft, 3in., 170 lb) complains of
    sudden onset of severe pain in the right upper abdomen
    “under the ribs” accompanied by sweating, nausea, and a
    feeling of imminent collapse. The pain lasts for about two
    hours and then persists as a dull ache. When seen several
    hours later, she has normal bowel sounds, is tender
    throughout the abdomen, especially in the right upper
    quadrant, and is faintly icteric. She has noticed her urine
    is darker than usual but has not passed stool recently. She
    recalls occasional episodes of “indigestion” referred to
    the right upper abdomen and radiating to the shoulder.
    This has occurred especially after eating fried foods or
    after eating a meal following a long period of fasting.
    She has no fever but is anxious and tachycardic.The
    tests available are a blood count and blood chemistry
    including liver enzymes, alkaline phosphatase, and
    bilirubin. She has a WBC of 10,000. Her cellular hepatic
    enzymes are: AST/SGOT = 52 (2-33) and ALT/SGPT = 70
    (4 to 44), alkaline phosphatase = 300 (17 to 91), bilirubin
    = 6.3 (0.2 to 1.0).The most probable diagnosis is
    A. Hepatitis A
    B. Intercostal neuritis
    C. Carcinoma of the head of the pancreas
    D. Gallstone obstructing common bile duct
    E. Biliary cirrhosis
A
  1. Answer: D
    Explanation:
    (Braunwald, 15/e, pp 255-259, 1785. Kumar, 6/e, pp 550-
  2. Junqueira, 9/e, pp 318-319. Guyton, l0/e, pp 800-801.)
    The most probable diagnosis is gallstones. The pattern
    of elevated liver enzymes, alkaline phosphatase, and
    bilirubin are consistent with obstructive jaundice (see
    table below). The presence of pain (in the right upper
    quadrant radiating to the shoulder) after eating a meal
    consisting of fried foods makes gallstones the most
    probable diagnosis. Similar pain often occurs in these
    patients when they have not eaten for long periods of time
    and then have a large meal. The pain is caused by the
    obstruction of the cystic duct or common bile duct that
    produces increased lumenal pressure within the bile
    vessels, which cannot be compensated for by
    cholecytokinin-induced contractions. The pain lasts for
    about one to four hours as a steady, aching feeling.
    Source: Klein RM and McKenzie JC 2002.
72
Q
  1. Which of the following words is defi ned as A chronic
    preoccupation with obtaining the substance of choice
    and misuse or overuse of the substance despite negative
    consequences?
    A. Tolerance
    B. Physical dependence
    C. Pseudo addiction
    D. Psychological dependence
    E. Addict
A
  1. Answer: D

Source: Day MR, Board Review 2006

73
Q
811. What is the most frequent initial site of metastatic tumor
spread to the spine?
A. Bone marrow
B. Vertebral pedicle
C. Nucleus pulposus
D. Epidural space
E. Posterior elements
A
  1. Answer: A
    Explanation:
    Because of its high vascularity, bone marrow is involved
    fi rst, with extension to pedicle and posterior elements.
    Extension to epidural space may occur from vertebra or
    through foramen.
    Source: Bieneman B, Board Review 2005
74
Q
812. Which of the following type A, or coronary-prone,
behavioral factors appears to be the best predictor of
coronary heart disease?
A. Hostility
B. Competitiveness
C. Time Urgency
D. Explosive speech
E. Hyperactivity
A
  1. Answer: A
    Explanation:
    (Baum, pp 144-148.)
    Among the psychosocial variables
    considered to be risk factors for coronary heart disease, the type behavior pattern is most prominent. The type A
    behavior pattern is most prominent. The type A behavior
    pattern consists of extremes of competitiveness, a chronic
    sense of time urgency, easily evoked hostility,
    aggressiveness, explosive speech, and increased rate of
    activity. More recent studies have shown that
    aggressiveness and hostility (especially unexpressed
    hostility) are the most consistent and important factors
    Source: Ebert 2004
75
Q
813. The most common cause of peripheral neuropathy is:
A. Idiopathic
B. Diabetes Mellitus
C. Nutritional Defi ciencies
D. ETOH
E. None of the above
A
  1. Answer: B

Source: Wirght PD, Board Review 2004

76
Q
814. In the clinical assessment of neuropathic pain, which
procedure(s) should be included in the diagnostic
workup?
A. EMG and/or NCV
B. Laboratory evaluations
C. Imaging studies
D. All of the above
E. None of the above
A
  1. Answer: D

Source: Giordano J, Board Review 2003

77
Q
  1. The best defi nition for a vertebral disc bulge is which of
    the following?
    A. An intraverterbral disk herniation (Schmorl’s node)
    B. Disruption of concentric fi bers of the annulus fi brosis
    C. Generalized extension of disc material beyond the edge
    of the vertebra involving 180°
    D. Localized displacement (
A
  1. Answer: C

Source: Bieneman B, Board Review 2005

78
Q
  1. A complete electrodiagnostic evaluation would include
    all the following EXCEPT
    A. Electromyography and late response studies
    B. Peripheral nerve conduction studies of motor and sensory
    nerves
    C. Muscle biopsy
    D. Somatosensory evoked potentials
    E. Radiologic evaluation
A
  1. Answer: C
    Explanation:
    Reference: Bonica, p 629.
    The electromyogram (EMG), peripheral nerve conduction
    studies (NCSs), late response studies, and somatosensory
    evoked potentials (SEPs) help to characterize the nature
    and location of the abnormality being studied.
    Determination of the cause of the abnormality can occur
    only after integration of the information obtained from
    the physical examination, history, and electrodiagnostic
    and radiologic studies.
    Muscle biopsy is not a component of electrodiagnostic
    evaluation.
    Source: Kahn and Desio
79
Q
817. A celiac-plexus block would not effectively treat pain
resulting from a malignancy involving the following
organs:
A. Ureter
B. Adrenal gland
C. Stomach
D. Pancreas
E. Gallbladder
A
  1. Answer: A
    Explanation:
    The celiac-plexus innervates most of the abdominal
    viscera, including the pancreas, liver, spleen, kidneys,
    adrenal glands, biliary tract, omentum, and small and large
    bowel.
    The pelvic organs are supplied by the hypogastric
    plexus.
80
Q
  1. Traditional psychotherapy emphasizes all of the following
    EXCEPT:
    A. Formation of defense mechanisms
    B. Psychosexual development
    C. The role of environmental reinforcement
    D. The therapeutic process of transference
    E. The relationship between conscious and unconscious
    processes
A
  1. Answer: C

Source: Janata JW, Board Review 2005

81
Q
  1. Positive reinforcement refers to:
    A. A stimulus that increases the likelihood that a certain
    behavior will be maintained or repeated
    B. A consequence that decreases the likelihood that a certain
    behavior will be maintained or repeated
    C. A consequence that increases the likelihood that a certain behavior will be maintained or repeated
    D. A stimulus that decreases the likelihood that a certain
    behavior will be maintained or repeated
    E. A consequence that does not infl uence the likelihood that
    a certain behavior will be maintained or repeated
A
  1. Answer: C

Source: Janata JW, Board Review 2005

82
Q
820. Which of the following medications does not potentiate
opioid respiratory depression?
A. Dexmetetomidine
B. Methohexital
C. Etomidate
D. Diazepan
E. Propofol
A
  1. Answer: A

Source: Day MR, Board Review 2005

83
Q
  1. Understanding the complex interaction of a patient’s pain
    and mood is best accomplished by:
    A. Integrating medical and psychological data from a variety
    of sources
    B. Performing a pain tolerance test in the laboratory
    C. Having the patient complete the Michigan Pain States
    Inventory (MPSI)
    D. Asking offi ce staff to observe waiting room pain behavior
    E. Obtaining family history of alcoholism
A
  1. Answer: A

Source: Janata JW, Board Review 2005

84
Q
822. Personality disorders can be readily assessed by:
A. Clinical interview
B. Careful history-taking
C. Utilizing the Symptom Checklist - 90
D. Observing waiting room behavior
E. MMPI or MCMI
A
  1. Answer: E

Source: Janata JW, Board Review 2005

85
Q
  1. A “fake bad” profi le can be estimated by using the:
    A. Medical Outcomes Survey (MOS)
    B. Sickness Impact Profi le (SIP)
    C. Minnesota Multiphasic Personality Inventory (MMPI)
    D. Coping Strategies Questionnaire
    E. Spielberger State-Trait Anxiety Inventory (STAI)
A
  1. Answer: C

Source: Janata JW, Board Review 2005

86
Q
  1. Which of the following statements regarding the superior
    hypogastric plexus block is not true?
    A. It is most appropriate for pelvic pain of visceral origin
    B. It is associated with few side effects
    C. It must be performed with the assistance of fl uoroscopy
    D. It is most appropriate for upper abdominal pain
    E. It must be performed at L5
A
  1. Answer: D
    Source: Raj P, Pain medicine - A comprehensive Review -
    Second Edition
87
Q
  1. A 30-year old patient presents with foot pain. She was
    diagnosed with a calcaneal heel spur. Non-steroidal antiinfl
    ammatory agents failed to provide her any signifi cant
    relief. Appropriate treatment is:
    A. Soft padding of the shoe
    B. Local steroid injection
    C. Strengthening exercises
    D. Surgical excision of spur
    E. Stretching exercises in combination with ultrasound
A
  1. Answer: A
    Explanation:
    Goals of therapy include controlling the abnormal
    biomechanics of the foot, decreasing the infl ammatory
    condition, and improving the fl exibility. Various
    modalities of treatments mentioned include the
    following:
    Non-steroidal anti-infl ammatory agent
    Rest
    Night splint
    Padding
    Physical therapy with stretching and strengthening
    exercises
    Physical therapy with ultrasound
    Orthosis
    Steroid injection
    Surgical removal
88
Q
  1. Spinal Shock:
    A. Occurs weeks to months after initial injury
    B. Is frequently associated with autonomic dysfunction
    C. Will usually result in full recovery of function
    D. All of the above
    E. None of the above
A
  1. Answer: B

Source: Wirght PD, Board Review 2004

89
Q
  1. The Mutidimensional Pain Inventory (MPI):
    A. Is a projective test
    B. Assesses malingering
    C. Yields depression and anxiety scores
    D. Utilizes three profi le types - Dysfunctional, Interpersonally
    Distressed and Adaptive Coper.
    E. Contains 576 true-false questions
A
  1. Answer: D

Source: Janata JW, Board Review 2005

90
Q
828. Factitious disorder is motivated by:
A. Secondary gain
B. Assumption of the sick role
C. Financial reward
D. Evasion of responsibility
E. External incentive
A
  1. Answer: B

Source: Janata JW, Board Review 2005

91
Q
829. Which of the following is most appropriate for the initial
treatment of tension headache?
A. Acetaminophen
B. Amitriptyline
C. Gabapentin
D. Oxycodone
E. Tramadol
A
  1. Answer: A
92
Q
  1. A thoracic epidural is placed at T10 after abdominal
    surgery. A test dose is given and the patient becomes
    numb above the incision for 2 hours. An epidural
    catheter is inserted and an infusion is started. After 24
    hours, she develops abdominal pain over 30 minutes.
    Your next course of action is:
    A. Notify surgeon
    B. Give IV NSAIDs
    C. Give IV morphine
    D. Test the epidural
    E. Apply TENS unit
A
  1. Answer: D
93
Q
  1. A 41-year-old man was recently in a motor vehicle
    accident (MVA) where he was the driver. He states he was
    wearing his seat belt at the time of the accident. A day
    after the accident, he developed neck pain that has now
    continued for 10 days. He notices crunching on extension
    and lateral bending of the neck. On physical examination,
    the patient has no neurologic defi cits. His neck has no
    areas of tenderness and there are no areas of spasm. He
    has normal lateral bend, extension, and fl exion of the
    neck. Which of the following is the most likely diagnosis?
    A. Ankylosing spondylitis
    B. Osteoarthritis
    C. Reiter syndrome
    D. Whiplash
    E. Wry neck
A
  1. Answer: D
    Explanation:
    (Tierney, 42/e, p 792.)
    The most likely diagnosis in this patient is whiplash or
    cervical musculoligamental sprain or strain. Whiplashassociated
    disorders begin after a symptom-free period
    following a hyperextension or hyperfl exion injury, usually in an MVA. It is vital to perform a complete neurologic
    examination to exclude other causes of neck pain.
    Ankylosing spondylitis is a chronic and progressive
    infl ammatory disease that most commonly affects spinal,
    sacroiliac, and hip joints.
    Osteoarthritis most often affects the weight-bearing joints.
    Reiter syndrome usually causes an arthritis of the hips, and
    there is often a history of urethritis, conjunctivitis, and
    foot involvement.
94
Q
832. The appropriate initial treatment for mild mandibular
pain following oral surgery is
A. Nonsteroidal antiinfl ammatory drug
B. Mandibular nerve block
C. Acetaminophen
D. Oxycodone
E. Gabapentin
A
  1. Answer: A
95
Q
  1. A 29-year old female with upper extremity complex
    regional pain syndrome undergoes a stellate ganglion
    block in your offi ce pain clinic. She is otherwise healthy
    with normal body habitus and normal airway. She has
    been NPO for 12 hours. 20cc of 0.25% bupivacaine is
    injected incrementally over one minute with no other
    medication administered. 5 minutes after injection
    the patient complains of generalized weakness which
    progresses to complete unresponsiveness, apnea and
    hypotension over the ensuing several minutes. Eight
    minutes after injection, the patient continues to be
    completely unresponsive and apneic with a systolic blood
    pressure of 50. ECG monitor shows sinus bradycardia.
    The patient has no IV. Your fi rst action should be:
    A. Start an IV
    B. Use an ambu bag to ventilate the patient
    C. Intubate the patient
    D. Administer subcutaneous epinephrine
    E. Begin CPR
A
  1. Answer: B
    Explanation:
    In resuscitation scenarios, always remember the ABCs:
    airway
    breathing
    circulation
    Immediate control of the airway is the most important
    and pressing concern in any arrest situation. This patient is
    completely apneic therefore airway management takes
    precedence over circulatory compromise. Since the patient
    has a normal airway and has been NPO, there is no
    immediate need to intubate if the airway can be
    maintained easily with a mask and ambu bag. The patient
    will likely regain the ability to ventilate on her own in a
    relatively short period of time (1-2 hours).Intubation
    in this patient would introduce a number of additional
    concerns including the potential for airway damage, the
    potential for bronchospasm, and issues related to timing
    of extubation.
    Intravenous access and epinephrine are circulation
    interventions. Starting an intravenous line is important
    but only after the airway is secure. Epinephrine is an
    extremely potent sympathomimetic and should be used
    very cautiously. It has the potential to cause severe
    hypertension and tachycardia in a patient who is not in
    full cardiac arrest (when the patient is in full cardiac arrest,
    epinephrine is indicated per ACLS guidelines). This
    patient is hypotensive and bradycardic secondary to
    sympathetic blockade. Intravenous volume perhaps with
    the edition of a milder vasopressor such as ephedrine is a
    better choice than epinephrine.
    Source: Schultz D, Board Review 2004
96
Q
834. Tissues that are more echogenic on sonography:
A. Appear darker
B. Include cysts
C. Have more refl ective surfaces
D. Do not include fat
E. Include moving blood
A
  1. Answer: C

Source: Bieneman B, Board Review 2005

97
Q
835. Mixed Upper and Lower Motor Neuron fi ndings can be caused by:
A. Vit B12 defi ciency
B. Nitrous Oxide Exposure
C. Cervical Spinal Stenosis
D. All of the above
E. None of the above
A
  1. Answer: D

Source: Wirght PD, Board Review 2004

98
Q
836. A L4-5 left paracentral disc protrusion will likely affect
which nerve root?
A. The left L5 nerve root
B. The left L4 nerve root
C. The left L4 and L5 nerve roots
D. The bilateral L5 nerve roots
E. The bilateral L4 nerve roots
A
  1. Answer: A

Source: Bieneman B, Board Review 2005

99
Q
  1. Which of the following is the most appropriate initial
    examination to evaluate for disc herniation?
    A. Enhanced MR
    B. Unenhanced MR
    C. Enhanced CT
    D. Unenhanced CT
    E. CT Myelogram
A
  1. Answer: B

Source: Bieneman B, Board Review 2005

100
Q
838. Aortic dissection is best diagnosed by what test?
A. Unenhanced CT
B. Ultrasound
C. Unenhanced and enhanced CT
D. MRI
E. Angiography
A
  1. Answer: C
    Explanation:
    Aortic dissection is an acute abdominal emergency and
    best imaged with CT due to the speed of examination and
    ability to characterize the type of dissection without delay.
    MRI and ultrasound may provide appropriate imaging of
    an aortic dissection in the non acute setting. Angiography
    is reserved for cases with clinical questions not answered
    by the initial imaging modality and for cases where further
    intervention is planned such as fenestration of a dissection
    of placement of a stent graft for aortic aneurysm
    Source: Bieneman B, Board Review 2005
101
Q
839. Increased activity on bone scintigraphy may be from all except:
A. Healing fracture
B. Prostate metastases
C. Stress fracture
D. Interrupted sympathetic nerve supply
E. Old orthopedic hardware
A
  1. Answer: E

Source: Bieneman B, Board Review 2005

102
Q
840. Vertebral discitis and osteomyelitis are best imaged by
A. MRI
B. Bone scan
C. CT-myelogram
D. Plain radiographs
E. Duplex ultrasound
A
  1. Answer: A

Source: Bieneman B, Board Review 2005

103
Q
  1. Bilateral cingulumotomy is a properly selected therapy in nociceptive back pain.
    A. If the patient has no obsessive compulsive feature
    B. In cancer patients who have no other option
    C. In most benign pain patient
    D. If the patient hass failed oral opiate analgesics and tricyclic
    antidepressant
    E. Two of the above
A
  1. Answer: B

Source: Feler C, Board Review 2005

104
Q
842. Tissues that appear denser on plain radiographs have:
A. More electron density
B. Less electron density
C. More neutron density
D. Less neutron density
E. None of the above
A
  1. Answer: A

Source: Bieneman B, Board Review 2005

105
Q
  1. Which of the following statements about DREZ lesions
    is correct?
    A. DREZ is currently recommended in the treatment of
    PHN
    B. DREZ lesions typically produce sensory loss that improves
    with time
    C. The techniques used to create DREZ lesions include
    ultrasonic aspiration
    D. Results of DREZ lesioning are predictable in all neuropathic
    pain state
    E. All of the above
A
  1. Answer: B

Source: Feler C, Board Review 2005

106
Q
844. Houndsfield units on CT:
A. Are a measure of enhancement
B. Water=200 HU
C. Acute hemorrhage = -100 HU
D. Are a measure of density
E. Air=5 HU
A
  1. Answer: D
    Explanation:
    (See lecture notes)
    Source: Bieneman B, Board Review 2005
107
Q
  1. In the fi eld of electromyography:
    A. The blink refl ex assesses the integrity of peripheral and
    central circuitry involving CN5 and CN7
    B. Presence of a blink refl ex is suggestive of frontal lobe
    dementia
    C. Repetitive stimulation is a technique unlikely to detect
    pathology at the NMJ
    D. Myopathies tend to be characterized by a reduced pattern
    of recruitment
    E. Neuropathies tend to be characterized by an early pattern of recruitment
A
  1. Answer: A

Source: Goodwin J, Board Review 2005

108
Q
  1. The following statements regarding TENS are true
    except:
    A. It has been used very successfully in the treatment of
    acute pain.
    B. It works by stimulating A-beta fi bers which in turn close
    the dorsal horn “gate” to nociceptive input.
    C. TENS has been shown to increase blood fl ow in the
    stimulated region
    D. The effects of TENS occur only during the stimulation
    period
    E. Conventional TENS uses a nearly continuous high frequency
    stimulation (60-100 Hz) and a relatively low
    intensity current (10-30 milliamps)
A
  1. Answer: D
    Explanation:
    Reference:
    Bonica’s Management of Pain, Third Edition, Chapter 98,
    Transcutaneous Electrical Nerve Stimulation
    TENS works by stimulating A-beta fi bers which in turn
    close the dorsal horn “gate” to nociceptive input. It has
    been shown in many well-controlled studies to be highly
    effective for acute pain. The results of TENS for chronic
    pain have been less impressive although there is data to
    suggest that TENS is helpful for well selected patients with
    chronic pain. Conventional TENS uses a nearly
    continuous high frequency stimulation (60-100 Hz) and a
    relatively low intensity current (10-30 milliamps). The
    effects of TENS have been shown to increase blood fl ow in
    the stimulated area with pain relief sometimes outlasting
    the period of stimulation by hours to days.
    Source: Schultz D, Board Review 2004
109
Q
847. Which of the following is most appropriate for a patient
with end-stage rectal cancer?
A. Lissauer tractotomy
B. Cingulotomy
C. Hypophysectomy
D. Commisural myelotomy
E. Percutaneous C1-2 cordotomy
A
  1. Answer: D
    Explanation:
    (Raj 2nd Ed., page 313)
    Midline or commissural myelotomy sections those
    midline fi bers just dorsal to the central canal of the spinal
    cord. The original intent was to lesion crossing
    spinothalamic neurons, which would eliminate pain, but
    preserve sensory function. However, pain relief extended
    caudally, without demonstrable caudal analgesia. This lead
    several investigators to postulate several alternate pain
    pathways. A multisynaptic short tract afferent pathway or
    an anterior tract located in between the posterior columns
    were proposed. The latter mediate pelvic and epigastric
    visceral pain. Nonetheless, myelotomy is indicated for
    bilateral pelvic and perineal pain of malignant origin.
    Unilateral percutaneous cordotomy is among the most
    useful procedures for unilateral cancer pain below C5. It
    targets the spinothalamic tract. Radiofrequency energy is
    used. Electrical stimulation (sensory to obtain a feeling of
    warmth or coolness on the contralateral side and motor to
    obtain ipsilateral cervical muscles; ipsilateral contraction
    of muscles below the neck implies the probe is in the
    corticospinal tract) is used to identify the lesion target
    Lissauer tractotomy is the goal of the dorsal root entry
    zone (DREZ) procedure…but all dorsal horn lamina (I-V)
    may be affected. The DREZ lesion is classically indicated
    for central nervous system damage related pain: brachial
    plexus avulsion, stump pain, spinal cord injury pain.
    Hypophysectomy is recommended in the treatment of
    metastatic prostate and breast cancer, irrespective of the
    hormonal responsiveness of the tumors. The analgesic
    mechanism is unknown, but limbic system or
    psychological effects are unlikely to be the reasons for pain
    relief.
    Cingulotomy, anterior capsulotomy (anterior limb of
    internal capsule), leucotomy (pre-frontal lobotomy), and
    hypothalamotomy have been used for intractable cancer
    pain in multiple sites and for psychiatric disorders, such as
    obsessive compulsive disorders.
    Source: Schultz D, Board Review 2004
110
Q
  1. Deep brain stimulation to treat primarily nociceptive
    pain would most likely target the:
    A. Periaqueductal grey
    B. Ventroposterolateral or Ventroposteromedial thalamus
    C. Caudalis subnucleus
    D. Nucleus gracilis
    E. Reticular formation
A
  1. Answer: A
    Explanation:
    (Raj, Pain Review 2nd Ed., pages 311; Bonica 3rd Ed.,
    pages 122, 130-2, & 153-4, ;
    Raj, Practical Mgmt of Pain 3rd Ed., pg 795)
    The periaqueductal and periventricular grey are located in
    the midbrain. The PAG and PVG can be excited by
    endogenous opioids or electrical stimulation to initiate
    descending antinociception. The VPM and VPL located in
    the thalamus are useful for deafferentation or neuropathic
    pain. The caudalis subnucleus, also known as the
    trigeminal spinal nucleus, is thought to be integral for a
    variety of head pain syndromes. The reticular formation
    may be responsible for some of the affective and
    motivational responses to pain and the regulation of spinal
    motor, respiratory, and autonomic functions: arousal and
    escape. Pos-synaptic touch and proprioception fi bers (dorsal column) project to the dorsal column nuclei:
    cuneatus and gracilis: lumbar and thoracic fi bers to the
    gracilis and cervical to the cuneatus.
    Source: Schultz D, Board Review 2004
111
Q
  1. All of the following are typically associated with the
    technical aspects of epidural anesthesia, except:
    A. Paramedian
    B. Bromage grip
    C. Hanging drop
    D. Taylor approach
    E. Sacral hiatus.
A
  1. Answer: D
    Explanation:
    (Raj, Practical Mgmt of Pain, 3rd Ed, pages 641-6, 634)
    A. Midline and paramedian approaches to the epidural
    space have been described.
    B. The Bromage grip is a useful technique for slow,
    controlled advancement of an epidural needle towards the
    ligamentum fl avum.
    The needle is fi rmly gripped between the thumb and
    index fi nger of the nondominant hand.
    The dorsum of the wrist is placed against the patient’s
    back.
    The needle is advanced by extension of the wrist while
    the dominant hand provides intermittent or constant
    pressure on the plunger, depending on whether one uses
    the loss-of-resistance technique to air or to saline solution,
    respectively.
    C. The negative pressure often found within the epidural
    space is the basis for the hanging-drop technique.
    This hanging-drop sign of Gutierrez is used to
    identify the epidural space and is usually applied
    for cervical epidural blockade in the seated patients.
    A winged needle is usually used and is advanced with
    both hands, as with the intermittent technique.
    A drop of fl uid is placed at the end of the needle once
    it is anchored in the interspinous ligament.
    Because of the persistent subatmospheric pressure
    within the epidural space, penetration of the ligamentum
    fl avum and entrance of the epidural space cause the drop
    to be sucked into the hub of the epidural needle.
    Injection of air or fl uid without resistance confi rms
    the position in the epidural space.
    D. The Taylor is used to identify the subarachnoid space by
    way of the L5 interspace, which is the largest interspace in
    the vertebral column.
    To enter this space, the operator introduces the spinal
    needle through the skin wheal approximately 1cm medial
    and 1cm inferior to the posterior superior iliac spine.
    The spinal needle is directed medial and cephalad to
    enter the subarachnoid space at the midline at the L5-S1
    interspace.
    E. Identifi cation of the sacral hiatus is important for
    caudal epidural procedures.
    The sacral hiatus is formed secondary to nonfusion of
    the fi fth sacral vertebral arch.
    The hiatus is covered by the sacrococcygeal membrane and
    bordered by two cornua (large bony processes on each side
    of the hiatus).
    The sacral hiatus is most easily identifi ed with the
    patient lying in the prone or lateral position.
    Firm pressure is used to identify the coccyx with the
    nondominant index fi nger.The fi rst pair of bony
    protuberances in moving cephalad are the two cornua,
    surrounding the sacral hiatus.
    Source: Shah RV, Board Review 2005
112
Q
850. Back pain developing after spine surgery is best imaged by
A. Unenhanced MR
B. Contrast Enhanced MR
C. Unenhanced CT
D. Enhanced CT
E. Myelography
A
  1. Answer: B

Source: Bieneman B, Board Review 2005

113
Q
  1. What is the co-morbid condition of body dysmorphic
    disorder?
    A. Depression
    B. Multiple recurrent somatic complaints without medical
    findings
    C. Delusion
    D. “La belle indifference”
    E. Fear of having a serious illness despite adequate medical evaluation
A
  1. Answer: C
    Explanation:
    (A) Major depression is a comorbid condition of both
    somatization disorder and hypochondriasis, but not a
    major diagnostic feature.
    (B) Somatization disorder is characterized by the
    recurrence of multiple somatic complaints not accounted
    for by medical fi ndings. It is a chronic condition with
    female predominance.
    (C) Delusion is the common feature of body dysmorphic
    disorder.
    (D) La belle indifference is an associated feature of
    conversion disorder, where symptoms do not conform to
    anatomic pathways. Delusional disorder may be a
    comorbid condition in body dysmorphic disorder.
    (E) Hypochondriasis is a chronic condition characterized
    by a fear or belief that one has a serious illness despite
    adequate medical evaluation. Its prevalence is 4% to 9% of
    medical outpatients with equal incidence between men
    and women.
    Source: Laxmaiah Manchikanti, MD
114
Q
852. The dorsal columns of the spinal cord primarily carry:
A. Pain sensation
B. Temperature sensation
C. Spinothalamic tracts
D. All of the above
E. None of the above
A
  1. Answer: E

Source: Wirght PD, Board Review 2004

115
Q
853. In a patient with midthoracic back pain who reports
tenderness to palpation over the T-6 vertebral body, the
most likely diagnosis is:
A. Thoracic disk herniation
B. Metastatic neoplasm
C. Facet osteoarthropathy
D. Rheumatoid arthritis
E. Epidural hematoma
A
  1. Answer: B
    Explanation:
    Local spine tenderness elicited when palpating directly
    over the vertebral body is highly suggestive of vertebral
    body neoplasm or infection. Neoplastic conditions or
    infectious-infl ammatory disorders (osteomyelitis) may
    distend the periosteum, causing local tenderness. This
    discrete local tenderness should be differentiated from
    more diffuse muscle spasm seen with a herniated disk.
    Source: Neurology for the Psychiatry specialty Board
    Review By Leon A. Weisberg, MD
116
Q
854. Which of the following sedative - hypnotic medications
should not be used in a patient with increased intracranial
pressure?
A. Thiopental
B. Etomidate
C. Ketamine
D. Propofol
E. Midazolam
A
  1. Answer: C

Source: Day MR, Board Review 2006

117
Q
  1. A “fake bad” profile is provided in scoring the:
    A. Spielberger State-Trait Anxiety Inventory (STAI)
    B. Minnesota Multiphasic Personality Inventory (MMPI)
    C. Coping Strategies Questionnaire
    D. Medical Outcomes Survey (MOS)
    E. Sickness Impact Profi le (SIP)
A
  1. Answer: C

Source: Janata J, Board Review 2006

118
Q
  1. Which of the following is not defi ned as a disc
    herniation?
    A. Protrusion
    B. Localized displacement of disc material beyond the confi
    nes of the disc space
    C. Bulge
    D. Extrusion
    E. Intravertebral end plate disruption secondary to disc
    material
A
  1. Answer: C
    Explanation:
    A disc bulge is not a herniation
    Source: Bieneman B, Board Review 2005
119
Q
  1. Understanding the complex interaction of a patient’s
    pain and mood is best accomplished by:
    A. Obtaining family history of depression
    B. Asking offi ce staff to observe waiting room pain behavior
    C. Having the patient complete the Michigan Pain States
    Inventory (MPSI)
    D. Integrating medical and psychological data from a variety
    of sources
    E. Performing a pain tolerance test in the laboratory
A
  1. Answer: E

Source: Janata J, Board Review 2006

120
Q
  1. Mesencephalic tractotomy is indicated in patients who
    suffer from the pain of head and neck cancer.
    A. If there is no preoperative neurologic defi cit
    B. If the neck pain does not come higher than the C5 segment
    C. If the patient has a preoperative myelopathy
    D. If the pain involves the lower face
    E. Two of the above
A
  1. Answer: E

Source: Feler C, Board Review 2005

121
Q
859. Which of the following drugs shows a good correlation between the blood level and the clinical effect?
A. Phenelzine
B. Trazodone
C. Fluoxetine
D. Paroxetine
E. Imipramine
A
  1. Answer: E
    Explanation:
    Blood level can be obtained for all antidepressant drugs.
    But not all of them have shown a correlation between the
    therapeutic effect and the blood level.
    Plasma level measurements of imipramine,
    desmethylimipramine, and nortriptyline are unequivocally
    clinically useful in certain situations. For imipramine, the
    percentage of favorable responses correlates with plasma
    levels in a linear manner between 200 and 250 ng/mL, but
    some patients may respond at a lower level. At levels that
    exceed 250 ng/mL, there is no improved favorable
    response, and side effects increase.
    Source: Laxmaiah Manchikanti, MD
122
Q
  1. Some physiological consequences of epidural blockade
    may include:
    A. increased peristalsis
    B. interference with satiety
    C. impaired respiratory function, by modifying respiratory
    drive, reducing diaphragmatic contractility, permitting
    increased airway hyperactivity, and impairing V/Q
    mismatch
    D. hypertension
    E. degree of sympathetic blockade that correlates with the
    degree of sensory blockade
A
  1. Answer: A
    Explanation:
    (Raj, Practical Mgmt of Pain, 3rd Ed, pages 639-41)
    A. The gastrointestinal tract is innervated by both the
    sympathetic and parasympathetic systems.Visceral afferent
    parasympathetic fi bers transmit sensations of satiety,
    distention, and nausea (but not pain).
    Parasympathetic efferent outfl ow increases tonic
    contraction, sphincter tone, peristalsis, and secretions.
    Pain is mediated via sympathetic afferents, whereas
    sympathetic efferent fi bers inhibit peristalsis and gastric
    secretions, constrict vasculature, and increase sphincter
    tone.
    Sympathetic denervation of the gastrointestinal tract
    by neuroaxial blockade conceivably may lead to
    generalized contraction of the bowel secondary to
    unopposed parasympathetic efferent outfl ow.
    The degree to which the bowel is affected after neural
    blockade depends on the extent of the blockade.
    C. It is conceivable that a high thoracic or cervical block
    may impair respiratory function by affecting sensory
    function (modifying respiratory drive), motor function (decreasing abdominal muscles, intercostal muscles, and
    diaphragmatic strength), and sympathetic function
    (unopposed cholinergic tone can lead to hyperreactive
    airways).
    Sympathetic block may diminish pulmonary blood fl ow
    and ventilation-perfusion ( V/Q ) mismatch.
    All of these changes have the potential to lead to airway
    closure, atelectasis, decreased blood fl ow, and diminished
    functional reserve capacity (FRC), causing (V/Q)
    mismatch and hypoxemia.
    Experimental evidence, however, does not support this
    scheme.
    D. The principal cardiovascular consequences of extensive
    epidural blockade are hypotension and bradycardia.
    E.The extent of sympathetic blockade, however, correlates
    very poorly with the sensory level. The amount of
    sympathetic denervation and sensory blockade might be
    larger than the classically taught two levels.
    It has been demonstrated that the sympathetic block
    could extend six or more spinal segments above the level
    of sensory blockade.
    Source: Shah RV, Board Review 2005
123
Q
  1. What is the major diagnostic feature of somatization
    disorder?
    A. Depression
    B. Multiple recurrent somatic complaints without medical
    fi ndings
    C. Delusion
    D. “La belle indifference”
    E. Fear of having a serious illness despite adequate medical evaluation
A
  1. Answer: B
    Explanation:
    (A) Major depression is a comorbid condition of both
    somatization disorder and hypochondriasis, but not a
    major diagnostic feature.
    (B) Somatization disorder is characterized by the
    recurrence of multiple somatic complaints not accounted
    for by medical fi ndings. It is a chronic condition with
    female predominance.
    (C) Delusion is not a common feature of either
    somatization disorder or hypochondriasis.
    (D) La belle indifference is an associated feature of
    conversion disorder, where symptoms do not conform to
    anatomic pathways. Delusional disorder may be a
    comorbid condition in body dysmorphic disorder.
    (E) Hypochondriasis is a chronic condition characterized
    by a fear or belief that one has a serious illness despite
    adequate medical evaluation. Its prevalence is 4% to 9% of
    medical outpatients with equal incidence between men
    and women.
    Source: Laxmaiah Manchikanti, MD
124
Q
  1. Which one of the following drugs causes Grand mal
    seizure as the most prominent side effect?
    A. Venlafaxine (Effexor®)
    B. Phenelzine (Nardil®)
    C. Fluoxetine (Prozac®)
    D. Amitriptyline (Elavil®)
    E. Bupropion (Wellbutrin®)
A
  1. Answer: E
    Explanation:
    Bupropion is associated with grand ,al seizures in
    approximately 0.4% (4/1000) of patients treated at doses
    up to 450 mg/day.This incidence of seizures may exceed
    that of other marketed antidepressants by as much as 4-fold. This relative risk is only an approximate estimate
    because of the lack of direct comparative studies. The
    estimated seizures incidence for Bupropion increases
    almost 10-fold between 450 and 600 mg/day, which is
    twice the usually required daily dose (300 mg).
    Source: Laxmaiah Manchikanti, MD
125
Q
863. Electroconvulsive therapy is least likely to be successful in which of the following diseases?
A. Major depression
B. Acute schizophrenia
C. Acute manic episodes
D. Chronic schizophrenia
E. Obsessive-compulsive disorder
A
  1. Answer: D
    Explanation:
    Catatonia, mania, major depression, and acute
    schizophrenia are established indications of
    electroconvulsive therapy (ECT). Other indications of
    electroconvulsive therapy with less evidence of its
    effectiveness include Parkinson disease, obsessivecompulsive
    disorder, neuroleptic malignant syndrome,
    and intractable epilepsy
    Source: Laxmaiah Manchikanti, MD
126
Q
864. The CAGE questionnaire is used in case of
A. Mental retardation
B. Bipolar disorder
C. Major depression
D. Opioid abuse
E. Alcohol abuse
A
  1. Answer: D
    Explanation:
    Four clinical interview questions, the CAGE questions,
    have proved useful in helping to make a diagnosis of
    alcoholism. The questions focus on Cutting Down,
    Annoyance by Criticism, Guilty Feeling, and Eye-Openers.
    The acronym “CAGE” helps the physician recall the
    questions:
    “C”: Have you ever felt you should cut down on your
    drinking?
    “A”: Have people annoyed you by criticizing your
    drinking?
    “G”: Have you ever felt bad or guilty about your drinking?
    “E:: Have you ever had a drink fi rst thing in the morning
    to steady your nerves or to get rid of a hangover?
    Source: Laxmaiah Manchikanti, MD
127
Q
  1. Cordotomy is most useful in treating patients with pain
    complaints involving the extremities.
    A. The open procedure is a lesion of the anterior cord.
    B. The lesion is made in the anterior spinothalamic tract.
    C. The lesion is made in the intermediolateral cell column
    D. Results are optimal if the procedure is done bilaterally
    E. The percutaneous procedure gives excellent relief of
    pain to the C2 segment
A
  1. Answer: A

Source: Feler C, Board Review 2005

128
Q
  1. The following is true regarding cordectomy operations:
    A. An optimally selected patient has normal preoperative
    neurologic function
    B. It is most useful in pain of the upper extremities
    C. It is a commonly performed procedure in patients who
    have cancer pain
    D. A patient with a preoperative transverse myelopathy is
    well selected for the procedure
    E. Two of the above
A
  1. Answer: D

Source: Feler C, Board Review 2005

129
Q
  1. Which of the following statements about major
    depression is TRUE?
    A. Thirty percent of individuals with a single episode of
    major depression develop bipolar disorder
    B. The lifetime prevalence rates for adult men range from
    3% to 9%.
    C. Full recovery from major depression occurs in 25% of
    patients by 6 months
    D. Relapse after a single episode is about 50%.
    E. The average age of onset of unipolar major depression is
    50 years
A
  1. Answer: D
    Explanation:
    A. Five to ten percent of individuals with a single episode
    of major depression will eventually develop bipolar
    disease.
    B. The National Comorbidity Survey carried out a
    structured psychiatric interview of a representative sample
    of the general population and reported a lifetime rate of
    major depression of 21.3% in women and 12.7% in men
    producing a female-to-male ration of 1.0 to 1.7. A gender
    difference was found beginning in early adolescence and
    persisting through the mid-50s. Although this increased
    tendency for depression in women refl ects a long-term
    trend,over the short term, an increase has also been seen in
    the rate of depression among young women. The highest
    rate occurs in adult women aged more than 44 years
    C. 50% of cases of major depression will have full
    recovery by 6 months.
    D. Major depression is a recurrent illness; the risk of
    relapse after one episode is about 50%, whereas it is
    greater than 80% after 3 episodes. The average lifetime
    number is 4.
    E. The average age of onset of unipolar depression is 29
    years
    Source: Laxmaiah Manchikanti, MD
130
Q
868. The catheter location for continuous infusion for post-op pain relief for lower abdominal surgery should be
A. T2-8
B. T4-L1
C. T10 - L3
D. T12 - L3
E. L1 - L3
A
  1. Answer: C

Source: Raj, Pain Review 2nd Edition

131
Q
  1. Four days after a left total hip arthroplasty, an obese a
    62-year-old woman complains of severe back pain in the
    region where the epidural was placed. Over the ensuing
    48 hours, the back pain gradually worsens and a severe
    aching pain that radiates down the left leg to the knee
    develops. The most likely diagnosis is
    A. Epidural abscess
    B. Epidural hematoma
    C. Anterior spinal artery syndrome
    D. Arachnoiditis
    E. Meralgia paresthetica
A
  1. Answer: A
    Explanation:
    A.Epidural abscess is an exceedingly rare complication of
    spinal and epidural anesthesia.
    * Symptoms from an epidural abscess may not become
    apparent until several days after placement of the block.
    * The usual symptoms include severe back pain, sensory
    disturbances, and motor weakness.
    * Patients with epidural abscesses will complain of
    radicular pain approximately 3 days after development of
    the back pain.
    B. In an epidural hematoma severe back pain is the key
    feature.
    C. Anterior spinal artery syndrome is characterized
    predominantly by motor weakness or paralysis of the
    lower extremities.
    D. Arachnoiditis starts as a minimal cellular infl ammatory
    response.
    *It may follow trauma, surgery, tumors, infections,
    hemorrhage orsome intrathecal compound administration
    * Onset of symptoms varies from hours to months,
    resulting in delay in diagnosis
    * Symptoms include:
    - Radicular pain
    - Perineal sensory loss
    - Lower extremity paresis or paralysis
    * Diagnosis can be made by CT, MRI or myelography
    E. Meralgia paresthetica is related to entrapment of the
    lateral femoral cutaneous nerve as it courses below the
    inguinal ligament and is associated with burning pain over
    the lateral aspect of the thigh. It is not a complication of
    epidural anesthesia.
132
Q
  1. A 49-year-old man presents with painful, recurring
    episodes of swelling in his left great toe. He takes 25 mg
    of hydrochlorothiazide daily for blood pressure control
    but otherwise is in good health. On physical examination,
    the patient is afebrile but his great toe is warm, swollen,
    erythematous, and exquisitely tender to palpation. He has
    several subcutaneous nodules in his pinna. The following
    is the most likely diagnosis:
    A. Calcium pyrophosphate dihydrate deposition disease
    B. Calcium oxalate deposition disease
    C. Monosodium urate deposition disease
    D. Calcium phosphate deposition disease
    E. Osteoarthritis of the great toe
A
  1. Answer: C
    Explanation:
    (Tierney, 42/e, pp 786-790.)
    Tophaceous gout is characterized by the fi nding in synovial
    fl uid of monosodium urate crystals that are needle-shaped and strongly negative birefringent (bright yellow when
    parallel to the axis). Gouty attacks may be precipitated by
    trauma, medications that inhibit tubular secretion of uric
    acid (aspirin, hydrochlorothiazide), surgery, stress,
    alcohol, or a high-protein diet. The patient may have an
    accumulation of tophi in and around the joints and
    earlobe.
    Radiographs may show “rat bite” erosions. Pseudogout is
    due to calcium pyrophosphate dihydrate (CPPD)
    deposition disease; the crystals here are rhomboid-shaped
    and weakly positive birefringent (blue when parallel to the
    axis). Calcium oxalate deposition disease is usually seen in
    patients with end-stage renal disease; calcium phosphate
    deposition disease causes calcifi c tendinitis or Milwaukee
    shoulder.
133
Q
  1. Your patent is a 38-year-old male who plays in a weekend volleyball league on a regular basis. He has developed posterior shoulder pain that is aching in nature and increases with increased slamming of the ball over the net. You have noticed that his symptoms are provoked
    with passive internal rotation and adduction of his arm
    behind his back, followed by passive cervical sidebending
    to the contralateral side. Which disorder do you suspect?
    A. Acromioclavicular arthritis
    B. Bennett’s lesion
    C. Posterior glenohumeral labral tear
    D. Suprascapular nerve entrapment
    E. Adhesive Capsulitis
A
  1. Answer: D

Source: Sizer Et Al - Pain Practice March & June 2003

134
Q
  1. When testing a patients extraocular muscle movements,
    you detect that the right eye cannot adduct past the
    midline. However, when you move a fi ngertip toward
    the patient’s nose, convergence does occur. Which of the
    following is the most likely diagnosis?
    A. Paralysis of cranial nerve VI
    B. Paralysis of cranial nerve III
    C. Internuclear ophthalmoplegia
    D. Retrobulbar optic neuritis
    E. Paralysis of cranial nerve II
A
  1. Answer: C
    Explanation:
    (Goldman, 21/e, p 2240.)
    Internuclear ophthalmoplegia (INO) is caused by a lesion
    in the medial longitudinal fasciculus (MLF) and may be
    due to glioma in children, multiple sclerosis in young
    adults, or vascular infarction in the geriatric age group.
    INO commonly causes paresis of adduction of the
    ipsilateral eye (patients cannot look medially), horizontal
    nystagmus in the contralateral abducting eye, and vertical
    nystagmus with upward gaze, but convergence is intact.
135
Q
  1. According to psychoanalytic theory, which of the
    following statements about the development of the
    superego is true?
    A. It is present at birth
    B. It begins to develop during the fi rst two years of life
    C. It begins to develop during the fi fth or sixth year of life
    D. It begins to develop during puberty
    E. It begins to develop in late adolescence
A
  1. Answer: C
    Explanation:
    (Kaplan, pp 206-223.)
    Freud maintained that the
    superego begins to develop around the age of 5 or 6 as part
    of the resolution of the Oedipus complex.At the end of the
    phallic stage of psychosexual development (which lasts
    from around 211> to 6 years of age), children must
    abandon the sexual and aggressive impulses that were
    directed toward their parents to avoid the parents’ strong
    disapproval. In abandoning these impulses, children
    identify with their parents. Part of this identifi cation
    involves the internalization of parental standards of
    morality; this internalization marks the beginning of the
    superego.
    Source: Ebert 2004
136
Q
  1. At a follow-up visit one month after a 22-year-old male
    was newly diagnosed with schizophrenia and started
    on chlorpromazine, he has several complaints, listed
    below. Which of the following cannot be attributed to
    chlorpromazine?
    A. Restless feeling
    B. Sexual dysfunction
    C. Urinary hesitancy
    D. Vomiting
    E. None of the above
A
  1. Answer: D
    Explanation:
    Antipsychotic agents, particularly prochlorperazine, are
    also useful as antiemetic agents, thought to be due to
    dopamine blockade at the stomach and at the
    chemoreceptor trigger zone of the medulla.
    Source: Stern - 2004
137
Q
875. Which of the following negative emotional states or
conditions most commonly precedes relapse in the
treatment of addictive behaviors?
A. Stress
B. Depression
C. Anxiety
D. Anger
E. Frustration
A
  1. Answer: C
    Explanation:
    (Taylor, pp 108-114.)
    Negative emotional states of
    anxiety, depression, anger, frustration, and stress are
    related to relapse in the treatment of addictive behaviors
    involved in such disorders as alcoholism, smoking,
    obesity, and drug addiction. Most patients (about 70%)
    have negative affects preceding the relapse. The most
    common negative affect or mood state is anxiety related to
    the need for the addicting substance to relieve the anxiety.
    This is followed by anger, frustration, and depression.
    Furthermore, patients are at increased risk for relapse if
    they smoke, drink, eat, and so on in an attempt to reduce
    negative affect.
    Source: Ebert 2004
138
Q
  1. A 40-year-old man is asked to be evaluated by the company
    physician because he failed a mandatory random drug
    screen. The history indicates a pattern of substance abuse.
    The psycosocial factor most likely to be found in this
    person is
    A. Delusions of grandeur
    B. Depression mood
    C. Rationalization
    D. Denial
    E. Antisocial behavior
A
  1. Answer: D
    Explanation:
    (Sierles, pp 295-296. Ebert pp 233-259.)
    Even though there is no single personality type associated
    with substance abuse, denial is the major psychosocial
    factor in these persons. Denial can also complicate the
    treatment, even though the substance abuser may admit to
    the addiction. Substance abusers do frequently display
    delusions of grandeur, are often in a depressed mood,
    frequently display antisocial behavior, and often rationalize
    their behavior or situation, but denying the seriousness of
    their drinking or drug problem and its effect on their life
    or loved ones is the most common psychosocial factor
    seen. Denial is a form of self-deception that permeates
    their psychological and social behavior.
    A physician must be constantly aware of the infl uence of
    denial on the patient’s self-reported history and the effect
    of denial on prognosis.
    Source: Ebert 2004
139
Q
  1. If resisted shoulder external rotation is the MOST
    painful procedure during the shoulder basic functional
    examination, which tendon insertion is most likely the
    pain generator?
    A. Supraspinatus
    B. Infraspinatus
    C. Subscapularis
    D. Deltoid
    E. Biceps
A
  1. Answer: B

Source: Sizer Et Al - Pain Practice March & June 2003

140
Q
  1. What is the approximate scapulo-humeral movement
    ratio produced between the glenohumeral joint and
    scapulothoracic complex at 40° of arm elevation?
    A. A ratio of 1 : 1
    B. A ratio of 7 : 1
    C. A ratio of 3 : 1
    D. A ratio of 5 : 1
    E. A ratio of 2 : 1
A
  1. Answer: B

Source: Sizer Et Al - Pain Practice March & June 2003

141
Q
  1. A 40-year-old male with a diagnosis of moderate to severe
    asthma is placed on zileuton. What is the mechanism of
    action of zileuton?
    A. Inhibition of cytokine production
    B. Inhibition of leukotriene production
    C. Inhibition of mediator release
    D. Inhibition of muscarinic receptor action
    E. Inhibition of calcium (Ca2+) channel activity
A
  1. Answer: B
142
Q
  1. In a patient with spinal stenosis at L5/S1 levels with
    history of low back and lower extremity pain, the likely
    electrodiagnostic fi ndings are as follows:
    A. Reduced amplitude of H-have response
    B. Increased amplitude of the somatosensory evoked response
    C. Normal F-wave response
    D. Reduced conduction velocity of the genitofemoral
    nerve
    E. Fibrillation in tibialis anterior
A
  1. Answer: A
    Explanation:
    Slowing of nerve conduction velocity occurs through the
    region of neurologic impairment. Needle EMG may not
    show evidence of membrane instability. Motor unit action
    potentials may have increased amplitude and duration
    because of collateral innervation, which occurs over time.
    Somatosensory evoked potentials will be abnormal to
    varying degrees in the dermatomes of the affected nerve
    roots. Both F- and H-wave late responses will be
    abnormal. Fibrillation or sharp waves in tibialis anterior
    are seen with involvement of L4 nerve root.
143
Q
  1. You were performing a stellate ganglion block at C6. You withdrew the needle 0.2 cm after inserting the needle
    to a depth of 1.4 cm. You were unable to inject due to
    resistance and pain. The tip of the needle is most likely
    located within the
    A. Periosteum
    B. Longus colli
    C. Vertebral artery
    D. Intervertebral disc
    E. Subarachnoid space
A
  1. Answer: B
144
Q
882. Epidural use of which of the following opioids would
result in the greatest incidence of delayed respiratory
depression?
A. Sufentanyl
B. Fentanyl
C. Morphine sulfate
D. Hydromorphone
E. Meperidine
A
  1. Answer: C
    Explanation:
    Water-soluble drugs such as morphine have a higher
    potential for inducing delayed respiratory depression
    through cephalad migration in the CNS.
145
Q
  1. A 36-year-old male has been experiencing intense pressure to be more productive at work. This has resulted
    in his becoming extremely anxious, which makes it very
    diffi cult for him to function effectively. He wishes to keep
    his job. Physical examination and blood chemistries are
    normal. He is given diazepam, which diminishes his
    anxiety and allows him to concentrate on his work. What
    is the mechanism of action of diazepam?
    A. It directly opens the Cl¯ channel of the GABA receptor
    B. It increases the frequency of the Cl¯ channel of the GABA
    receptor
    C. It prolongs the duration of opening of the Cl¯ channel of
    the GABA receptor
    D. It simulates k receptors
    E. It simulates m receptors
A
  1. Answer: B
    Explanation:
    Reference: Hardman, pp 365-367.
    Benzodiazepines, such as diazepam, bind to the GABA
    receptor/ion channel complex, enhancing GABA-induced
    Cl¯ currents related to more frequent bursts of Cl¯
    channel opening by GABA.
    kappa and μ receptors are opioid receptors.
    Source: Stern - 2004
146
Q
  1. An elderly patient presents with a complaint of pain in
    the distribution of the trigeminal nerve. The patient has
    no other medical problems except a history of congestive
    heart failure for which he takes digoxin and thiazide.
    In addition to his chief complaint, the patient over the
    last 72 hours has complained of dysesthesia in the feet,
    diffi culty with vision, and emesis on 3 or 4 occasions. The
    most appropriate step at this time would be
    A. Trigeminal nerve block with bupivacaine
    B. Obtain neurologic workup for multiple sclerosis
    C. Administration of fentanyl and ondansetron
    D. Initiate therapy with carbamazepine
    E. Obtain a digoxin level
A
  1. Answer: E
    Explanation:
    The early signs of digitalis toxicity include loss of appetite
    and nausea and vomiting. In some patients there may be
    pain that is similar to trigeminal neuralgia. Pain or
    discomfort in the feet and pain and discomfort in the
    extremities may be a feature of digitalis toxicity. Transient
    visual disturbances have been reported in patients with
    digitalis toxicity.
    Source: Hall and Chantigan.
    Source: Hall and Chantigan
147
Q
  1. An elderly woman presents with persistent and prolonged
    thoracic pain after a herpes zoster infection. Which of the
    treatments below would be the LEAST effi cacious in the
    treatment of her pain?
    A. Topical capsaicin ointment
    B. Oral clonidine
    C. Topical lidocaine patch
    D. Oral amitriptyline
    E. Transcutaneous electrical nerve stimulation
A
  1. Answer: B
    Explanation:
    Postherpetic neuralgia is defi ned as pain persisting
    beyond the healing of the herpes zoster lesions.
    The incidence of postherpetic neuralgia increases with
    age and occurs in 20% to 50% of patients older than 50
    years and greater than 50% in patients older than 80 years.
    A, C, D, E. Treatment of established postherpetic neuralgia
    has been shown to be resistant to interventions and thus
    can be diffi cult. Proven therapies include tricyclic
    antidepressants, antoconvulsants, topical local anesthetics,
    topical capsaicin, and sympathetic blocks.
    B. Oral clonidine, which is used to treat hypertension and
    opioid withdrawl, has not been shown to be an effective
    treatment for postherpetic neuralgia.
148
Q
  1. Complex regional pain syndrome type II (causalgia) is
    differentiated from complex regional pain syndrome type
    I (refl ex sympathetic dystrophy) by knowledge of its
    A. Etiology
    B. Rapidity of onset
    C. Type of symptoms
    D. Affected body region
    E. Chronicity
A
  1. Answer: A
    Explanation:
    Complex regional pain syndrome type I (refl ex
    sympathetic dystrophy) is a clinical syndrome of
    continuous burning pain usually occurring after an
    injury or surgery. Patients present with variable
    sensory, motor, autonomic, and trophic changes.
    Complex regional pain syndrome type II (causalgia)
    exhibits the same features of refl ex sympathetic dystrophy,
    but the etiology is damage to a major nerve.
149
Q
  1. A patient presents with acute onset of pain which started when he was stepping off a curb located over hip and buttock area which is referred to groin and lower
    extremity. Physical examination showed no leg length
    discrepancy but pain over superior iliac spine. The most
    likely diagnosis is:
    A. Lumbar facet joint pain
    B. Osteoarthritis of hip
    C. Lumbar radiculopathy
    D. SI joint pain
    E. Trochanteric bursitis
A
  1. Answer: D
150
Q
  1. In traditional psychoanalysts, transference is the process wherein:
    A. Psychic energy, or libido, is transferred from the id to the
    ego and superego
    B. A patient invests the analyst with attitudes and feelings
    derived from vital earlier associations
    C. Certain psychological symptoms seemingly defer to
    new symptoms that frequently are more accessible to
    analysis
    D. Early object choices are gradually decathected
    E. Latent dream content is transformed into manifest content
A
  1. Answer: B
    Explanation:
    (Kaplan, pp 885-888.)
    In traditional psychoanalytic treatment, analysts purposely
    reveal very little about themselves to their patients. That is
    intended to help promote transference-to create an
    ambiance that facilitates a patient’s ability to transfer his or
    her past emotional attachments to the psychoanalyst. The
    analyst becomes a substitute for the parental fi gure. In
    positive transference, the patient becomes attached to the
    analyst to obtain love and emotional satisfaction, where as
    in negative transference the analyst is seen as an unfair,
    unloving, and rejecting parental fi gure. Interpretations of
    transference may help the patient see the positive or
    negative feelings as a refl ection of previous of emotional
    entanglements
    Source: Ebert 2004
151
Q
  1. The therapeutic action of b-adrenergic receptor blockers
    such as propranolol in angina pectoris is believed to be
    primarily the result of
    A. Reduced production of catecholamines
    B. Dilation of the coronary vasculature
    C. Decreased requirement for myocardial oxygen
    D. Increased peripheral resistance
    E. Increased sensitivity to catecholamines
A
  1. Answer: C
    Explanation:
    Beta-adrenergic receptor blockers cause a slowing of heart
    rate, lower blood pressure, and lessened cardiac
    contractility without reducing cardiac output. There is also
    a buffering action against adrenergic stimulation of the
    cardiac autoregulatory mechanism. These hemodynamic
    actions decrease the requirement of the heart for oxygen.
    Source: Hardman, pp 855-856
152
Q
  1. True statements with worker’s compensation coverage are as follows:
    A. State-mandated worker’s compensation programs also
    cover all types of federal employees.
    B. Diffi cult cases are automatically settled after 12 months.
    C. Self-insured employers that do not subscribe to state
    laws are foolproof from litigation
    D. Self-insured employers that subscribe to state laws and
    administer their own benefi ts are very rigid and do not
    accommodate injured workers at light duty positions.
    E. Inherent problems with worker’s compensation system
    include poor understanding of the cause of pain, particularly
    in the absence of defi nitive diagnostic tests
    resulting in unsuccessful return to work and ineffective
    case management, etc.
A
  1. Answer: E
    Source: Cole and Hearring to the evaluation of Permanent
    Impairment, 2001.
153
Q
  1. A 39-year-old man presents with progressive weakness of his arms and legs. He noticed diffi culty in performing tasks such as buttoning up his shirt several months ago, and his symptoms have continued to worsen. On physical examination, cranial nerve and sensory findings are normal. Severe atrophy and fasciculations are seen in the legs, arms, and tongue. The patient has a spastic
    muscle tone, hyperactive refl exes, and bilateral extensor
    plantar refl exes. Which of the following is the most likely
    diagnosis?
    A. Werdnig-Hoffmann disease
    B. Multiple sclerosis
    C. Pott’s disease
    D. Amyotrophic lateral sclerosis
    E. Todd’s paralysis
A
  1. Answer: D
    Explanation:
    (Tierney, 42/e, pp 990-991.)
    Amyotrophic lateral sclerosis (ALS) is a degenerative
    disease that is the result of lower (anterior horn cells) and
    upper (corticospinal tracts) motor neuron loss. Patients
    present with asymmetric muscle weakness, atrophy,
    fasciculations, spasticity, hyperactive refl exes, and extensor
    plantar refl exes. Patients may complain of dysphagia and
    diffi culty holding the head up. Pott’s disease tuberculosis
    of the thoracic vertebral bodies. Todd’s paralysis is a
    transient paralysis following a seizure. Werdnig-Hoffmann disease is fl oppy baby disease; infants present
    with fasciculations. Poliomyelitis is a 1m motor neuron
    disease.
154
Q
892. The following contrast in recommended in interventional
techniques
A. Hypaque
B. Renographin
C. Non-ionized water soluble contrast
D. Ionized water soluble contrast
E. Ionized non water soluble contrast
A
  1. Answer: C

Source: Racz G. Board Review 2003

155
Q
  1. A 34-year-old man has been diagnosed with chronic
    paranoid schizophrenia for 10 years. He is currently
    in a psychiatric hospital and is not on psychotropic
    medications. More than 50% of individuals with this
    diagnosis and off medications would have abnormalities
    in which of these tests?
    A. Lactate infusion test
    B. Dexamethasone suppression test
    C. Eye pursuit test
    D. Thyrotropic releasing hormone (TRH) stimulation test
    E. Prolactin stimulation test
A
  1. Answer: C
    Explanation:
    (Sierles, pp 185-187. Ebert, pp 268-270.)
    · Many psychiatric disorders manifest evidence of brain
    dysfunction.
    · Evidence of brain dysfunction has been found in 50% or
    more of patients with schizophrenia. For example, a
    neurologic examination will demonstrate soft signs (e.g.,
    grasp refl ex, rooting refl ex, motor impersistence) in 70%
    of adult schizophrenics; 75% (whether ill or recovered)
    will also demonstrate abnormalities in eye pursuit; 75%
    will have moderate-to-severe bilateral impairment on
    neuropsychological tests; and 50% will have nonspecifi c
    abnormalities on the EEG. Also, 50% will have some
    cortical atrophy and ventricular enlargement on imaging
    tests.
    · The lactate infusion test induces panic behavior in 80%
    of patients with panic disorder, but not in patients with
    schizophrenia.
    Source: Ebert 2004
156
Q
  1. A 23-year-old woman complains of periodic, throbbing,
    right-sided headaches accompanied by nausea and
    vomiting. On physical examination during the time of
    headache, the patient demonstrates a right oculomotor
    nerve palsy. MRI is normal. Choose correct type of
    headache:
    A. Complicated migraine
    B. Basilar migraine
    C. Classic migraine
    D. Common migraine
    E. Temporal arteritis
A
  1. Answer: A
    Explanation:
    (Tierney, 42/e, pp 947-949)
    A. Complicated migraines may be preceded by aura and
    are headaches accompanied by sensory or motor defi cits
    or muscle palsies.
    The patient described is having a specifi c kind of
    complicated migraine called an ophthalmoplegic migraine.
    A mnemonic for migraine is POUND (Pulsatile, lasts
    One day, Unilateral, Nausea, and interferes with Daily
    activities).
    B. Basilar artery migraine is a variant of classic migraine
    in which the aura consists of drop attacks, confusion,
    blindness, and vertigo (all signs of basilar artery
    ischemia).
    C. Classic migraine is a unilateral headache that is
    pulsatile and throbbing in nature and is preceded by a
    prodromal aura consisting of scotomas (black spots),
    scintillations (light fl ashes), or hemianopsia.
    D. Common migraines lack a prodromal aura.
    E. Patients with temporal arteritis are older (>50 years
    old) and have headaches along with jaw claudication and
    tenderness over the temporal artery.
157
Q
  1. A patient has had an implanted intrathecal infusion
    pump for post-laminectomy syndrome for the past 3
    years. He has had relatively good pain control with a
    combination infusion of morphine, bupivacaine, and
    baclofen. You are asked to evaluate him in the emergency
    room for increasing low back pain associated with new
    onset of right leg pain and right leg weakness. Physical
    examination reveals positive right straight leg raising
    with loss of right Achilles refl ex. Plain x-ray has identifi ed
    the titanium catheter tip marker at the T8 level in the
    ventral intrathecal space. The following statement is
    false:
    A. Most patients who are diagnosed with catheter tip granuloma
    present with gradual loss of pain control associated
    with gradual onset of lower extremity neurological
    defi cits evolving over weeks and months.
    B. When catheter tip granuloma is diagnosed, surgical
    removal of the catheter and pump is the treatment of
    choice.
    C. MRI with and without gadolinium enhancement is the
    imaging study of choice to assess catheter tip granuloma.
    D. Baclofen when used alone in the pump for spasticity
    management has not been implicated in catheter tip
    granuloma.
    E. Right lumbar radiculopathy is a much likelier diagnosis
    than catheter tip granuloma in this patient
A
  1. Answer: B
    Explanation:
    Reference:
    Management of Intrathecal Catheter-Tip Infl ammatory
    Masses: A Consensus Statement
    Hassenbusch et. Al. Pain Medicine 2002
    Infl ammatory mass formation at the tip of an implanted
    intrathecal catheter is a rare but potentially devastating
    complication of intrathecal drug infusion. Hassenbush et.
    Al. reviewed published and unpublished case reports and
    their own experiences to recommend methods to diagnose
    and treat catheter-tip infl ammatory masses in the above
    article.
    After comprehensive review, the Hassenbush consensus
    panel concluded that:
    Fluctuations in patients’ subjective symptoms and
    underlying pain levels are common after the implantation
    of drug delivery systems, but the occurrence of new or
    extraordinary complaints that require unexpected
    analgesic dose changes should alert physicians to consider
    a catheter-tip mass among other possibilities in the
    differential diagnosis. Gradual, insidious neurological
    deterioration weeks or months after the appearance of
    subjective symptoms was the most common clinical
    course for catheter tip granulomas before the onset of
    myelopathy or cauda equina syndrome in cases reported to
    date.
    Physicians should have a low threshold for performing
    an imaging study to confi rm or rule out the presence of a
    catheter-tip mass in patients with suspicious symptoms or
    physical fi ndings. Unless medically contraindicated, MRI
    with and without intravenous gadolinium contrast
    enhancement is the imaging procedure of choice. CT
    myelogram is an acceptable alternative and is equally
    sensitive and reliable. Catheter-tip masses are visualized
    best on intravenous contrast-enhanced T2-weighted
    images. The mass appears as an enhancing lesion having
    the tip of the drug administration catheter embedded
    within it.
    Not all patients with catheter tip granuloma require
    catheter and pump removal. When catheter tip granuloma
    is diagnosed, optimal management should take into
    account the patient’s clinical condition, the wishes of the
    patient and the available options for chronic pain
    management. Mildly symptomatic patients with small
    masses that are diagnosed during investigation of
    diminished analgesic effi cacy or other subjective
    complaints have been managed safely and successfully
    without open surgical decompression or removal of the
    mass. These masses did not signifi cantly compress neural
    structures, nor compromise neurological function, and
    were treated with prompt discontinuation of intrathecal
    drug administration. Shrinkage or disappearance of the
    mass was documented on follow-up imaging studies after
    an interval of 2-5 months. Consequently, catheter-tip
    infl ammatory masses that are detected early in the clinical
    course can be treated safely and effectively by maneuvers
    directed at modifying rather than removing the drug
    infusion system. If the decision is made to leave the
    infusion system in place, the responsible physician
    eventually must decide whether to continue intrathecal
    therapy and whether to change the dose, concentration, or
    the drug(s) being infused. Alternatives to complete
    removal of the catheter and pump include ceasing or
    changing drug infusion and:
    1.Repositioning of the catheter at a different spinal level.
    2.Placing a new catheter to replace the existing catheter.
    3.Allowing the catheter and pump to remain dormant for
    a period of time.
    In contrast, patients presenting with paraplegia or
    progressive myelopathy or with apparently fi xed
    neurological defi cits of short duration may require
    emergent operative intervention because of concern that
    delayed treatment could foreclose the possibility of
    neurological recovery. Surgical intervention to remove the
    mass and/or de-compress the spinal canal has restored
    neurological function or prevented further neurological
    deterioration in several reported cases. The extent of
    resection was limited in some cases owing to adhesions to
    the spinal cord or nerve roots or because of the ventral
    location of a mass beneath the thoracic spinal cord.
    Because the masses were not neoplastic, in several cases the
    postoperative residual mass gradually shrank or
    disappeared over time.
    Source: Schultz D, Board Review 2004
158
Q
  1. A 70-year old patient presents with a history of increasing
    pain in the back, buttocks and leg. Pain in the leg worsens
    with standing and walking. Pain is relieved on bending
    forward. No neurological defi cits were identifi ed on
    physical examination. Acetaminophen gives minimal
    relief. Your next treatment would be administration of:
    A. Opioids
    B. Epidural steroid injections
    C. Non-steroidal anti-infl ammatory drugs
    D. Facet joint injections
    E. Transcutaneous electrical stimulation
A
  1. Answer: C
    Explanation:
    The pain noted by this patient may include
    musculoskeletal pain associated with spondylosis, as well
    as radicular pain or neurogenic claudication.The fi rst
    approach should be noninvasive, and NSAIDs are
    appropriate, because acetaminophen was minimally
159
Q
  1. A 60-year-old man was involved in a motor vehicle
    accident and suffered multiple long bone fractures and a
    severe injury to the pelvis. Two days following admission
    to the hospital, he develops fever, tachypnea, and
    tachycardia. The rest of his physical examination reveals
    chest, neck, and conjunctival petechiae. Respiratory exam
    reveals scattered crackles bilaterally but no wheezes.
    Pulse oximetry reveals a hemoglobin saturation of 80%
    on room air. Which of the following is the most likely
    diagnosis?
    A. Pneumothorax
    B. Pneumonia
    C. Exacerbation of chronic obstructive pulmonary disease
    (COPD)
    D. Anemia from traumatic blood loss
    E. Fat embolism syndrome
A
  1. Answer: E
    Explanation:
    (Goldman, 21/e, p 448.)
    The signs and symptoms of fat embolism syndrome are
    those of adult respiratory distress syndrome (ARDS) in
    association with musculoskeletal trauma. It usually occurs
    2 to 4 days after the injury. The predominant feature is
    respiratory failure. Petechiae are found in 50 to 60% of
    patients, generally on the anterior chest and neck, axillae,
    and conjunctiva.Although fractures of the pelvis may
    cause life-threatening blood loss and subsequent
    hypovolemic shock, the patient will probably have other symptoms, such as oliguria, hypotension, pale conjunctiva,
    clouded sensorium, and cool extremities.
160
Q
  1. Following cholecystectomy, a patient is receiving
    bupivacaine by intrapleural infusion at 8mL/hr. The
    patient is noted to have a Horner’s syndrome and
    inadequate pain relief. The next step in managing this
    patient is to :
    A. Increase the rate of bupivacaine infusion
    B. Remove and reinsert the catheter
    C. Obtain neurology consultation
    D. Obtain an MRI of the head
    E. Perform a chest radiograph
A
  1. Answer: B
    Explanation:
    Assuming that you wish to continue the postoperative
    intrapleural infusion, the catheter should be removed and
    replaced. A Horner’s syndrome is a recognized side effect
    of interpleural infusions. Poor pain relief means that the
    catheter not covering the area of the incision. The
    Horner’s syndrome means that the infusion is not
    intravascular
161
Q
  1. Patients with a strong sense of an external locus of
    control of health will be more apt to respond to inpatient
    treatment in the following way:
    A. They delegate control of their health to their doctor or
    signifi cant other
    B. They can be relied on to follow treatment orders when
    they are discharged for outpatient follow-up
    C. They respond poorly and less comfortably to inpatient
    care
    D. They prefer to make as many decisions about their care
    as possible
    E. They prefer to maximize their own decision making
    about their own health care
A
  1. Answer: A
    Explanation:
    (Sierles, pp 103-104. Wedding, pp 378-390.)
    A. Persons with a strong sense of an external locus of
    health control delegate responsibility for their health to an
    external force, such as fate, powerful others, chance, or
    God.
    B. As outpatients they cannot be relied on to take
    responsibility for their own care.
    C. They can be relied on to follow treatment orders in the
    hospital, where they are in an authoritarian system.
    D. They prefer to make as few decisions as possible about
    their own health and prefer to accept the authority and
    orders of their own physician, except if the authority is not
    present to follow up on them.
    E. Patients with a strong sense of internal control tend to
    accept responsibility for and control their own health.
    This information can be of help to physicians by allowing
    them to establish follow-up procedures that will ensure
    maximum compliance.
    Source: Ebert 2004
162
Q
  1. Patients with low back pain have been found to have:
    A. Normal levels of aerobic fi tness compared to normal
    controls.
    B. An inability to improve their aerobic capacity.
    C. Protection against low back pain at work after a period
    of aerobic training.
    D. Spine problems that would prohibit most forms of aerobic
    exercise.
    E. Lumbar disc herniation in 90% of cases
A
  1. Answer: C

Source: Malanga G, Board Review 2003

163
Q
901. Patient notes the “worst headache ever” after exercise. Physical examination shows neck pain with movement. The next course of action is:
A. MRI of head
B. MRI of neck
C. CT of head
D. CT of neck
E. Cervical spine films
A
  1. Answer: C
    Explanation:
    the patient needs a head CT to evaluate for possible
    subarachnoid hemorrhage.The subsequent step would be
    a spinal tap, assuming no mass lesion or shift.
164
Q
  1. A patient presents with acute low back and lower
    extremity pain. Motor examination showed weakness
    with foot inversion. There was sensory defi cit on the
    medial aspect of the leg. The most likely diagnosis is:
    A. L3/4 disc herniation with L4 nerve root involvement
    B. L5 nerve root involvement with L4/5 disc herniation
    C. S1 nerve root involvement with L5/S1 disc herniation
    D. L3 nerve root involvement with L2/3 disc herniation
    E. L4/5 disc herniation with S1 nerve root involvement
A
  1. Answer: A
    Explanation:
    L4 nerve root involvement with L3/4 disc herniation
    shows weakness of tibialis anterior demonstrated by
    weakness of foot inversion. Refl exes are patellar and
    sensation is on the medial leg.
    Source: Hoppenfeld S. Orthopaedic Neurology. A
    Diagnostic Guide to Neurologic Levels. Philadelphia,
    LWW, 1997.
165
Q
903. Which of the following is not an indication for
stretching?
A. Prolonged immobilization
B. Restricted mobilitiy
C. Connective tissue diseases
D. Structural damage due to trauma
E. Recent fracture
A
  1. Answer: E
    Explanation:
    (Raj, Practical Mgmt of Pain, 3rd Ed., page 535-536)
    Indications
    Prolonged immobilization leading to adhesions and
    contractures.
    Restricted mobility.
    Connective tissue or neuromuscular diseases.
    Structural damage secondary to trauma.
    Congenital or acquired bony deformities.
    Contraindications
    Restricted motion secondary to a bony block.
    After a recent fracture.
    Evidence of an acute infl ammatory or infective process,
    either in or around a joint.
    Patients in whom contractures are the chief means of
    providing joint stability
    Source: Shah RV, Board Review 2005
166
Q
  1. A 46-year-old woman has a l-month history of headache.
    She has no past medical history of headache and no
    family history of headache. She does not use illicit drugs,
    drink alcohol, or smoke cigarettes. Physical examination
    reveals alexia, agraphia, acalculia, right-left confusion,
    and linger agnosia. An MRI of the brain with gadolinium
    is most likely to show which of the following?
    A. Frontal lobe lesion
    B. Parietal lobe lesion
    C. Temporal lobe lesion
    D. Occipital lobe lesion
    E. Cerebellar lesion
A
  1. Answer: B
    Explanation:
    (Tierney, 42/e, p 969-971.)
    MRI will most likely reveal a lesion of the parietal lobe.
    Parietal lobe lesions may produce contralateral
    hyperpathia and pain (thalamic syndrome) and
    Gerstmann syndrome (alexia, agraphia, acalculia, right-left
    confusion, and fi nger agnosia).
    Occipital lobe lesions produce partial fi eld defects.
    Temporal lobe lesions produce seizures, lip smacking,
    olfactory or gustatory hallucinations, and behavioral
    changes. Frontal lobe lesions lead to intellectual decline
    and personality changes. The most common adult primary
    tumors are gliomas.
167
Q
  1. A college student presents with complain of pain in
    fi ngers with blanching and cyanosis of her fi ngertips in
    cold weather and numbness. She has a 6-month history
    of dysphagia and arthralgias. She does not smoke or
    take any medications. On physical examination, the
    skin of her hands appears to be taut and atrophic with a
    fl exion deformity from the tight skin (sclerodactyly). The
    following is the most likely diagnosis:
    A. Rheumatoid arthritis
    B. Progressive systemic sclerosis
    C. Dermatomyositis
    D. Ulcerative colitis
    E. Sarcoidosis
A
  1. Answer: B
    Explanation:
    (Tierney, 42/e, pp 813-814.)
    The patient presents with symptoms suggestive of
    scleoderma or progressive systemic sclerosis (PSS). This
    disease, when diffuse, involves the skin, joints, lungs,
    heart, and gastrointestinal system. Limited systemic
    sclerosis (lSSc) was formerly known as the CREST
    syndrome (Calcinosis cutis, Raynaud’s phenomenon,
    Esophageal dysfunction, Sclerodactyly, and
    Telangiectasia).
    Raynaud’s phenomenon may be associated with tobacco
    use, medication use (ß-adrenergic blockers), or diseases
    such as systemic lupus erythematosus, rheumatoid
    arthritis, carpal tunnel syndrome, or thromboangiitis
    obliterans.
    Dermatomyositis is a systemic disease characterized by a
    violaceous rash of the eyelids and periorbital areas
    (heliotrope) and fl at, violaceous papules over the knuckles
    (Gottron sign). The rash seen in ulcerative colitis is
    pyoderma gangrenosum. These painful ulcers are large and
    irregular and drain purulent, hemorrhagic exudates.
    Sarcoidosis is a systemic disease with skin
    manifestations, bilateral hilar adenopathy, and pulmonary
    disease. Patients with sarcoidosis may present with
    erythema nodosum, which typically takes the form of
    multiple fi rm, red, painful plaques that are bilateral and
    most frequently distributed on the legs. Musculoskeletal
    fi ndings in sarcoidosis include arthritis and tenosynovitis
168
Q
906. What is transmitted in the tarsal tunnel?
A. Anterior tibial tendon
B. Posterior tibial nerve
C. Flexor hallucis longus tendon
D. Posterior tibial tendon
E. Flexor digitorum longus tendon
A
  1. Answer: A
    Explanation:
    The tarsal tunnel is bounded by a fl exor retinaculum that
    spans the medial malleolus and the calcaneus
    The tibialis posterior, fl exor digitorum longus, and fl exor
    hallucis longus tendons and the posterior tibial artery and
    nerve pass through the tarsal tunnel
    Source: Shah RV, Board Review 2004
169
Q
  1. A 30-year-old obese woman presents with a 2-month
    history of a nonthrobbing headache that is constant and
    dull in nature. The headache is worsened with bending
    over or sneezing and on awakening in the morning. The
    patient also complains of blurred vision and occasional
    diplopia. Funduscopic examination reveals blurring of
    the optic discs bilaterally and no other neurologic defi cit.
    Which of the following is the most likely diagnosis?
    A. Infratentorial brain tumor
    B. Pseudo tumor cerebri
    C. Supratentorial brain tumor
    D. Pituitary adenoma
    E. Metastatic brain tumor
A
  1. Answer: B
    Explanation:
    (Tierney, 421e, p 974.)
    Patients with pseudotumor cerebri (benign intracranial
    hypertension) present with headache and papilledema.
    They are often obese women in their childbearing years.
    Other possible causes include hypervitaminosis A and the
    use of oral contraceptives or antibiotics (tetracycline).
    Lumbar puncture will reveal an elevated opening pressure.
    Treatment includes weight reduction and repeated lumbar
    punctures to reduce intracranial pressure. A complication
    of pseudo tumor cerebri is blindness; patients with visual
    changes may require emergency optic nerve sheath
    decompression.Pituitary adenomas are benign tumors that
    may cause a bitemporal hemianopsia and endocrine
    disturbances, such as hyperprolactinemia (galactorrhea),
    acromegaly or gigantism, and Cushing’s disease. A
    ruptured berry aneurysm causes a subarachnoid
    hemorrhage (SAH). Patients present with the acute onset
    of severe headache, photophobia,and neck stiffness.Adults
    commonly have supratentorial primary brain tumors
    (astrocytoma including glioblastoma multiforme is the
    most common), while children have infra tentorial
    primary brain tumors (medulloblastoma is the most
    common). Overall, metastatic brain tumors are more
    common than primary brain tumors. The most common
    metastatic brain tumors come from the Lung, Breast, Skin,
    Kidney, or GI tract (mnemonic: Lots of Bad Stuff KillsGlia). The headache of tumor is often continuous;
    exacerbated by coughing, sneezing, movement, or the
    Valsalva maneuver; and worse in the morning.
170
Q
  1. A 24-year-old woman has a 2-year history of recurrent
    right-sided headaches that are throbbing in nature and
    are preceded by 30 min of scintillating scotomas and
    fortifi cations. Choose correct type of headache:
    A. Complicated migraine
    B. Basilar migraine
    C. Classic migraine
    D. Common migraine
    E. Temporal arteritis
A
  1. Answer: C
    Explanation:
    (Tierney, 42/e, pp 947-949)
    A.Complicated migraines may be preceded by aura and are
    headaches accompanied by sensory or motor defi cits or
    muscle palsies.
    The patient described is having a specifi c kind of
    complicated migraine called an ophthalmoplegic migraine.
    A mnemonic for migraine is POUND (Pulsatile, lasts
    One day, Unilateral, Nausea, and interferes with Daily
    activities).
    B. Basilar artery migraine is a variant of classic migraine
    in which the aura consists of drop attacks, confusion,
    blindness, and vertigo (all signs of basilar artery
    ischemia).
    C. Classic migraine is a unilateral headache that is
    pulsatile and throbbing in nature and is preceded by a
    prodromal aura consisting of scotomas (black spots),
    scintillations (light fl ashes), or hemianopsia.
    D. Common migraines lack a prodromal aura.
    E. Patients with temporal arteritis are older (>50 years
    old) and have headaches along with jaw claudication and
    tenderness over the temporal artery.
171
Q
909. A long-distance runner develops foot pain with exercise.
CHOOSE CORRECT DIAGNOSIS:
A. Hammer toe
B. March fracture
C. Genu valgum
D. Genu varum
E. Bunion
A
  1. Answer: B
    Explanation:
    (Seidel, 5/e, p 732.)
    Improper footwear results in lateral deviations of the great
    toe, extensor, and fl exor hallucis longus tendons (bunion
    formation). Hammer toe often affects the second toe. The
    metatarsophalangeal joint is dorsifl exed and the proximal
    interphalangeal joint displays plantar fl exion. A stress
    fracture of a metatarsal is called a march fracture. Stress
    fractures result in bone resorption followed by insuffi cient
    remodeling due to continued activity Stress fractures
    occur in the tibia as well as the metatarsal; examination
    typically reveals point tenderness and swelling. In genu
    varum (bowleg), the lateral femoral condyles are widely
    separated when the feet are placed together in the
    extended position. In genu recurvatum, the knee
    hyperextends, and in genu impressum, there is fl attening
    and bending of the knee to one side with displacement of
    the patella. Pes planus is a fl attened longitudinal arch of
    the foot, often called fl at foot. Morton’s neuroma causes
    pain in the forefoot that radiates to one or two toes with
    tenderness between the two metatarsals. The pain may be
    further aggravated by squeezing the metatarsals together.
172
Q
  1. The mechanism of cryotherapy’s affect on pain occurs
    by:
    A. Increasing metabolic rate of tissues.
    B. Vasodilatation of blood vessels.
    C. As a counter-irritant.
    D. Improving contractility of muscle and ligament fi bers.
    E. Increasing nerve conduction along pain pathways.
A
  1. Answer: C

Source: Malanga G, Board Review 2003

173
Q
911. An aphasia is most likely to be associated with a lesion
of
A. The hippocampus
B. The temporal lobe
C. The parietal lobe
D. The limbic system
E. The reticular activating system
A
  1. Answer: B
    Explanation:
    (Guyton, pp 669-671.)
    Aphasia is a language disorder in which a person is unable
    to properly express or understand certain aspects of
    written or spoken language. It is caused by lesions to the
    language centers of the brain, which, for the majority of
    persons, are located within the left hemisphere in the
    portions of the temporal and frontal lobes known as
    Wernicke’s and Broca’s areas, respectively. Language
    disorders caused by memory loss, which could be the
    result of a hippocampal lesion, are not classifi ed as
    aphasias.
174
Q
  1. McKenzie exercises:
    A. Would include repetitive extension even if radicular
    symptoms are increased.
    B. Stresses “centralization” of back pain symptoms.
    C. Is most helpful in chronic nonorganic low back pain.
    D. Is contra-indicated in acute disc herniations.
    E. Would stress on fl exion exercises
A
  1. Answer: B

Source: Malanga G, Board Review 2003

175
Q
  1. Parrafi n Wax is most helpful in which of the following
    conditions:
    A. Following an acute burn injury.
    B. Joint pain of the hands from osteoarthritis.
    C. Carpal tunnel syndrome.
    D. An acute hand fracture.
    E. Ankle Strain
A
  1. Answer: B

Source: Malanga G, Board Review 2003

176
Q
  1. The most important role of the gamma motoneurons is
    to
    A. Stimulate skeletal muscle fi bers to contract
    B. Maintain afferent activity during contraction of muscle
    C. Generate activity in Ib afferent fi bers
    D. Detect the length of resting skeletal muscle
    E. Prevent muscles from producing too much force
A
  1. Answer: B
    Explanation:
    (Berne, 3/e, pp 117-118.)
    The gamma motoneurons innervate the intrafusal fi bers of
    the muscle spindles. When a skeletal muscle contracts, the
    intrafusal muscle fi ber becomes slack and the Ia afferents
    stop fi ring. By stimulating the intrafusal muscle fi bers
    during a contraction,the gamma motoneurons prevent the
    intrafusal muscle fi bers from becoming slack and thus
    maintain fi ring during the contraction
177
Q
915. A 55-year-old woman walks by lifting one foot further off the ground than the other. Choose correct description of
gait:
A. Ataxic gait
B. Parkinsonian gait
C. Spastic hemiplegic gait
D. Steppage gait
E. Scissor gait
A
  1. Answer: D
    Explanation:
    (Berne, 3/e, pp 117-118.)
    The gamma motoneurons innervate the intrafusal fi bers of
    the muscle spindles. When a skeletal muscle contracts, the
    intrafusal muscle fi ber becomes slack and the Ia afferents
    stop fi ring. By stimulating the intrafusal muscle fi bers
    during a contraction,the gamma motoneurons prevent the
    intrafusal muscle fi bers from becoming slack and thus
    maintain fi ring during the contraction
178
Q
  1. A Middle aged woman presents with a one year history
    of pain and morning stiffness accompanied by swelling
    of her wrists and the proximal interphalangeal joints of
    both hands. She also has knee pain and swelling of knee
    joints. Physical examination reveals synovial tenderness
    and swelling of her knees, wrists, and proximal
    interphalangeal joints. She has subcutaneous nodules
    in the extensor area of her right forearm.The right knee
    has a positive bulge sign consistent with an effusion. The
    most likely diagnosis is:
    A. Osteoarthritis
    B. Rheumatoid arthritis
    C. Septic arthritis
    D. Chondrocalcinosis
    E. Scleroderma
A
  1. Answer: B
    Explanation:
    (Tierney, 42/e, pp 829-831.)
    C. A septic joint will usually produce systemic symptoms
    such as fever.
    A. Osteoarthritis produces a short period of morning
    stiffness and often affects the distal interphalangeal joints.
    D. Chondrocalcinosis is a radiologic fi nding (destructive
    arthropathy) associated with pseudogout or CPPD
    crystals.
    B. The patient most likely has rheumatoid arthritis since
    she meets four of the seven criteria as classifi ed by the
    American College of Rheumatology:
    Symmetric polyarthritis for over 3 months
    Morning stiffness lasting more than 1 h
    Rheumatoid nodules
    Arthritis of more than three joint areas
    Involvement of the joints of the hands and wrists;
    patients may have swan-neck deformity
    (hyperextension of the proximal interphalangeal joints
    with compensatory fl exion of the distal joint),
    boutonniere deformity (extension of the distal
    interphalangeal joint), or ulnar deviation of the digits
    A positive rheumatoid factor (RF)
    Erosions or decalcifi cation on radiographs
179
Q
917. All of the following psychiatric disorders are diagnosed
more often in women than in men. The most frequently
diagnosed disorder in women is
A. Depression
B. Obsessive-compulsive disorders
C. Anxiety disorders
D. Bulimia
E. Anorexia nervosa
A
  1. Answer: A
    Explanation:
    (Fauci, pp 21-24.)
    The most frequently diagnosed
    psychological disorders in women are depression, anxiety
    disorders, bulimia, and anorexia nervosa. Obsessivecompulsive
    disorders are almost equally distributed
    between adult men and women(prevalence about 2%), but
    with a slightly higher prevalence among boys than girls.
    Psychological disorders may have a higher prevalence in
    women because men are more reluctant to consult a
    physician for emotional problems. Another explanation is
    that physicians may be more apt to diagnose vague mood
    and anxiety complaints as psychological if there is no
    obvious organic basis.
    Source: Ebert 2004
180
Q
  1. A 31-year-old man complains of daily throbbing
    headaches for the last 2 weeks. He has approximately eight
    episodes per day, each lasting 20 min. The headaches are
    localized to the left periorbital area and are accompanied
    by tearing of the left eye, left ptosis, rhinorrhea, and left
    facial redness. The patient remembers having a similar
    problem 2 years ago that lasted for 3 weeks. He did not
    seek medical help at that time. The patient feels that the
    headaches are often precipitated by drinking a glass of
    wine. Which of the following is the most likely diagnosis?
    A. Migraine headache
    B. Cluster headache
    C. Tension headache
    D. Trigeminal neuralgia
    E. Sinusitis
A
  1. Answer: B
    Explanation:
    (Tierney, 42/e, pp 948-949.)
    Cluster headaches are often referred to as “suicide
    headaches” because of the severity of the symptoms. These
    recurring headaches are accompanied by facial fl ushing,
    nasal stuffi ness, tearing, and a partial Horner syndrome
    (there is no anhidrosis). They are more common in men
    (the usual age is 20 to 50)than women and are exacerbated
    by alcohol use.Migraine headaches do not have this timing
    or duration. Tension headaches are bilateral,non throbbing, and symmetric. They are usually located in
    the frontal or occipital areas of the skull and are thought to
    be related to muscle contraction. They are often described
    as being viselike. The headache of sinusitis is not abrupt in
    onset or cessation, and patients often have tenderness with
    percussion of the sinuses. Trigeminal neuralgia (tic
    douloureux) is a paroxysmal severe facial pain over the
    distribution of the trigeminal nerve. Women are affected
    more than men, and patients are usually over the age of 40.
    The pain of trigeminal neuralgia can be triggered by
    simply touching the skin near the nostril.
181
Q
919. For diagnostic lumbar sympathetic block commonest
sites include:
A. L1-L2
B. L2-L3
C. L3-L4
D. L4-L5
E. L5-S1
A
  1. Answer: B

Source: Racz G. Board Review 2003

182
Q
  1. Migraine symptoms are most likely due to:
    A. Vasoconstriction.
    B. Epileptiform discharges.
    C. Cerebral edema.
    D. Decreased cerebral metabolism due to spreading cortical depression.
    E. Vasodilatation
A
  1. Answer: D
    Explanation:
    Studies of migraine have focused on vascular factors
    indicating that vasoconstrictive drugs reduce the
    amplitude of pulsation in the superfi cial temporal artery
    but that this does not always reduce headache.It is believed
    that extracranial vasodilatation is the cause of headache
    and intracranial vasoconstriction is the cause of
    neurological symptoms. Currently, the concept that
    “spreading cortical depression,” which is a primary neural
    (not vascular) event, is the major migraine mechanism.
    This cortical depression leads to hypometabolic state and
    hypoperfusion. The role of unstable serotonergic
    neurotransmission in this cortical depression in migraine
    is being explored. (Neurology 43 [suppl. 3], p. 51, 1993;
    Journal of Neurophysiology 7, pp. 359-390, 1941;
    Source: Neurology for the Psychiatry specialty Board
    Review By Leon A. Weisberg, MD
183
Q
  1. A 35-year old man presented with constant low back pain
    that radiated to the left or right upper buttock region
    with occasional radiation to the thigh and calf posteriorly
    with tingling sensation in the left heel. The symptoms
    started approximately a year ago when he lifted a heavy
    box which caused the gradual onset of low back pain at
    the time with increasing intensity in a week. His motor
    examination was grossly within normal limits. However,
    he had a positive left straight leg raising at 50°. There was
    decreased sensation to pin prick on the lateral side of the
    foot on the left side. The following MRI shows:
    A. L4/5 disc herniation
    B. L5/S1 disc herniation
    C. Large osteophyte pressing on L5 nerve root
    D. Large osteophyte pressing on L4 nerve root
    E. Facet joint arthritis causing spinal stenosis
A
  1. Answer: B
    Explanation:
    Axial T2-weighted MRI scan at the lumbosacral level.
    The arrow shows the degree of disc protrusion and the
    effect that it is having on the pain sensitive anterior part of
    the dural tube (D) and, to some extent, on the S1 nerve
    roots (small white arrows). R = right side of patient. The
    rectangle shows the approximate area shown in C.
    Lateral T2 weighted MRI scan showing the lumbosacral
    spine. S1 = fi rst sacral segment. The posterior disc
    protrusion at the L5/S1 level is shown by the black arrow;
    it can be seen compressing the anterior part of the dural
    tube (D) (thecal sac). Note that the disc is becoming
    ‘black’ between L5 and S1 which indicates that it is
    undergoing dehydration (desiccation) as a result of injury.
    The L4/5 disc shows some early desiccation with
    essentially normal disc hydration at the levels above.
    A 200-micron thick histological section from a cadaver
    with a similar but less extensive, disc protrusion; this is to
    orientate the reader to the various anatomical structures.
    The histological section is represented approximately by
    the area within the rectangle on (D). R = right nerve roots
    budding off from the dural tube (D) containing small nerve roots from the cauda equina (C). H = hyaline
    cartilage on the zygapophysial joint facet surfaces. L =
    ligamentum fl avum; N = spinal nerve; S = spinous process.
    Open arrow head = intervertebral disc protrusion.
    Source: Giles LGF. 50 Challenging Spinal Pain Syndrome
    Cases. Edinburgh, Butterworth Heinemann, 2003.
184
Q
  1. Sumatriptan succinate is effective for the treatment of
    acute migraine headaches by acting as
    A. An antagonist at BETA1 – and BETA2 – adrenergic receptors
    B. A selective antagonist at histamine (H1) receptors
    C. An inhibitor of prostacyclin synthase
    D. An agonist at nicotinic receptors
    E. A selective agonist at 5-hydroxytryptamine 1D (5-HT1D)
    receptors
A
  1. Answer: E
    Explanation:
    Sumatriptan is closely related to serotonin (5-HT) in
    structure, and it is believed that the drug is effective in the
    treatment of acute migraine headaches by virtue of its
    selective agonistic activity at 5-HT1D receptors. These
    receptors, present on cerebral and meningeal arteries,
    mediate vasoconstriction induced by 5-HT. In addition, 5-
    HT1D receptors are found on presynaptic nerve terminals
    and function to inhibit the release of neuropeptides and
    other neurotransmitters.It has been suggested that the
    pain of migraine headaches is caused by vasodilation of
    intracranial blood vessels and stimulation of
    trigeminovascular axons, which cause pain and release
    vasoactive neuropeptides to produce neurogenic
    infl ammation and edema. Sumatriptan acts to reduce
    vasodilation and the release of neurotransmitters and,
    therefore, reduces the pain that is associated with migraine
    headaches. Oher antimigraine drugs (e.g.,ergotamine and
    dihydroergotamine) also exhibit high affi nities for the 5-
    HT1D-receptor site
    Source: Katzung, pp 280-281.
185
Q
923. What of the following is not an indication for a
glossopharyngeal nerve block?
A. Glossopharyngeal neuralgia
B. Atypical facial pain
C. Wisdom tooth extraction
D. Tonsillectomy
E. Pharyngeal cancer pain
A
  1. Answer: C
    Explanation:
    (Raj Pain Review, 2nd Ed.,)
    Primary (idiopathic) and secondary (oropharyngeal
    cancer) glossopharyngeal neuralgia (cranial nerve 9) are
    indications for the block of this nerve. Atypical facial pain
    and tonsillectomy (pre-emptive analgesia) are also
    indications. Maxillary nerve block is indicated for upper
    teeth extraction and mandibular is indicated for lower
    teeth extractions.
    Source: Shah RV, Board Review 2003
186
Q
924. Sphincter detrusor dyssynergia may respond to
transsacral stimulation at:
A. S1 nerve stimulation
B. S2 nerve stimulation
C. S3 nerve stimulation
D. S4 nerve stimulation
E. S5 nerve stimulation
A
  1. Answer: C

Source: Racz G. Board Review 2003

187
Q
  1. A patient complains of worsening chronic headache,
    despite treatment with aspirin, butalbital, caffeine and
    ergotamine. MRI of the head was normal, but MRI of the
    neck demonstrated spondylosis. Headache most likely is
    due to:
    A. Migraine
    B. Drug rebound
    C. Cervical spondylosis
    D. Pseudo-tumor cerebri
    E. Vasodilation due to ergotamine
A
  1. Answer: B
188
Q
  1. A middle aged woman complains of abdominal pain that
    began at age 8. Multiple medical and surgical evaluations
    have been completely within normal limits. The patient
    states that she has “always been sickly” and that her
    mother “had the same problem.” There is no history of
    childhood trauma or abuse. She has been unable to work
    for the last 4 years. Which of the following psychological
    diagnoses best describes her condition?
    A. Somatization disorder
    B. Conversion disorder
    C. Hypochondriasis
    D. Major depression
    E. Generalized anxiety disorder
A
  1. Answer: A
    Explanation:
    A. The patient has a somatization disorder (also called
    hysteria or Briquet’s syndrome).
    * Patients often present with a long history of physical
    complaints before 30 years of age.
    - Many have undergone comprehensive medical
    evaluations and surgical interventions without diagnosis
    of any signifi cant disease process.
    - They also have impaired social development because of their perceived illness.
    - Occupational development is also affected.
    - Many do not work or work at jobs limited by their
    perceived pain.
    B. In conversion disorder, there is loss of a physical
    function that is temporarily related to a psychosocial
    stressor.
    * Sexual dysfunction, pain, blindness, and paralysis have
    been described as manifestations of the psychological
    confl ict experienced by patients with conversion disorder.
    C. Hypochondriasis is the excessive preoccupation with
    disease and with one’s health. Hypochondriacs believe that
    a disease process exists despite medical evaluation and
    reassurance over long periods of time.
    * Patients must pay obsessive attention to perceived pain
    symptoms without signifi cant fear or depression.
    - They complain to family and physicians and are not
    reassured by normal medical examinations and test results.
    D. Major depression is symptomatically different from
    somatization disorder.
    E. Generalized anxiety disorder is symptomatically
    different from somatization disorder.
189
Q
  1. The daughter of a 65-year-old man describes her father as
    having changed from an active, vivacious, caring person
    to one who occasionally has trouble learning new facts,
    has very little motivation to do any activity, and rarely
    expresses feelings or emotions for his grandchildren
    whom he has adored. The area of the brain most apt to be
    involved in this type of behavior change is the
    A. Hypothalamus
    B. Reticular activating system
    C. Heteromodal association areas
    D. Limbic system
    E. Unimodal association areas
A
  1. Answer: D
    Explanation:
    (Carlson, pp 91-94.)
    · The limbic system includes regions of the limbic cortex,
    as well as a group of interconnected structures that
    surround the core of the forebrain.
    · The limbic system forms a circuit whose primary
    function was formerly regarded as modulating motivation
    and emotional responses.
    · Studies have discribed that the hippocampal formation
    and the limbic cortex that surround it are involved in
    learning and memory, rather than emotional behavior.
    However, the remaining sections of the limbic system are
    responsible for emotions, feelings, moods, and motivation.
    Thus, limbic system is the site primarily responsible for
    his learning diffi culty, lack of motivation, and emotional
    feelings.
    Source: Ebert 2004
190
Q
  1. Which of the following is false with respect to tennis
    elbow?
    A. Forearm fl exors are typically involved
    B. The involved muscles have tendinous attachments to the
    lateral epicondyle of the humerus
    C. The backhand stroke may be impaired
    D. Corrective action includes loosening tight racquet
    strings
    E. Corrective action includes enlarging the racquet grip
A
  1. Answer: A
    Explanation:
    (Shah, Musculoskeletal Examination Presentation)
    The wrist extensors are involved, typically due to
    overuse/infl ammation/degeneration at their insertion on
    the lateral epicondyle of the humerus. Forearm fl exors are
    involved in medial epicondylitis, ‘golfer’s’ elbow. The
    above corrective actions are true
    Source: Shah RV, Board Review 2004
191
Q
929. A celiac plexus block is not indicated for:
A. Pancreatic cancer
B. Chronic pancreatitits
C. Sigmoid colon diverticulitis
D. Hepatic metastases
E. Chronic cholecystitis
A
  1. Answer: C
    Explanation:
    (Raj, Pain Review 2nd ed.)
    The classic indication for celiac plexus block and
    neurolysis is pancreatic cancer. The liver and gallbladder
    are also indicated. The sigmoid colon is innervated by the
    lumbar sympathetic chain Sympathetic innervation to the
    gut distal to the mid-transverse colon is supplied by the
    lumbar sympathetics. Note that the celiac plexus contains
    both parasympathetic and sympathetic fi bers. However,
    preganglionic sympathetics coalesce to form the greater
    splanchnic (T5-T9) and lesser splanchnic (T10-T11)
    nerves. These do not synapse in the sympathetic chain but
    synapse in the celiac, aortico-renal, and superior
    mesenteric ganglia.
    Source: Shah RV, Board Review 2003
192
Q
  1. The most common complication associated with a
    supraclavicular brachial plexus block is
    A. Blockade of the phrenic nerve
    B. Intravascular injection into the vertebral artery
    C. Spinal blockade
    D. Blockade of the recurrent laryngeal nerve
    E. Pneumothorax
A
  1. Answer: E
    Explanation:
    The most common complication associated with a
    supraclavicular brachial plexus block is pneumothorax.
    Other potential complications include phrenic nerve
    paralysis, Horner’s syndrome, nerve damage or neuritis, or
    intravascular injection.
193
Q
  1. A 59-year-old female with mild CHF is treated with
    furosemide. What is its primary mechanism of action?
    A. Inhibition of sodium-potassium (Na+, K+) adenosine
    triphosphatase (ATPase)
    B. Inhibition of Na+, K+, choloride (Cl-) co-transporter
    C. Inhibition of Na+, Cl- co-transporter
    D. Inhibition of Cl- transporter
    E. Inhibition of Ca2+ divalent cation (Ca2+) transporter
A
  1. Answer: B
    Explanation:
    The primary action of furosemide is inhibition of the Na+,
    K+,Cl- transporter in the thick ascending limb of the loop
    of Henle
    Source: Hardman, p 697
194
Q
  1. The most frequent work-related musculoskeletal disorder
    found in the upper extremity is:
    A. Carpal tunnel syndrome
    B. Tendopathy of the extensor carpi radialis brevis
    C. Posterior interosseus nerve entrapment
    D. Shoulder external impingement
    E. Cubital tunnel Syndrome
A
  1. Answer: A

Source: Sizer et al - Pain Practice - March & June 2004

195
Q
  1. True statement concerning phantom limb pain is:
    A. Trauma amputees have a higher incidence of phantom
    limb pain than nontrauma amputees
    B. The incidence of phantom limb pain increases with more
    distal amputations
    C. Nerve blocks are commonly used to treat phantom limb
    pain
    D. Most amputees do not experience phantom limb pain
    E. Most phantom limb pain becomes more severe with
    time
A
  1. Answer: C
    Explanation:
    The incidence of phantom limb pain is estimated to be 0
    - 88%. The incidence of phantom limb
    pain does not differ between traumatic and nontraumatic
    amputees.
    The incidence of phantom pain increases with more
    proximal amputation.
    Although very diffi cult to treat, and there is no clinical
    evidence nerve blocks are commonly used in an attempt to
    treat phantom pain. These include trigger point injections,
    peripheral and central nerve blocks, and sympathetic
    blocks.
196
Q
  1. A 60-year-old man ambulates with his upper torso
    stooped forward. His feet shuffl e and he has lost his arm
    swing. Choose correct description of gait:
    A. Ataxic gait
    B. Parkinsonian gait
    C. Spastic hemiplegic gait
    D. Steppage gait
    E. Scissor gait
A
  1. Answer: B
    Explanation:
    (Seidel, Sle, pp 791-792.)
    A. Ataxic gait is often characterized by clumsiness; when
    steps are taken, the advancing foot is lifted high.The foot is
    then brought down in a slapping or stamping manner.
    B. Parkinsonian gait is noted for the forward stoop of the
    head and shoulders, with arms slightly abducted and
    forearms partially fl exed; there is decreased arm swing as
    the feet shuffl e.
    C. Spastic hemiplegic gait is the result of spasticity of the
    involved limb. The limb is moved forward by abduction
    and circumduction.
    D. Steppage gait occurs with footdrop (paralysis of the
    peroneal nerve); the affected foot is raised higher than
    normal to prevent dragging of the toe. Bilateral footdrop
    results in a gait resembling that of a high-stepping horse.
    E. Spastic diplegia gait or scissor gait occurs with
    extrapyramidal disorders. The patient uses short steps and
    drags the foot; the legs are extended and stiff and cross on
    each other.
197
Q
  1. Peripheral nerve stimulation for CRPS II:
    A. peripheral nerve stimulation is more effective than spinal
    cord stimulation
    B. peripheral nerve stimulation and spinal cord stimulation
    together is better than either alone
    C. peripheral nerve stimulation should be used in mononeuropathy
    D. psychological assessment should be done to rule out
    contraindications
    E. All of the above
A
  1. Answer: E

Source: Racz G. Board Review 2003

198
Q
936. Most common indication for gasserian ganglion block is
A. Glossopharyngeal neuralgia
B. Atypical facial pain
C. Trigeminal neuralgia
D. Migraine headache
E. Tension headache
A
  1. Answer: C

Source: Raj, Pain Review 2nd Edition

199
Q
  1. Patrick’s test is a common physical exam technique to
    elicit pain in which condition?
    A. Sacro-iliac joint mechanical dysfunction and pain
    B. Radicular pain due to lumbar spinal stenosis
    C. Radicular pain due to a lumbar disc protrusion
    D. Lumbar facet arthropathy
    E. Lumbar discogenic pain
A
937. Answer: A
Explanation:
(Bonica, 3rd Ed., page 1587; Raj, Pain Review, 2nd Ed.,
page 139)
Source: Shah RV: 2003
200
Q
  1. A 22-year-old woman presents with the chief complaint
    of diplopia for several weeks. She admits to occasional
    vertigo and ataxia. Six months ago, she had urinary
    incontinence for 1 month. Examination of the eyes
    reveals nystagmus, and funduscopic exam reveals
    swelling of the optic nerve (papillitis). The patient has
    increased muscle tone of the lower extremities and is
    hyperrefl exic. She has bilateral extensor plantar refl exes
    and loss of position sense. Which of the following is the
    most likely diagnosis?
    A. Multiple sclerosis
    B. Friedreich’s ataxia
    C. Acute transverse myelitis
    D. Brown-Sequard syndrome
    E. Syringomyelia
A
  1. Answer: A
    Explanation:
    A. The patient most likely has multiple sclerosis, a
    demyelinating disease characterized by visual impairment,
    an afferent pupillary defect (Marcus Gunn pupil),diplopia,
    nystagmus, limb weakness, spasticity, hyperrefl exia,
    extensor plantar refl exes, vertigo, ataxia, dysarthria,
    scanning speech, emotional lability, and bladder
    dysfunction.
    Patients with optic neuritis are at risk for developing
    blindness.
    B. Friedreich’s ataxia is an autosomal recessive disease in
    which young patients present with pes cavus foot
    deformity, spasticity, arefl exia, ataxia, and cardiomyopathy.
    C. Patients with acute transverse myelitis initially present
    with back pain followed by weakness and loss of sensation
    below the level of the pain.
    Often, there may be bladder and bowel incontinence. Transverse myelitis may be seen after vaccination or
    infections.
    D. Brown-Sequard syndrome (cord hemisection) is
    characterized by contralateral loss of pain and temperature
    and ipsilateral spasticity, weakness, hyperreflexia, extensor
    plantar reflex, and loss of proprioception (vibration and
    position sense).
    E. Patients with syringomyelia have bilateral paralysis,
    muscle atrophy, and fasciculations along with pain and
    temperature sensory loss in a shawl-like or capelike
    distribution.
    Source: Tierney, 42/e, pp 983-984.
201
Q
  1. A patient experienced a prolonged stay in one position
    during a recent surgery and postoperative recovery that
    resulted in compression of the common peroneal nerve
    against the fi bular head. Which of the following motor
    defi cits would be most likely to occur?
    A. Loss of extension at the knee
    B. Loss of plantar fl exion
    C. Loss of fl exion at the knee
    D. Loss of eversion
    E. Loss of medial rotation of the tibia
A
  1. Answer: D
    Explanation:
    Compression of the common peroneal nerve would affect
    all muscles innervated by this nerve, including tibialis
    anterior, peroneus longus, and extensor digitorum longus.
    Loss of dorsifl exion and eversion is usually complete. The
    extensors of the knee joint (quadriceps femoris) are
    supplied by the femoral nerve, whereas the fl exors of the
    knee joint (the hamstrings and gracilis) are supplied by the
    tibial nerve and obturator nerve, respectively. The
    gastrocnemius and soleus muscles are the principal plantar
    flexors of the foot and are innervated by the tibial nerve.
    The popliteus is the prime medial rotator of the tibia and
    is also innervated by the tibial nerve.
    Source: Klein RM and McKenzie JC 2002.
202
Q
  1. A patient presents to the emergency room 18 hours
    after recovering from a spinal anesthetic, in which 5%
    lidocaine was used. He complains of moderate to severe
    pain in the lower back, buttocks, and posterior thighs.
    The neurological and genito-urinary exams are normal. A
    lumbar spine MR is normal. What is this patient suffering
    with?
    A. cauda equina syndrome
    B. anterior spinal artery syndrome
    C. transient radicular irritation
    D. spontaneous intracranial hypotension
    E. epidural hematoma
A
  1. Answer: C
    Explanation:
    (Raj, Practical Mgmt of Pain, 3rd Ed., page 371; Stoelting,
    Pharmacology and Physiology of Anesthetic Practice, 3rd
    Ed., page 168-169)
    A. Given the absence of any neurological or GU signs, one
    would most likely suspect transient radicular irritation
    and exclude cauda equina syndrome.
    B. Anterior spinal artery syndrome often presents with
    isolated leg weakness.
    C. Hyperbaric lidocaine that is injected intrathecally can
    present as severe low back, buttock, and groin pain,
    secondary to transient neurological irritation. Treatment
    is conservative.
    D. Spontaneous intracranial hypotension presents with
    headache and neurological symptoms.
    E. The MRI would exclude a new epidural hematoma.
    Source: Shah RV, Board Review 2004
203
Q
941. Increased activity of the sympathetic nervous system
causes
A. Penile erection
B. Pupillary constriction
C. Accommodation for near vision
D. Bronchiolar dilation
E. Gallbladder emptying
A
  1. Answer: D
    Explanation:
    (Rhoades, pp 118-120.)
    Activation of the sympathetic nervous system produces relaxation of the smooth muscles surrounding the
    bronchioles, leading to bronchiolar dilation. The
    parasympathetic nerves are responsible for penile erection,
    pupillary constriction, contraction of the ciliary muscle
    during accommodation for near vision, and
    gallbladder emptying. Sympathetic stimulation causes
    ejaculation and pupillary dilation but does not affect the
    activity of the ciliary muscle or the gallbladder.
204
Q
  1. A 51-year-old alcoholic presents to the emergency room
    with horizontal nystagmus, ataxic gait, and confusion.
    Which of the following is the most likely diagnosis?
    A. Wernicke syndrome
    B. Niacin deficiency
    C. Korsakoff syndrome
    D. Kliiver-Bucy syndrome
    E. Delirium tremens
A
  1. Answer: A
    Explanation:
    (Tierney, 42/e, p 985.)
    The triad of nystagmus and paralysis of eye muscles,ataxia,
    and confusion is associated with Wernicke syndrome.
    Korsakoff syndrome consists of confabulation, confusion,
    and recent memory loss.These disorders are often found in
    thiamine (B1) defi cient malnourished alcoholics and are
    secondary to lesions in the mamillary bodies. Niacin
    defi ciency (pellagra or vitamin B3 defi ciency) causes the
    triad of D’s (Dementia, Dermatitis, and Diarrhea). Kluver-
    Bucy syndrome is due to lesions in the amygdala; patients
    present with hypersexuality, compulsive attention to detail,
    docile behavior, and an inability to recognize objects
    visually (agnosia). Delirium tremens is seen 48 to 96 h
    following abstinence from alcohol; patients present with
    insomnia, confusion, tremors, delusions, visual
    hallucinations, and hyperactivity of the autonomic
    nervous system (i.e., sweating, tachycardia, fever,and
    dilated pupils).
205
Q
  1. The condition in which the covering of the spinal
    cord, along with enclosed neural tissue, forms a saclike
    projection through a dorsal defect in the vertebral
    column is termed
    A. Rachischisis
    B. Anencephaly
    C. Meningocele
    D. Meningomyelocele
    E. Hydrocephaly
A
  1. Answer: D
    Explanation:
    In the family of conditions known as spina bifi da,failure of
    the dural portions of the developing vertebrae may expose
    a portion of the spinal cord and its covering. This usually
    occurs near the caudal end of the neural tube.If there is no
    projection of the spinal cord or its covering through the
    bony defect, the condition is generally hidden (spina bifi da
    occulta). However, it is termed spina bifi da cystica when
    spinal material traverses the defect.
    A. Rachischisis is an extreme example of spina bifi da
    cystica in which the neural folds underlying the vertebral
    defect fail to fuse, leaving an exposed neural plate.
    B.Anencephaly occurs when the cranial neural tube fails to
    fuse, thus resulting in lack of formation of forebrain
    structures and a portion of the enclosing cranium.
    C. In a meningocele, this is a saclike projection formed
    only by the meninges.
    D. If the projection contains neural material, it is a
    meningomyelocele.
    E. Hydrocephaly results from blockage of the narrow
    passageways between the ventricles or between the
    ventricles and the subarachnoid space. Resultant swelling
    of the ventricles compresses the brain against the cranial
    vault and may cause serious mental defi cits.
206
Q
944. The most appropriate drug for reversing myasthenic crisis in a patient who is experiencing diplopia, dysarthria, and difficulty swallowing is
A. Neostigmine
B. Pilocarpine
C. Pralidoxime
D. Succinylcholine
E. Tubocurarine
A
  1. Answer: A
207
Q
945. A supraclavicular brachial plexus block, blocks the
following section of the plexus:
A. Roots
B. Trunks
C. Divisions
D. Cords
E. Branches
A
  1. Answer: B
    Explanation:
    The advantages of the supraclavicular block are fourfold.
    The plexus is blocked where it is most compact, namely at
    the level of the trunks.
    A small volume of anesthetic is required and no part of the
    plexus is spared as with axillary or interscalene block.
    The block can be performed with the arm in any position.
208
Q
  1. A patient presents with right low back and hip pain
    following a motor vehicle accident several weeks ago.
    Pain is made substantially worse with internal rotation of
    the right lower extremity. Hip fl exion and extension are
    not painful. MRI demonstrates an L4/5 disc herniation.
    The source of pain is most likely arises from which of the
    following structures?
    A. Disc
    B. Facet joint
    C. Hip joint
    D. Sacroiliac joint
    E. Piriformis muscle
A
  1. Answer: E
209
Q
  1. A college student has a headache history of 3 months.
    Headache is bilateral, constricting with nausea, but no
    vomiting. Physical and neurological exams are normal.
    The drug of choice is:
    A. Acetaminophen
    B. Oxycodone
    C. Gabapentin
    D. Amitriptyline
    E. Sumatriptan
A
  1. Answer: A
    Explanation:
    The patient most likely has tension headache.
210
Q
948. A patient with rectal cancer with infi ltration develops a
new onset of low back pain. He is on oxycodone and antidepressant therapy. For treatment of low back pain, the
recommended addition is as follows:
A. Ibuprofen
B. Gabapentin
C. Mexiletine
D. Morphine
E. Transdermal Fentanyl
A
  1. Answer: A
211
Q
  1. A 25-year-old woman is involved in a motor vehicle
    accident. Among her injuries is a lumbar vertebral body
    fracture. Which of the following most likely contributed
    to this injury?
    A. Flexion
    B. Extension
    C. Torsion
    D. Spondylolisthesis
    E. Subluxation
A
  1. Answer: A
    Explanation:
    the person is upright. Fracture of a lumbar vertebral body
    may be seen in vehicular accidents when the victim is
    restrained during a high-speed impact by a seat belt
    without a shoulder harness. The rapid and extreme
    forward fl exion of the lumbar spine may produce a variety
    of spinal injuries, ranging from fractures to dislocations.
    Fractures suffered during falls in which the person is
    upright, such as may occur when someone jumps off a
    building, are usually compression fractures of the
    vertebral body. Fracture of the vertebral body will usually
    produce pain coincidental with the injury. Patients with
    fractures of the vertebral body that occur without trauma
    or with inconsequential trauma must be investigated for
    malignant processes, such as metastatic carcinoma,
    multiple myeloma, and unsuspected osteomyelitis.
    Source: Anschel 2004
212
Q
  1. The carpal bone that is most likely to dislocate anteriorly
    and cause a form of carpal tunnel syndrome is the
    A. Capitate
    B. Hamate
    C. Lunate
    D. Navicular
    E. Scaphoid
A
  1. Answer: C
    Explanation:
    A. The capitate is frequently fractured but does not tend to
    dislocate into the carpal arch.
    B. The hamate provides an anchor for the transverse carpal
    ligament and is, therefore, located lateral to the carpal
    tunnel.
    C. The lunate bone tends to dislocate anteriorly into the
    transverse carpal arch, thereby entrapping the tendons of
    the extrinsic digital fl exors and compressing the median
    nerve.
    D, E. The navicular (scaphoid) bone has a tendency to
    fracture but does not dislocate into the carpal tunnel.
213
Q
  1. A 35-year-old man has acute onset of low back pain, lower
    extremity weakness, and bladder dysfunction. He had a
    lumbar laminectomy two years ago. A myelogram shows
    disc herniation L4-5. The most appropriate management
    is
    A. Bed rest
    B. Administration of nonsteroidal anti-inflammatory agent
    C. Epidural administration of a corticosteroid
    D. Epidural administration of a local anesthetic
    E. Surgical decompression
A
  1. Answer: E
214
Q
952. Clinical fi ndings due to S-1 radiculopathy include:
A. Absent ankle (Achilles) refl ex
B. Weakness of foot dorsifl exion
C. Neurogenic bladder
D. Positive unilateral Babinski sign
E. All of the above
A
  1. Answer: A
    Explanation:
    With S-1 radiculopathy, there is reduction of ankle refl ex
    due to gastrocnemius muscle weakness. Dorsifl exion of
    foot is normal, as this involves the L-4 and L-5 roots.
    Neurogenic bladder is seen with spinal cord or S-2, S-3,
    and S-4 root involvement. Babinski sign is seen with
    spinal cord, not spinal root lesions.
    Source: Neurology for the Psychiatry specialty Board
    Review By Leon A. Weisberg, MD
215
Q
953. A herniated T-8 thoracic disk may cause which of these findings:
A. Paraparesis
B. Autonomic bladder
C. Bilateral Babinski signs
D. Absent abdominal refl exes
E. All of the above
A
  1. Answer: E
    Explanation:
    A herniated T-8 thoracic disk may compress the thoracic
    spinal cord, causing all the listed neurological
    disturbances. It can also cause thoracic radiculopathy
    resulting in bandlike sensory disturbance in the thoracic
    or abdominal region. This latter pattern may simulate
    shingles (herpes zoster without rash).
    Source: Neurology for the Psychiatry specialty Board
    Review By Leon A. Weisberg, MD
216
Q
  1. In Hirschsprung’s disease, neural crest cells fail to migrate to, or invade, the wall of the lower colon, resulting in a loss of peristalsis in that region and often fatal obstruction. Preganglionic neurons, which would innervate the absent intramural ganglia, originate in
    A. The nucleus ambiguus
    B. Cervical intermediolateral cell column
    C. Sacral levels two to four of the spinal cord
    D. The motor nucleus of the vagus nerve
    E. The ventral horn at spinal levels T12, L1, L2
A
  1. Answer: C
    Explanation:
    A. The nucleus ambiguus is the source of preganglionic
    parasympathetic neurons that innervate the heart via the
    vagus nerve and cardiac plexus.
    B. Neurons arising in the cervical intermediolateral cell
    column are sympathetic preganglionics.
    C. Preganglionic parasympathetic neurons to the lower
    colon arise from the spinal cord at sacral levels two to four
    and reach the wall of the colon via pelvic splanchnic
    nerves.
    D. Preganglionic parasympathetic neurons arising from
    the motor nucleus of the vagus innervate the upper GI
    tract.
    E. Neurons arising from the ventral horn are primary
    somatic motor neurons to skeletal muscle.
217
Q
  1. A 32-year-old man was admitted for neurologic
    evaluation of a gun-shot wound received fi ve days
    previously A 9-mm bullet had passed through both the
    medial and lateral heads of the gastrocnemius muscle.
    The bullet had not struck bone or signifi cant arteries.
    Neurologic examination revealed losses of dorsifl exion
    and eversion of the left foot. The patient could not feel
    pinprick or touch on the dorsum of the left foot or
    anterolateral surface of the left leg. Which nerve was most
    likely involved in the injury?
    A. Sciatic nerve
    B. Femoral nerve
    C. Sural nerve
    D. Common peroneal nerve
    E. Tibial nerve
A
  1. Answer: D
    Explanation:
    A. The sciatic nerve generally divides into the tibial and
    common peroneal nerves superior to the popliteal fossa.
    Damage to it might result in defi cits in both plantar
    fl exion and dorsifl exion.
    B. The femoral nerve innervates the quadriceps muscles of
    the anterior thigh.
    Damage to it would impair fl exion of the thigh at the
    hip.
    C. The common peroneal nerve innervates all muscles in
    the anterior and lateral compartments of the leg.
    The common peroneal nerve provides sensory
    innervation to the dorsum of the foot and the
    anterolateral surface of the legs via the superfi cial and
    sural/lateral sural cutaneous nerves, respectively.
    D. The common peroneal nerve is the lateral terminal branch of the sciatic nerve.
    After arising near the apex of the popliteal fossa, it
    descends on the popliteus muscle and winds superficially
    around the fibular neck.
    It is extremely vulnerable in this position and is the
    most often injured nerve in the lower extremity.
    E. The tibial nerve innervates plantar flexors of the
    posterior compartment.
218
Q
956. A tumor in the infratemporal fossa may gain entrance to the orbit through which of the following?
A. The optic foramen
B. The ethmoidal sinuses
C. The pterygoid canal
D. The inferior orbital fi ssure
E. The superior orbital fi ssure
A
  1. Answer: D
    Explanation:
    A, E. The optic foramen and superior orbital fi ssure open
    into the middle cranial fossa and transmit the optic nerve
    and the oculomotor, trochlear, and abducens nerves,
    respectively.
    B. The ethmoidal sinuses are mucosa-lined cavities within
    the ethmoid and adjacent bones. They drain into the nasal
    cavity.
    C. The pterygoid canal connects the middle cranial fossa
    with the pterygopalatine fossa and transmits the vidian
    nerve.
    D. The infratemporal fossa communicates directly with
    the orbit via the inferior orbital fi ssure and the
    pterygopalatine fossa.
    The fi ssure normally transmits branches of the
    maxillary nerve and branches of the infraorbital
    vessels.
219
Q
  1. Which statement regarding cervical nerve roots is true:
    A. The C7 spinal nerve exits through the C7-T1 foramen
    B. The C2 spinal nerve exits through the C1-2 neuroforamen.
    C. Sensory innervation to the occiput is supplied primarily
    by branches from C1
    D. The greater occipital nerve originates from the ventral
    root of C2
    E. The C6 and C7 spinal nerves are most commonly involved in cervical radiculopathy
A
  1. Answer: E
    Explanation:
    References:
    2.Gray’s Anatomy, Thirteenth American Edition. Page
    960.
    3.The Anatomic Relation Among the Nerve Roots,
    Intervertebral Foramina, and Intervertebral Discs of the
    Cervical Spine
    4.Tanaka, The Anatomic Relation Among the Nerve Roots,
    Intervertebral Foramina, and Intervertebral Discs of the
    Cervical Spine SPINE 2000;25:286-291
    5.Mercer, The Ligaments and Anulus Fibrosus of Human
    Adult Cervical Intervertebral Discs, SPINE 1999;24:619
    There are 7 cervical spinal levels and 8 cervical spinal
    nerves. The fi rst two cervical nerves (C1 and C2) exit the
    spinal canal posterior to the atlanto-occipital and atlantoaxial
    joints respectively. These two nerves do not exit via a
    foramen. The fi rst cervical foramen is C2-3 which
    transmits the C3 nerve. From C2-3 to C7-T1, the spinal
    nerve exiting the foramen is named by the last number of
    the level (i.e. C3 exits the C2-3 foramen, C4 exits the C3-4
    foramen and C7 exits the C6-7 foramen). The C8 nerve
    exits the C7-T1 foramen. Below T1, the numbering
    convention reverses and the exiting nerve is named for the
    fi rst number of the level (i.e. T2 exits the T2-3 foramen
    and L4 exits the L4-5 foramen).
    The greater occipital nerve is the medial branch of the
    dorsal primary ramus of C2. It supplies most of the
    sensory innervation to the occiput. The C1 spinal nerve is
    primarily motor.
    Degenerative changes of the intervertebral discs and nerve
    root impingement in the intervertebral foramen occur
    most commonly at the C5–C6 and C6–C7 levels. Kelsey et
    al investigated the epidemiology of prolapsed cervical
    discs in an attempt to provide descriptive statistics on this
    disorder and to identify possible risk factors. Most
    patients (75%) had prolapsed discs at either the C5–C6 or
    C6–C7 level. Likewise, according to Murphey et al, the
    frequency of cervical radiculopathy was 26% for C6, 61%
    for C7, and 8% for C8. The incidence of nerve root
    compression is high for C6 and C7.
    Cadaveric dissection data from Tanaka et. Al. predicts a
    higher incidence of radiculopathy for the C5, C6 and C7
    nerve roots. The C5 nerve roots were found to exit over
    the middle aspect of the intervertebral disc,whereas the C6
    and C7 nerve roots were found to traverse the proximal
    part of the disc. The C8 nerve roots had little overlap with
    the C7–T1 disc in the intervertebral foramen. The C6 and
    C7 rootlets passed two disc levels in the dural sac. Also, a
    high incidence of the intradural connections between the
    dorsal rootlets of C5, C6, and C7 segments was found.
    Source: Schultz D, Board Review 2004
220
Q
  1. Pattern of limitation most frequently accompanying
    subacromial impingement
    A. Glenohumeral abduction limits
    B. Glenohumeral adduction limits
    C. Glenohumeral external rotation limits
    D. Glenohumeral internal rotation limits
    E. Glenohumeral external and abduction limits
A
  1. Answer: D

Source: Sizer Et Al - Pain Practice March & June 2003

221
Q
  1. Intervertebral disks have a tendency to herniate into the
    intervertebral foramen because the
    A. Annulus fi brosus is attenuated in the posterolateral
    regions
    B. Interspinous ligament reinforces the disks anteriorly and
    anterolaterally
    C. Ligamentum fl avum reinforces the intervertebral disks
    posteriorly
    D. Lumbar intervertebral disks are thicker posteriorly than
    anteriorly
    E. Posterior longitudinal ligament is stronger and more
    complete posteriorly than posterolaterally
A
  1. Answer: E
    Explanation:
    (Moore, Anatomy, 4/e, pp 451-453.)
    Intervertebral disks are strongly reinforced ventrally
    and laterally by the anterior longitudinal ligaments. The
    posterior longitudinal ligament, although it is denticulate
    and attenuated laterally, reinforces the posterior aspect of
    the intervertebral disk. Because the posterolateral region of the disk is supported least by ligamentous structures, a
    nucleus pulposus that is herniated through the annulus
    fibrosus of the intervertebral disk will take the line of least
    resistance and move posterolaterally into the
    intervertebral foramen.In so doing, the herniation is apt to
    impinge on a spinal nerve of the next lower vertebral
    level.
    Source: Klein RM and McKenzie JC 2002.
222
Q
  1. A 75-year-old female in congestive heart failure (CHF)
    is unable to climb a fl ight of stairs without experiencing shortness of breath. Digoxin is administered to improve
    cardiac muscle contractility. Within two weeks, she has
    marked improvement in her symptoms. What cellular
    action of digoxin accounts for this?
    A. Inhibition of cyclic adenosine 5’-monophosphate
    (cAMP) synthesis
    B. Inhibition of mitochondrial calcium (Ca2+) release
    C. Inhibition of the sodium (Na+) pump
    D. Inhibition of b-adrenergic stimulation
    E. Inhibition of adenosine triphosphate (ATP) degradation
A
  1. Answer: C
223
Q
  1. The DSM-IV classifi cation of psychiatric disorders
    represent a major advance in psychopathology by
    A. Detailing the treatments for various mental disorders
    B. Predicting the outcome of less severe psychological
    problems
    C. Evaluating the effi cacy of various drug treatments
    D. Assessing the potential etiology of abnormal behaviors
    E. Defi ning by empirical criteria a wide variety of psychiatric
    disorders
A
  1. Answer: E
    Explanation:
    (Baum, pp 264-271.)
    The DSM-IV is a multiaxial classifi cation and
    categorization manual for a wide variety of psychological
    disorders. One of its newer contributions is that each
    individual is scored according to broad categories and axes
    so that an individual may be classifi ed as having several
    disorders rather than being forced into a single category or disorder.
    The various categories of axes are Axis I: Primary psychiatric disorders (including physical
    and sexual abuse, medication-induced disorders,
    noncompliance)
    Axis II: Personality disorders and mental retardation (can
    also be used for maladaptive personality and defense
    mechanisms)
    Axis III: General medical conditions (general physical
    health important to the total diagnostic picture)
    Axis IV: Psychosocial and environmental problems
    (family, personal, or situational problems that might affect
    the diagnosis, treatment, or program)
    Axis V: Global assessment of functioning (a scale of the
    level of functioning at the time of evaluation and at other
    time periods)
    Source: Ebert 2004
224
Q
962. Which of the following conditions mimics thalamic pain
syndrome?
A. Wallenberg’s syndrome
B. Syringomyelia
C. Lateral medullary syndrome
D. Parietal cortical lesion
E. Lumbar Radioculopathy
A
  1. Answer: B
    Source: Raj P, Pain medicine - A comprehensive Review -
    Second Edition
225
Q
963. Acute Herpes zoster infection (shingles) of 3 week’s
duration is most appropriately treated by which of the
following?
A. Topical lidocaine patch
B. Peripheral nerve blockade
C. Topical capsaicin cream
D. Spinal cord stimulation
E. Intrathecal steroids
A
  1. Answer: B
    Explanation:
    Lidocaine patches should only be used on intact skin.
    Similarly, capsaicin should not be used on broken skin.
    Peripheral nerve blockade, such as intercostal blocks, is
    the best choice of the options provided. SCS and
    intrathecal steroids may be appropriate for postherpetic
    neuralgia.
226
Q
  1. Among combat veterans, the greatest risk for
    posttraumatic stress disorder is among those who
    A. Were violent prior to service
    B. Have a prior history of depression
    C. Have coexisting sociopathy
    D. Participated in violence towards noncombatants
    E. Have a history of substance abuse
A
  1. Answer: D
    Explanation:
    (Sierles, pp 264-266. Ebert, pp 341-350.)
    Posttraumatic stress disorder (PTSD) is a cluster of
    symptoms that can occur in a person after exposure to a
    severely stressful event (e.g., rape, combat, natural
    disaster). There are three categories of symptoms:
    reexperiencing the event (e.g., nightmares, daydreams,
    obsessions, fl ashbacks), withdrawal (e.g., avoiding movies
    about war and rape and feeling detached from others who
    have not experienced the event), and hyperarousal (e.g.,
    insomnia, irritability, hypervigilance, severe anxiety). Most
    patients with PTSD recover, especially those with good
    premorbid functioning and support. The greatest risk for
    PTSD among combat veterans is among those who killed
    noncombatants, participated in atrocities, or were
    wounded. Another factor associated with increased risk for
    PTSD is violence or behavioral problems, sociopathy, or
    psychiatric disorders prior to the trauma. Substance
    abuse, including alcoholism, is also relevant. It is estimated
    thats many as 480,000 American veterans of the war in
    Vietnam have PTSD.
    Source: Ebert 2004
227
Q
  1. In general, modalities such as heat, electrical stimulation
    and ultrasound:
    A. Should be used until the patient is cured of their pain.
    B. Are the best method to treat patients with chronic pain.
    C. When combined with injections are the only thing necessary
    to treat the majority of pain conditions.
    D. Should be used to facilitate an active exercise program
    for a short course.
    E. Should never be used following interventional techniques
A
  1. Answer: D

Source: Malanga G, Board Review 2003

228
Q
966. A 36-year-old man presents with left hand weakness and atrophy of the first dorsal interosseous muscle. This may indicate damage to spinal roots
A. C5 and C6
B. C6 and C7
C. C7 and C8
D. C8 and T1
E. T1 and T2
A
  1. Answer: D
    Explanation:
    The fi rst dorsal interosseous muscle is innervated by the
    ulnar nerve. The fi bers of the ulnar nerve reaching this
    muscle originate a the C8 and T1 roots. If the ulnar nerve
    itself is the neural element injured, it is usually because of
    damage at the elbow, where the ulnar nerve runs
    superfi cially in the groove over the ulnar condyle. All the
    interosseous muscles of the hand are supplied by the ulnar
    nerve: complete transection of that nerve will produce
    interosseous wasting and impaired fi nger adduction and
    abduction. Although the lumbrical muscles are situated
    alongside the interosseous muscles of the hand, only two
    lumbricals – those on the ulnar metacarpals – are
    innervated by the ulnar nerve. The other two lumbricals
    are innervated by the median nerve. All four lumbricals
    insert on the extensor sheaths of the fi ngers and
    participate in extension of the digits.
    Source: Anschel 2004
229
Q
  1. A 49-year-old woman is brought to the emergency room
    after suddenly losing consciousness. Her husband states
    that the patient was in good health until 2 h ago, when
    she suddenly complained of a severe headache. After
    one episode of vomiting, the patient lost consciousness.
    The husband states that there were no seizure-like
    movements and no incontinence. The patient did not
    take any medications, smoke, drink, or use illicit drugs.
    On physical examination, the patient has a regular heart
    rate of l00/min, respiratory rate of 16/min, and blood
    pressure of 120/80 mmHg, and is afebrile. Heart and
    lung examinations are normal. On neurologic exam, the
    patient responds only to painful stimuli and her deep
    tendon refl exes are bilaterally equal. She has bilateral
    fl exor plantar responses. She has neck stiffness and
    attempts to resist forward fl exion. Which of the following
    is the most likely diagnosis?
    A. Carotid artery thrombosis
    B. Embolic infarction of the brain
    C. Frontal lobe hemorrhage
    D. Subarachnoid hemorrhage
    E. Complicated migraine
A
  1. Answer: D
    Explanation:
    (Tierney, 42/e, pp 961-967.)
    There are three types of stroke: subarachnoid hemorrhage,
    cerebral infarction, and intracerebral hemorrhage. This
    patient presented after complaining of a severe headache.
    She has neck stiffness and no focal defi cit on neurologic
    exam. The loss of consciousness requires bihemispheral
    dysfunction, and this along with the abrupt history is
    most consistent with a subarachnoid hemorrhage (SAH).
    Common causes of SAH include ruptured aneurysm (i.e.,
    berry) and arteriovenous malformation (AVM).
    Intracerebral hemorrhage (ICH) rarely produces coma
    (must be signifi cantly large to do so), and patients do not
    complain of headache (does not involve the meninges).
    Patients with ICH have focal defi cits that appear abruptly
    slowly progress over hours. An embolic stroke can involve
    any carotid artery but must be bilateral to cause loss of
    consciousness. Patients have a history of atrial fi brillation
    or cardiac problems.
230
Q
  1. During a C7 stellate ganglion block, 2 cc of bupivacaine
    with epinephrine were injected. The patient developed
    myoclonic activity and lost consciousness. The injection
    most likely was into the
    A. Subdural space
    B. Vertebral artery
    C. Epidural vein
    D. Subarachnoid space
    E. Internal jugular vein
A
  1. Answer: B
231
Q
  1. A 62-year-old man walks with his feet widely spaced;
    steps occur with each foot lifted abruptly and too high
    and brought down in a stamping manner. Choose correct
    description of gait:
    A. Ataxic gait
    B. Parkinsonian gait
    C. Spastic hemiplegic gait
    D. Steppage gait
    E. Scissor gait
A
  1. Answer: A
    Explanation:
    (Seidel, Sle, pp 791-792.)
    A. Ataxic gait is often characterized by clumsiness; when
    steps are taken, the advancing foot is lifted high.
    The foot is then brought down in a slapping or stamping
    manner. Spastic hemiplegic gait is the result of spasticity
    of the involved limb. The limb is moved forward by
    abduction and circumduction.
    B. Parkinsonian gait is noted for the forward stoop of the head and shoulders, with arms slightly abducted and
    forearms partially fl exed; there is decreased arm swing as
    the feet shuffl e.
    Steppage gait occurs with footdrop (paralysis of the
    peroneal nerve); the affected foot is raised higher than
    normal to prevent dragging of the toe.
    Bilateral footdrop results in a gait resembling that of a
    high-stepping horse.
    C. Spastic diplegia gait or scissor gait occurs with
    extrapyramidal disorders.
    The patient uses short steps and drags the foot; the legs
    are extended and stiff and cross on each other.
232
Q
  1. A 35-year-old woman with Complex Regional Pain
    Syndrome I of the right upper extremity, develops miosis,
    ptosis and enophthalmos after undergoing a stellate
    ganglion block. She does not notice a signifi cant pain
    relief. No signifi cant rise in skin temperature changes was
    recorded to the right upper extremity. What is the most
    likely cause?
    A. Inadequate concentration of the local anesthetic
    B. Intravascular injection
    C. Subarachnoid block
    D. Anomalous Kuntz nerves
    E. Brachial plexus block
A
  1. Answer: D
    Explanation:
    The sympathetic supply to the upper extremity is
    through the grey rami communicantes of C7, C8 and T1
    with occasional contributions from C5 and C6.
    This innervation is through the stellate ganglion. Blocking
    the Stellate ganglion would effectively cause a sympathetic
    denervation of the upper extremity.
    In some cases the upper extremity maybe supplied by the
    T2 and T3 grey rami communicantes. These fi bers do not
    pass through the stellate ganglion.
    These are Kuntz’s fi bers and have been implicated in
    inadequate relief of sympathetically maintained pain
    despite a good stellate ganglion block.
    These fi bers can be blocked by a posterior approach.
    Successful block of the sympathetic fi bers to the head is
    indicated by the appearance of Horner’s syndrome.
    Successful block of the sympathetic block of the upper
    extremity is indicated by a rise in skin temperature,
    engorgement of veins on the back of the hand, loss of skin
    conductance response and a negative sweat test.
    Source: Chopra P. 2004
233
Q
971. A 35-year-old woman falls 12 ft off a ladder and fractures her c-spine, causing damage at the C4 level. She is initially a flaccid quadriplegic with arefl exia. This arefl exia and flaccidity usually evolve into hyperrefl exia and spasticity within
A. 2 to 4 months
B. 1 to 2 months
C. 3 days to 3 weeks
D. 1 to 3 h
E. 5 to 25 min
A
  1. Answer: C
    Explanation:
    Spinal shock is a transient phenomenon that occurs with
    damage to fi bers from upper motor neurons.The spasticity
    that usually develops within a few days of the spinal cord
    injury is presumed to represent exaggeration of the normal
    stretch refl exes in the limbs disconnected from upper
    motor neuron control. The evolution from spinal shock to
    spasticity is much more typical of spinal cord injuries than
    it is of cerebrocortical injuries, but even with
    cerebrocortical injuries there is usually an interval of hors
    to days during which limbs that eventually become
    hyperrefl exic and spastic are hyprorefl exic and fl accid.
    Source: Anschel 2004
234
Q
  1. A 29-year old female with upper extremity complex
    regional pain syndrome undergoes a stellate ganglion
    block in your offi ce pain clinic. She is otherwise healthy
    with normal body habitus and normal airway. She has
    been NPO for 12 hours. 20cc of 0.25% bupivacaine is
    injected incrementally over one minute with no other
    medication administered. 5 minutes after injection
    the patient complains of generalized weakness which
    progresses to complete unresponsiveness, apnea and
    hypotension over the ensuing several minutes.The
    following is the most likely diagnosis:
    A. Overdose of bupivacaine
    B. Total spinal anesthesia
    C. Spinal cord infarction
    D. Anaphylactic shock
    E. Vertebral artery injection
A
  1. Answer: B
    Explanation:
    Reference:
    Gilbert, Complications and Controversies in Regional
    Anesthesia, in ASA Refresher Course, Chapter 6, Volume
    3, ASA 2003
    Neural Blockade, Cousins and Bridenbaugh, Second
    Edition, Chapter 22 Complications of Local Anesthetic
    Neural Blockade, pp. 695-718.
    Total spinal anesthesia refers to the condition in which an
    overdose of intrathecal local anesthetic is administered,
    resulting in blockade of the entire intraspinal neuraxis.
    Patients with total spinal will manifest a complete and
    total, albeit temporary, paralysis. Manifestations include:
    Blockade of C3, C4 and C5 nerve roots (C3, 4 and 5
    keep the diaphragm alive) as well as all thoracic spinal
    nerves resulting in diaphragm and chest wall paralysis
    with apnea.
    Blockade of sympathetic fi bers with hypotension
    secondary to vasodilation and bradycardia.
    Complete muscle paralysis with loss of all voluntary
    movement including speech an eye opening.
    Unless hypotension is severe, the patient may remain
    awake and aware but completely unable to respond. Total
    spinal is the most likely diagnosis here because of the
    signifi cant risk of dural root sleeve injection with stellate
    ganglion block and the delayed and gradual onset of the
    event, taking several minutes to develop.
    The other choices can be eliminated as follows:
    Overdose of bupivacaine:
    20 cc of 0.25% bupivacaine contains 50 mg of
    bupivacaine (one can easily calculate the mg/ml from the
    milliliters and percent of any local anesthetic. Simply
    multiply the percent (0.25) by 10. This will give the
    number of mg per milliliter (2.5). Multiply this number
    by the volume of 20ml to arrive at 50 mg).
    The following are recommended maximum single doses
    for common local anesthetics:
    lidocaine: 300 mg without epinephrine, 500 mg with
    epinephrine
    bupivacaine: 175 mg to 225 mg
    Although there are case reports of cardiac toxicity with
    direct intravascular injection of as little as 50 mg of
    bupivacaine, direct intravascular injection would have
    resulted in immediate, not delayed effects. Soft tissue
    infi ltration overdose of bupivacaine would require a dose
    in the range of 175 mg.
    Spinal cord infarction would be exceedingly unlikely
    from an injection of plain local anesthetic and the time
    course would be quicker.
    Vertebral artery injection would cause immediate
    seizures.
    Anaphylactic shock is a possibility but unlikely with the
    use of an amide local anesthetic.
    Source: Schultz D, Board Review 2004
235
Q
  1. A 36-year old felt a sharp pain in the neck, radiating to the dorsal aspect of the forearm when he was lifting a large
    box. He started experiencing numbness of the thumb
    and index fi nger, with decreased ability to perform biceps
    fl exion. On examination, a diminished biceps refl ex was
    found. What is the most likely cause of the patient’s
    problems?
    A. Fractured C5 vertebra
    B. C5/6 disc protrusion
    C. Facet syndrome at C5/6
    D. Compression of the C5 nerve root by an osteophyte
    E. C4/5 disc protrusion
A
  1. Answer: B
    Explanation:
    The patient has evidence of C6 root compression, most
    likely due to C5/6 disc protrusion. Pain in the neck,
    shoulder, medial scapula, anterior chest, lateral aspect of
    the upper arm, and dorsal aspect of the forearm associated
    with biceps and extensor carpi radialis weakness is
    frequently present.The patient may complain of numbness
    of the thumb and index fi nger. The biceps refl ex may be
    diminished or absent (Wall, p 715)
    Source: Kahn CH, DeSio JM. PreTest Self Assessment and
    Review. Pain Management. New York, McGraw-Hill, Inc.,
    1996.
236
Q
  1. A type I diabetic patient has been treated with relaxation
    techniques daily for one month. This treatment is likely
    to affect the management of her diabetes by
    A. Increased levels of plasma cortisol
    B. Increased sensitivity to insulin
    C. Increased glucose-stimulated secretion of insulin
    D. Signifi cant improvement in glucose tolerance
    E. No signifi cant change in requirements for exogenous
    insulin
A
  1. Answer: D
    Explanation:
    (Taylor, pp 530-531.)
    · The use of relaxation techniques to reduce stress has
    proven very effective.
    · Studies of diabetic patients who practiced progressive
    muscle relaxation showed signifi cant improvement in
    glucose tolerance following relaxation training.
    · Plasma cortisol levels were also reduced in patients
    trained in relaxation.
    · Relaxation, however, did not affect insulin sensitivity or
    glucose-stimulated secretion of insulin.
    · Stress reduction techniques, such as relaxation, are
    effective in reducing requirements for exogenous insulin
    and in the management of both insulin-dependent and
    non-insulin-dependent diabetes.
    Source: Ebert 2004
237
Q
  1. A 31-year-old female has been treated with fl uoxetine for
    two months with no improvement in her depression. You
    decide to switch antidepressant therapy to phenelzine and
    instruct her to wait one week after stopping fl uoxetine to
    start taking the new pills. She begins therapy immediately
    with phenyline without discontinuing fl uoxetine. Two
    days later, she is brought to the ED with unstable vital signs, muscle rigidity, myoclonus, and hyperthermia.
    What caused these fi ndings?
    A. Increased serotonin (5-HT) in synapses
    B. Increased norepinephrine in synapses
    C. Increased acetylcholine in synapses
    D. Increased dopamine in synapses
    E. decreased norepinephrine in synapses
A
  1. Answer: A
    Explanation:
    Reference: Hardman, p 444.
    This patient has the serotonin syndrome.
    Serotonin is already present in increased amounts in
    synapses because of blockade of its reuptake by the SSRIs.
    The amount of serotonin that is present further increased
    when breakdown by MAO is inhibited.
    The serotonin syndrome can be life threatening.
    Source: Stern - 2004
238
Q
  1. A 40-year old construction worker presents with pain
    over the dorsal aspect of the forearm and inability to
    fully extend the arm at the elbow. Physical examination
    reveals diminished sensation over the dorsal aspect of
    the index and middle fi ngers as well as an absent triceps
    refl ex. The most likely diagnosis:
    A. C5
    B. C6
    C. C7
    D. C8
    E. T1
A
  1. Answer: C
    Explanation:
    Pain in the posterior aspect of the arm is likely due to a C7
    root lesions, whereas medial anterior or lateral arm pain
    may be due to C6 or C7 nerve root lesions. A C7 nerve
    root lesion will also produce symptoms (pain and
    paresthesias) in the index and middle fi ngers as well as a
    diminished or absent triceps refl ex. Absence of a
    brachioradialis refl ex is an indication of a C6 nerve root
    lesion (Raj, pp 272-273).
    Source: Kahn CH, DeSio JM. PreTest Self Assessment and
    Review. Pain Management. New York, McGraw-Hill, Inc.,
    1996.
239
Q
977. If nystagmus is a prominent symptom of a cerebellar
lesion, the lesion is within
A. The dentate nucleus
B. The fl occulonodular lobe
C. The lateral cerebellum
D. The cerebrocerebellar cortex
E. The superior cerebellar peduncle
A
  1. Answer: B
    Explanation:
    (Guyton, p 655.)
    The fl occulonodular lobe is known as the
    archeocerebellum because it is, phylogenetically, the oldest
    portion of the cerebellum. It is connected to the vestibular
    nuclei and participates in the control of eye movements.
    Lesions to the fl occulonodular lobe will cause nystagmus.
    Lesions to the other regions of the cortex, the deep nuclei
    of the spinocerebellar tracts, cause a variety of
    abnormalities in motor coordination referred to as ataxia.
240
Q
  1. A patient presents with onset of upper extremity pain.
    The physical examination revealed weakness of elbow
    extension and loss of sensation of the middle fi nger. The
    correct diagnosis in this patient is:
    A. C4 nerve root involvement
    B. C5 nerve root involvement
    C. C6 nerve root involvement
    D. C7 nerve root involvement
    E. C8 nerve root involvement
A
  1. Answer: D
    Explanation:
    The C7 (radial nerve) supplies the triceps, which is the
    primary elbow extensor while the triceps, wrist fl exors,
    and fi nger extensors are partially innervated by the C8,
    they are predominantly C7 muscles.
    C7 supplies sensation to the middle fi nger. Since the
    middle fi nger sensation is also occasionally supplied by C6
    and C8, there is no conclusive way to test the C7 sensation.
    Source: Hoppenfeld S. Orthopaedic Neurology. A
    Diagnostic Guide to Neurologic Levels. Philadelphia,
    LWW, 1997
241
Q
979. Finger fl exion best tests for what nerve root?
A. C5
B. C6
C. C7
D. All of the above
E. None of the above
A
  1. Answer: E

Source: Wirght PD, Board Review 2004

242
Q
  1. A 22-year old woman goes on a date. Following dinner,
    her date tries to be affectionate. She becomes anxious and
    develops weakness with inability to walk. Previous history
    includes sexual abuse at age of 16, with hospitalization
    and psychotherapy. She has improved with treatment
    and diazepam. The most likely diagnosis is
    A. Conversion reaction
    B. Somatoform disorder
    C. Psychoaffective disorder
    D. Fictitious disorder
    E. Malingering
A
  1. Answer: A
243
Q
981. Intrathecal baclofen is indicated for:
A. peripheral neuropathy
B. spasticity from cerebral palsy
C. post-laminectomy syndrome
D. spasticity from fi bromyalgia
E. central thalamic pain
A
  1. Answer: B
    Explanation:
    Intrathecal baclofen is indicated for spasticity from
    cerebral palsy, multiple sclerosis, spinal cord injury, and
    hypoxic brain trauma. Peripheral neuropathy, central thalamic pain, and post laminectomy syndrome are not
    primarily spasticity issues. Although severe cases of
    fibromyalgia have apparently responded to intrathecal
    baclofen, it is not a primary treatment.
    Source: Trescot AM, Board Review 2004
244
Q
982. Intrathecal clonidine may be indicated for all conditions,
EXCEPT:
A. Neuropathic pain
B. Failed laminectomy syndrome
C. Complex regional pain syndrome
D. Cancer pain
E. Lumbar disc herniation
A
  1. Answer: E
    Explanation:
    All of these conditions may respond to intrathecal
    clonidine except for Lumbar disc herniation
    Source: Trescot AM, Board Review 2004
245
Q
  1. When using intrathecal opioids, speed of onset of
    analgesia is:
    A. Directly related to lipid solubility
    B. Inversely related to lipid solubility
    C. Indirectly related to lipid solubility
    D. Unrelated to lipid solubility
    E. Speed and duration are directly related to lipid solubility
A
  1. Answer: A
    Explanation:
    The more lipid soluble the opioid, the faster the onset of
    analgesia. The duration of action is inversely related to the
    lipid solubility.
    Source: Trescot AM, Board Review 2004
246
Q
  1. Which of the following is the most appropriate
    pharmacologic therapy for trigeminal neuralgia
    A. Buprenorphine
    B. Carbamazepine
    C. Chlorpromazine
    D. Pentazocine
    E. Phenelzine
A
  1. Answer: B
    Source: American Board of Anesthesilogy, In-trainnig
    examination
247
Q
  1. Which represents an important diagnostic”red fl ag” in
    the patient with headache?
    A. Abrupt onset with progressively increasing severity
    B. Distinct temporal pulse
    C. Retro-orbital pain with lacrimation lasting 5-10 minutes
    D. Pain worse with extension
    E. Normal bladder function
A
  1. Answer: A

Source: Giordano J, Board Review 2003

248
Q
986. Early medical treatment of CRPS includes:
A. Anti-infl ammatories
B. Steroids
C. Antidepressants
D. Anti-seizure medications
E. Any of the above
A
  1. Answer: E

Source: Racz G. Board Review 2003

249
Q
  1. The following is characteristic of trigeminal neuralgia:
    A. Usually due to multiple sclerosis
    B. Episodes may be aborted by certain antiepileptic or antispasticity
    medications.
    C. Sensory loss is detected on the face.
    D. Weak masseter muscle function.
    E. Bursts of pain last 30-60 min.
A
  1. Answer: B
    Explanation:
    Trigeminal neuralgia develops due to demyelination of the
    trigeminal nerve (sensory portion). This could be due to
    MS plaque, neoplasm in the cerebello-pontine angle, or
    vascular lesion compressing the trigeminal nerve. In most
    cases of trigeminal neuralgia, no etiology is found and
    neurological examination is normal. Bursts of “electrical
    shock” pain usually last less than 30 sec and are confi nedto
    one division of the trigeminal nerve (mandibular is most
    common). Prior to diagnosis being established, dental
    origin for pain is considered, and many patients undergo
    unnecessary tooth extractions. Treatment includes
    carbamazepine, phenytoin, or baclofen. Surgical rhizotomy
    may be needed if medical therapy is not effective. There is
    a theory that the pain is due to compression of the
    trigeminal nerve by abnormal blood vessels, and if this is
    the case, microvascular decompression would be
    warranted.
    Source: Neurology for the Psychiatry specialty Board
    Review By Leon A. Weisberg, MD
250
Q
  1. Parsonage Turner syndrome is:
    A. Also known as idiopathic brachial plexitis
    B. Can be bilateral in 20% of patients
    C. Associated with a 90% recovery rate within 3 years
    D. All of the above
    E. None of the above
A
  1. Answer: D

Source: Wirght PD, Board Review 2004

251
Q
989. If a patient has thunderclap headache and CT scan shows
blood in the left sylvian fi ssure, the next diagnostic study
would be:
A. EEG
B. MRI
C. LP
D. Left carotid angiogram
E. Four-vessel cerebral angiogram
A
  1. Answer: E
    Explanation:
    The term thunderclap headache implies the headache is
    sudden and severe. This pattern should alert the physician
    to the possibility of SAH. Although LP with CSF exam is
    the most defi nitive diagnostic study for SAH, CT was done and showed fi ndings characteristic of ruptured middle
    cerebral artery aneurysm. Since 20% of aneurysms are
    multiple, a four-vessel angiogram is needed to study the
    entire cerebral circulation; whereas a left carotid
    angiogram would likely show the causal aneurysm only
    and not screen for the possibility of multiple aneurysms.
    (Lancet 2, pp. 1247-1248, 1986; postgraduate Medicine 86,
    pp. 93-100, 1989).
    Source: Neurology for the Psychiatry specialty Board
    Review By Leon A. Weisberg, MD
252
Q
  1. In patients with neoplastic conus medullaris compression,
    clinical features usually include:
    A. Symmetrical paraplegia with analgesia at wrist level
    B. Normal ankle jerks
    C. Bladder dysfunction
    D. Plantar fl exor signs
    E. All of the above
A
  1. Answer: A
    Explanation:
    With conus medullaries lesion, the lowest portion of the
    spinal cord would be involved; therefore there would be
    leg weakness with upper motor neuron sings (plantar
    extensor sings) with early autonomic signs and loss of
    ankle refl exes.
    Source: Neurology for the Psychiatry specialty Board
    Review By Leon A. Weisberg, MD
253
Q
991. These drugs are effective in acute migraine management:
A. Isometheptene, dichloralphenazone.
B. Ergotamine
C. Caffeine
D. Imitrex
E. All of the above.
A
  1. Answer: E
    Explanation:
    All listed agents are effective in treating migraine.
    Isometheptene in combination with acetaminophen and
    dichloralphenazone (Midrin) as well as caffeine are
    effective, possibly due to vasoconstrictive effect, despite
    the debate as to whether vascular factors are primary or
    secondary. These medications also affect serotonin
    receptors. Ergotamine is most effective when used
    parenterally and is less orally. Caffeine may enhance the
    effect of ergotamine is most effective and is a serotonin
    receptor agonist.
    Source: Neurology for the Psychiatry specialty Board
    Review By Leon A. Weisberg, MD
254
Q

992.The following is characteristic of “cluster-type” headache:
A. Pupillary dilatation.
B. Relieved by sleep.
C. Long duration of pain episodes.
D. Prominent automatic discharge during headache.
E. Diplopia during attack.

A
  1. Answer: D
    Explanation:
    In cluster, patients awaken with severe short-lived
    headache. This is associated with autonomic dysfunction
    and Horner syndrome. The presence of headache with
    diplopia should suggest ruptured carotid aneurysm with
    oculomotor nerve dysfunction (ptosis, pupillary dilation,
    and extraocular muscle dysfunction). (Neurologic Clinics
    of North America 75, pp. 579-591, 1991;
    Source: Neurology for the Psychiatry specialty Board
    Review By Leon A. Weisberg, MD
255
Q
993. Medications utilized for migraine prophylaxis are:
A. Dihydroergotamine
B. Indomethacin
C. Acetazolamide
D. Calcium channel blockers
E. Sumatriptan
A
  1. Answer: D
    Explanation:
    Migraine treatment may be abortive or prophylactic.
    Drugs that affect the serotonergic brainstem raphe systemergot
    alkaloids, cyproheptadine, methysergide, calcium
    channel blockers, beta-blockers are effective I prophylaxis
    of migraine; whereas other drugs are effective in aborting
    an acute attack.
    Source: Neurology for the Psychiatry specialty Board
    Review By Leon A. Weisberg, MD
256
Q
994. Effective treatment strategies for “status migrainous”
include:
A. Adequate fl uid replacement.
B. DHE and Reglan.
C. Imitrex.
D. Phenothiazines.
E. All of the above
A
  1. Answer: E
    Explanation:
    As a result of vomiting, dehydration may be a signifi cant
    problem. This should be corrected, and pain is frequently
    relieved by rehydration only. Subcutaneous Imitrex is
    effective,but injection may need to be repeated due to pain
    recurrence.Parenterally administered Phenothiazines may
    be effective but may cause postural hypotension.
    Dihydroergotamine (DHE) and antiemetic
    (metoclopramide) Reglan are usually effective in
    refractory migraine. (New England Journal of Medicine
    329, pp. 1476-1482, 1993; Ref. 2, pp. 101-103).
    Source: Neurology for the Psychiatry specialty Board
    Review By Leon A. Weisberg, MD
257
Q
  1. A 30-year-old man develops “the fi rst and worst headache
    of his life” after 5 min of weight lifting. The headache is
    throbbing in quality. It causes him to stop lifting. The
    headache disappears in 10 min. When he goes to the
    emergency department (ED), he is asymptomatic and
    the exam is entirely normal. What is the most likely
    diagnosis?
    A. Subarachnoid hemorrhage.
    B. Bacterial meningitis.
    C. Benign exertional headache.
    D. Intracranial hypertension.
    E. Hypertensive encephalopathy.
A
  1. Answer: C
    Explanation:
    Sudden “thunderclap” headache suggests subarachnoid
    hemorrhage (SAH). Because the headache lasts only 10
    min and then resolves, this suggests effort migraine,
    especially since the patient has no meningeal sings. It
    would be unlikely for pain of SAH to resolve rapidly. Lack
    of fever excludes meningitis; normal blood pressure
    excludes hypertensive encephalopathy; lack of papilledema
    excludes intracranial hypertension. (Lancet 2, pp. 1247-
    1248, 1998).
    Source: Neurology for the Psychiatry specialty Board
    Review By Leon A. Weisberg, MD
258
Q
  1. A patient with lumbar disk disease requires lumbar epidural injection of a corticosteroid for control of low
    back pain. Which of the following statements concerning
    this treatment is true?
    A. Maximum effect occurs one hour after injection
    B. Maximum effect occurs when drug concentration peaks
    in cerebrospinal fl uid
    C. Maximum effect occurs during the acute phase of the
    disease
    D. The benefi cial effect results primarily from sympathetic
    neurolysis
    E. It is contraindicated the patient has had prior surgical
    procedures on the lumbar disks
A
  1. Answer: C
    Source: American Board of Anesthesilogy, In-trainnig
    examination
259
Q
  1. An 18-year-old gymnast heard a popping sound in her
    left knee while practicing for the Olympic Games. Her
    knee immediately became swollen and painful. On
    physical examination, it is obvious that the left knee has
    an effusion. The anterior drawer test and Lachman test
    are positive. McMurray test is negative. Which of the
    following is the most likely diagnosis?
    A. Anterior crudate ligament tear
    B. Posterior crudate ligament tear
    C. Torn medial meniscus
    D. Torn lateral meniscus
    E. Popliteal cyst
A
  1. Answer: A
    Explanation:
    (Seidel, 5/e, pp 737-738.)
    The anterior and posterior cruciate ligaments are
    intraarticular ligaments and contribute to the stability of
    the knee. The most likely diagnosis in this gymnast is tear
    of the anterior cruciate ligament (ACL). Both the Lachman
    test (the patient is placed in the supine position with the
    knee fl exed at 15° while the examiner stabilizes the distal
    thigh with one hand and grasps the patient’s leg distal to
    the tibiofemoral joint with the other hand; the test is
    positive if the examiner is able to move the tibia
    anteriorly) and the anterior drawer test (the foot is
    immobilized while the hip and knee are fl exed, then the
    tibia is moved anterior relative to the femur; a positive test
    occurs with forward displacement of the tibia of more
    than 0.5 cm) are positive in this kind of injury. The
    Lachman test is more sensitive than the drawer test.
    Aspirated joint fl uid is usually bloody in ACL injuries. An
    MRI is helpful in diagnosing this injury. A posterior
    cruciate ligament (PCL) tear would have a positive
    posterior drawer test whereby posterior displacement of
    the tibia is elicited on physical examination. A tom medial
    meniscus often causes the patient to complain of knee
    catching, locking, and clicking. The McMurray test (with
    the patient supine, fl ex the knee and hold the foot in one
    hand; rotate the leg and slowly extend the knee while palpating the posteromedial margins of the joint for a
    palpable click as the femur passes over the tom meniscus)
    is positive for a tom medial meniscus. A tom lateral
    meniscus is tested by palpating the posterolateral margin
    of the knee joint with the leg in full internal rotation as the
    knee is extended. Medial meniscus tears are more common than lateral meniscus tears and are usually due to twisting injuries. Unlike the immediate swelling seen with tears of vascular structures such as the ACL, the relatively
    avascular meniscus (cartilage) causes more gradual
    swelling.
260
Q
  1. If the recurrent laryngeal nerve were transected bilaterally,
    the vocal cords would
    A. Be paralyzed in the open position
    B. Be paralyzed in the closed position
    C. Be paralyzed in the intermediate position
    D. Not be affected unless the superior laryngeal nerve were
    also injured
    E. Appear exactly the same as if an intubating dose of succinylcholine were given
A
  1. Answer: B
    Explanation:
    The recurrent laryngeal nerve innervates all the muscles
    of the larynx except the cricothyroid muscle, which tenses
    the vocal cords and is innervated by the external branch of
    the superior laryngeal nerve.
    Bilateral transections of the recurrent laryngeal nerve
    would produce tense (because the superior laryngeal nerve
    remains intact) closed (because the muscle that opens the
    cords have been denervated) vocal cords. What is actually
    seen are fl accid closed cords.
    The cricothyroid muscle is evidently unable to tense the
    vocal cords without resistance from the other muscle in
    the larynx.
261
Q
999. A patient who presents with an intention tremor, “pastpointing,” and a “drunken” gait might be expected to have a lesion involving the
A. Cerebellum
B. Medulla
C. Cortical motor strip
D. Basal ganglia
E. Eighth cranial nerve
A
  1. Answer: A
    Explanation:
    (Guyton, pp 655-656.)
    Ataxia, dysmetria, and an intention tremor all are classic
    fi ndings in a patient with a lesion involving the
    cerebellum. Affected persons also exhibit
    adiadochokinesia, which is a loss of ability to accomplish a
    swift succession of oscillatory movements, such as
    external and internal rotation of the foot. These symptoms
    all result from destruction of the normal feedback
    mechanisms that are coordinated in the cerebellum
262
Q
1000. Which of the following types of interventional
procedures is associated with the greatest serum
concentration of lidocaine?
A. Intercostal
B. Caudal
C. Epidural
D. Brachial plexus
E. Femoral nerve block
A
  1. Answer: A
    Explanation:
    The site of injection of the local anesthetic is one of the
    most important factors infl uencing systemic local
    anesthetic absorption and toxicity.
    The degree of absorption from the site of injection
    depends on the blood supply to that site. Areas
    that have the greatest blood supply have the greatest
    systemic absorption.
    The greatest plasma concentration of local anesthetic
    occurs after an intercostal block, followed by caudal,
    epidural, brachial plexus, and femoral nerve block.
263
Q
  1. True statements regarding treatment of pain in multiple
    organ/system trauma include all of the following except:
    A. May require more than one modality of analgesia
    B. Head injury is an absolute contraindication to epidural
    placement
    C. An advantage of regional block techniques over IV PCA
    is improved blood fl ow in the area of the block.
    D. An interpleural catheter is a viable alternative for thoracic
    trauma, when an epidural is contraindicated.
    E. In a multitrauma patient needing an epidural catheter,
    treatment of the pain secondary to a thoracic injury
    takes precedent over other injuries.
A
  1. Answer: B
    Explanation:
    Ref: Rowels. Chapter 6. Trauma. In: Pain Medicine: A
    Comprehensive Review, 2nd Edition. Raj, Mosby, 2003,
    page 39-40.
    Source: Day MR, Board Review 2003
264
Q
  1. Spurling’s maneuver is a technique that
    A. is used to evaluate pain emanating from the cervical
    facet joint
    B. involves having the patient actively extend and rotate
    their neck
    C. would help in evaluating a patient that gives a history of
    arm pain that gets better when the arm is allowed rest
    on top the head
    D. evaluates the same problem as a Hoffman’s maneuver
    E. if positive, one would be inclined to order a bone scan
A
  1. Answer: B (pg227)
    Explanation:
    The plain fi lm radiograph depicted above shows a grade I
    spondylolisthesis of L5 on S1 with pars defect. A bone spicule
    projecting into the L5/S1 intervertebral foramen was present
    on the right side. The L5/S1 intervertebral disc was very then
    and there were anterior osteophytes adjacent to on the L5 and
    S1 bodies.
    Clinically, grade I spondylolisthesis of L5 on S1 may be causing
    entrapment of L5 nerve root with probable L5/S1 disc bulge and
    protrusion. However, to rule out further abnormalities with disc
    herniation, patient should undergo either a CT scan or MRI.
    Source: Giles LGF. 50 Challenging Spinal Pain Syndrome Cases.
    Edinburgh, Butterworth
    Heinemann, 2003.
265
Q
1003. The block that could be performed to confi rm the results of the differential epidural block in evaluation of pelvic pain would be
A. Splanchnic block
B. Lumbar sympathetic block
C. Hypogastric plexus block
D. Celiac plexus block
E. Sciatic nerve block
A
  1. Answer: B
    A. Axial T2-weighted MRI scan at the lumbosacral level. The
    arrow shows the degree of disc protrusion and the effect that it is
    having on the pain sensitive anterior part of the dural tube (D)
    and, to some extent, on the S1 nerve roots (small white arrows).
    R = right side of patient. The rectangle shows the approximate
    area shown in C.
    B. Lateral T2 weighted MRI scan showing the lumbosacral
    spine. S1 = fi rst sacral segment. The posterior disc protrusion
    at the L5/S1 level is shown by the black arrow; it can be seen
    compressing the anterior part of the dural tube (D) (thecal sac).
    Note that the disc is becoming ‘black’ between L5 and S1 which
    indicates that it is undergoing dehydration (desiccation) as a
    result of injury. The L4/5 disc shows some early desiccation with
    essentially normal disc hydration at the levels above.
    C. A 200-micron thick histological section from a cadaver with a
    similar but less extensive, disc protrusion; this is to orientate the reader to the various anatomical structures. The histological section is represented approximately by the area within the rectangle
    on (D). R = right nerve roots budding off from the dural tube (D) containing small nerve roots from the cauda equina (C). H =
    hyaline cartilage on the zygapophysial joint facet surfaces. L = ligamentum fl avum; N = spinal nerve; S = spinous process. Open
    arrow head = intervertebral disc protrusion.
    Source: Giles LGF. 50 Challenging Spinal Pain Syndrome Cases. Edinburgh, Butterworth
    Heinemann, 2003.
266
Q
  1. Although a patient was instructed not to use alcohol
    because of a medication he was taking, he did not listen
    to advice and decided to have a drink of alcohol. Within
    minutes, he developed fl ushing, a throbbing headache,
    nausea and vomiting. Which of the following medications
    was he taking?
    A. Naltrexone
    B. Diazepam
    C. Disulfi ram
    D. Phenobarbital
    E. Tranylcypromine
A
  1. Answer: A (pg 228)
    This is a CT scan at the L4/5 disc level. There is a fairly large right-sided soft disc prolapse at L4/5 that is impinging upon the right
    L5 root as it buds off from the theca.
    Source: Giles LGF. 50 Challenging Spinal Pain Syndrome Cases. Edinburgh, Butterworth
    Heinemann, 2003.
267
Q
1005. A dilated pupil in an alert patient would suggest:
A. Adie’s pupil
B. CN II palsy
C. Uncal herniation
D. All of the above
E. None of the above
A
  1. Answer: B
    The MRI in Figure A (pg 228)shows an axial T1 weighted MRI scan showing the right lateral protrusion of the L4/5 disc impinging upon
    the right L5 nerve root. The fi gure B shows a parasagittal T1 weighted MRI scan indicating L4/5 disc protrusion.
    In the examination, only one view may be provided. Most likely it is an axial MRI view.
268
Q
  1. Single-fi ber electromyographic (EMG) recordings are
    helpful in assessing
    A. Sensory nerve fi bers affected by ABC syndrome
    B. Jitter that occurs in some myopathies
    C. Postherpetic neuralgia
    D. Trigeminal neuralgia
    E. All of the above
A
  1. Answer: B

Source: Raj P, Pain medicine - A comprehensive Review -Second Edition

269
Q
1007. Spinal cord stimulation has been demonstrated to be
somewhat effective in which of the following disease
states?
A. Spasmodic torticollis
B. Mixed-migraine headaches
C. Temporal arteritis
D. Cluster headaches
E. Cervical disc herniation
A
  1. Answer: A

Source: Raj P, Pain Medicine - A Comprehensive Review Second Edition

270
Q
1008. A wrist drop would suggest a problem with which
peripheral nerve?
A. Ulnar
B. Median
C. Radial
D. All of the above
E. None of the above
A
  1. Answer: C

Source: Wirght PD, Board Review 2004

271
Q
1009. Each of the following is a potential complication of
lumbar sympathetic blocks EXCEPT
A. Puncture of the renal pelvis
B. Intravascular injection
C. Seizure
D. S1 nerve block
E. Accidental Subarachnoid injection
A
  1. Answer: D
    Explanation:
    Potential complications from lumbar sympathetic block
    include subarachnoid injection, puncture of a major
    vessel or renal pelvis, neuralgia, somatic nerve damage,
    perforation of a disk, infection, ejaculatory failure, and
    chronic back pain.
    Blockade of nerves arising from the lumbar plexus is
    possible, but given the anatomic location of the sacral
    plexus, blockade of an S1 nerve would be extremely
    unlikely if not impossible.
272
Q
1010. Which of the following agents is useful in the treatment
of malignant hyperthermia?
A. Baclofen
B. Diazepam
C. Cyclobenzaprine
D. Dantrolene
E. Halothane
A
  1. Answer: D
    Explanation:
    Reference: Hardman, p 188.
    Malignant hyperthermia (hyperpyrexia), a syndrome that
    is associated with the use of a general anesthetic
    (e.g.,halothane) in conjunction with a skeletal muscle
    relaxant, is characterized by tachycardia, hyperventilation,
    arrhythmias, fever, muscular fasciculation, and rigidity. It
    is caused by a sudden increase in the availability of calcium
    (Ca) ions in the myoplasma of muscle.
    Dantrolene, which interferes with release Ca ions from the
    sarcoplasmic reticulum, is indicated in treatment of the
    disorder. The fi rst three agents are centrally acting skeletal
    muscle relaxants that are not useful in the treatment of
    malignant hyperthermia.
    Source: Stern - 2004
273
Q

1011.The most common surgical option for trigeminal
neuralgia is:
A. Stereotactic radiosurgery
B. Gangliolysis
C. Microvascular decompression of the trigeminal nerve
D. Peripheral neurectomy
E. Intracranial Trigeminal Neurectony

A
  1. Answer: C
    Explanation:
    The artery causing compression over the trigeminal nerve
    as it leaves the Pons is repositioned. If it’s a vein, it is
    coagulated. This surgery has an 85% success rate over 5
    years. The surgical approach is by a suboccipital
    craniectomy.
    Stereotactic radiosurgery is performed using a gamma
    knife or a linear accelerator. A radiosurgical lesion is
    placed in the trigeminal root. The short term results are
    good.
    Gangliolysis is performed by positioning a percutaneous
    needle through the foramen ovale and into the rootlets
    behind the gasserian ganglion. The lesion is made either by
    radiofrequency destruction of the posterior roots,
    infl ation of a balloon in the Meckel’s cave or injection of
    glycerol into the cistern of the trigeminal ganglion. This
    procedure is indicated in debilitated patients who cannot
    tolerate major surgical procedures.
    Peripheral neurectomy is performed by repeat avulsions of
    the peripheral branches of the trigeminal nerve. This is
    sometime performed if patients fail treatment with
    Gangliolysis.
    Bonica - Source: Chopra P, 2004
274
Q
  1. Which of the following appears to have the best outcomes
    in terms of preventing low back injury at the worksite
    A. Back School
    B. Ergonomic adaptations according to NIOSH
    C. Pre-employment physical examination
    D. Prophylactic back belts
    E. Pre-employment X-ray screening
A
  1. Answer: B

Source: Sizer et al - Pain Practice - March & June 2004

275
Q
  1. Mesencephalotomy is a
    A. Procedure done in the midmenstrual cycle
    B. Lesion in the middle of the cerebral hemispheres
    C. Stereotactic lesion not often used today
    D. Special procedure with limited pain treatment
    E. None of the above
A
  1. Answer: C
    Source: Raj P, Pain medicine - A comprehensive Review -
    Second Edition
276
Q
  1. Complications of neurolytic lumbar sympathetic blocks
    with phenol:
    A. genito-femoral nerve neuralgia
    B. lateral femoral cutaneous nerve injury
    C. paralysis of lower extremity
    D. renal pelvis and ureter destruction
    E. all of the above
A
  1. Answer: E

Source: Racz G. Board Review 2003

277
Q
1015. The most common complication of a celiac plexus block
is
A. Hypotension
B. Subarachnoid injection
C. Seizure
D. Retroperitoneal hematoma
E. Constipation
A
  1. Answer: A
    Explanation:
    The sympathectomy produced by a celiac plexus block
    causes hypotension by decreasing pre-load to the heart.
    Subarachnoid injection is the most serious complication
    of celiac plexus block.
    Seizure is possible with an intravascular injection.
    Retroperitoneal hematoma is also possible but extremely
    rare.
    Celiac plexus block frequently relieves constipation by
    interrupting the sympathetic fi bers and leaving the
    parasympathetic fi bers unopposed.
    Source: Hall and Chantigan.
278
Q
  1. A 31-year-old female is treated with an antipsychotic
    agent because of a recent history of spontaneously
    removing her clothing in public places and claiming that
    she hears voices telling her to do so. Her blood pressure
    is normally 130/70 mmHg. Since being treated with a
    drug, she has had several bouts of syncope. Orthostatic
    hypotension was noted on physical examination. Which
    drug most likely caused this?
    A. Haloperidol
    B. Olanzapine
    C. Fluphenazine
    D. Chlorpromazine
    E. Sertindole
A
  1. Answer: D
    Explanation:
    Reference: Katzung, p 482.
    Although many antipsychotic agents can cause orthostatic
    hypotension, chlorpromazine is the most likely choice of
    the agents above for causing this adverse effect.
    Source: Stern - 2004
279
Q
  1. Phalen’s test involves
    A. Tapping on the volar wrist with a refl ex hammer to see if
    paresthesias could be elicited.
    B. using the tips of an unwound paper clip to evaluate areas
    of suspected sensory loss
    C. Actively fl exing the wrist for 30-60 seconds to see if pain
    is reproduced
    D. putting both hands in a prayer position for 30-60 seconds
    E. having the patient bring the thumb perpendicular to the
    palm against resistance
A
  1. Answer: C
    Explanation:
    All of the above are physical exam maneuvers to evaluate
    the median nerve compression in carpal tunnel syndrome.
    Tinel’s involves tapping at the proximal wrist and eliciting
    paresthesias into the index and middle fi ngers including
    the thumb
    2-point discrimination can be easily performed with a
    paper clip.
    Phalen’s involves wrist fl exion to increase carpal tunnel
    pressure
    The ‘prayer’ maneuver involves wrist extension, aka,
    reverse Phalen’s
    Resisted thumb abduction is a way to test the abductor
    pollicis brevis reliably and involvement may mean that the
    median nerve is severely compressed
    Source: Shah RV, Board Review 2004
280
Q
1018. The H reflex is commonly recorded from which
muscle(s):
A. Gastrocnemius
B. Biceps brachii
C. Temporalis
D. Soleus
E. All of the above
A
  1. Answer: D

Source: Wirght PD, Board Review 2004

281
Q
1019. When evaluating peripheral neuropathy, the most
informative nerve to test during nerve conduction studies
would be:
A. Median nerve
B. Sural nerve
C. Ulnar nerve
D. Plantar tibial nerve
E. Axillary nerve
A
  1. Answer: D

Source: Wirght PD, Board Review 2004

282
Q
  1. Spinal cord stimulation in treatment of CRPS:
    A. inhibits sympathetic outfl ow
    B. should be preceded by psychological assessment
    C. should be fi rst tested by trial stimulation
    D. double electrodes need to be close to the “sweet spot”
    E. All of the above
A
  1. Answer: E

Source: Racz G. Board Review 2003

283
Q
  1. Muscle contractions from poorly treated CRPS:
    A. can be surgically corrected by muscle lengthening
    B. should be manually stretched by a strong physical therapist
    C. Botox injections are ineffective
    D. Acupuncture, but only on Yang points are completely
    curative
    E. None of the above
A
  1. Answer: E

Source: Racz G. Board Review 2003

284
Q
1022. CRPS diagnostic absolute “gold standard”:
A. Bone scan
B. 3-phase bone scan
C. osteoporosis
D. overactive sympathetic nervous system
E. none of the above
A
  1. Answer: E

Source: Racz G. Board Review 2003

285
Q
  1. A patient presents with an acute onset of pain in the
    upper extremity. His physical examination showed
    weakness of wrist extension. The sensory examination
    showed hypoesthesia in the lateral forearm. What is the
    most likely involvement of disc herniation and nerve root
    in this patient?
    A. C6/7 disc herniation with C7 nerve root involvement
    B. C7/T1 disc herniation with C8 nerve root involvement
    C. T1/T2 disc herniation with T1 nerve root involvement
    D. C4/5 disc herniation with C5 nerve root involvement
    E. C5/6 disc herniation with C6 nerve root involvement
A
  1. Answer: E
    Explanation:
    Wrist extensors are supplied by C6 and partially by C7.
    The biceps has both C5 and C6 innervation. Under the
    radial extensors, extensor carpi radialis, longus and brevis,
    is supplied by radial nerve C6 in contrast to ulnar
    extensors supplied by extensor carpi ulnaris and C7
    innervation.
    C6 supplies sensation to the lateral forearm, the thumb,the
    index fi nger, and one half of the middle fi nger. To
    remember the C6 sensory distribution more easily, form
    the number 6 with your thumb, index, and middle fi ngers
    by pinching your thumb and index fi nger together while
    extending your middle fi nger.
    Source: Hoppenfeld S. Orthopaedic Neurology. A
    Diagnostic Guide to Neurologic Levels. Philadelphia,
    LWW, 1997.
286
Q
1024. Intermittent Horner syndrome may be seen in this
headache disorder:
A. Migraine with aura.
B. Migraine without aura.
C. Temporal arteritis.
D. Benign intracranial hypertension.
E. Cluster
A
  1. Answer: E
    Explanation:
    Intermittent Horner syndrome is most likely to occur with
    cluster, due to distention of the internal carotid artery
    wall as the sympathetic fi bers travel within the carotid
    artery.Horner syndrome is partial, with ptosis and miosis
    but no anhidrosis.Other autonomic signs are present
    (perspiration, tachycardia, bradycardia, lacrimation),
    which suggests autonomic instability. (Medical Clinics of
    North America 75, pp. 579-591, 1986).
    Source: Neurology for the Psychiatry specialty Board
    Review By Leon A. Weisberg, MD
287
Q
1025. Spasticity can be caused by sectioning
A. The corticospinal fi bers
B. The vestibulospinal fi bers
C. The afferent fi bers
D. The corticoreticular fi bers
E. The reticulospinal fi bers
A
  1. Answer: D
    Explanation:
    (Guyton, pp 639-640.)
    Spasticity results from over activity of the alpha
    motoneurons innervating the skeletal musculature. Under
    normal circumstances, these alpha motoneurons are
    tonically stimulated by reticulospinal and vestibulospinal
    fi bers originating in the brainstem. These brainstem fi bers
    are normally inhibited by fi bers originating in the cortex.
    Cutting the cortical fi bers releases the brainstem fi bers
    from inhibition and results in spasticity Cutting the fi bers
    from the reticular formation, vestibular nuclei, or the Ia
    afferents will reduce the spasticity.
288
Q
  1. A patient presents with pain and paresthesia in the
    left leg. The distribution of the pain-running down the
    medial aspect of the leg and the medial side of the foot
    and including the great toe-is suggestive of a herniated
    intervertebral disk. The most likely location of herniation
    is:
    A. L3-L 4 intervertebral disk
    B. L4-L5 intervertebral disk
    C. L5-S1 intervertebral disk
    D. S1-S2 intervertebral disk
    E. Insuffi cient data to determine
A
  1. Answer: A
    Explanation:
    (April, 3/e, pp 133, 140.)
    The deep incisure in the inferior border of the pedicle
    ensures that the spinal nerve associated with that vertebra
    will exit through the intervertebral foramen well above
    the intervertebral disk so that it will not be affected by
    a herniation at that level. However, a posterolateral
    herniation (the usual direction) will impinge on the next
    lower nerve as it courses toward its associated
    intervertebral foramen. In this case, pain was distributed
    along the medial side of the leg and foot as far as the
    great toe-the distribution of the saphenous branch of
    the femoral nerve (L4). Herniation of the third lumbar
    intervertebral disk between vertebral bodies L3-L4 would
    affect nerve L4.
    Source: Klein RM and McKenzie JC 2002.
289
Q
  1. A middle aged man is administered morphine via
    patient-controlled analgesia (PCA) pump after a left total
    hip arthroplasty. The pump is programmed to deliver a
    maximum dose of 2 mg every 15 minutes (lockout time)
    as needed for patient comfort. The total maximum dose
    that can be delivered in 4 hours is 30 mg. On the fi rst day
    the patient receives 15 doses every 4 hours by pressing the
    delivery button every 15 to 18 minutes. How should his
    pain control be further managed?
    A. Discontinue the PCA pump and administer intramuscular
    morphine
    B. Increase the lockout time from 15 to 25 minutes
    C. Change the analgesic from morphine to fentanyl
    D. Increase the dose to 3mg every 15 minutes as needed up
    to a total maximum dose of 40 mg every 4 hours
    E. Make no changes
A
  1. Answer: D
    Explanation:
    Frequent dosing by a patient receiving postoperative
    analgesia through a PCA pump suggests the need to
    increase the magnitude of the dose.
    A patient also should be given a suffi cient loading dose
    of narcotic before initiative therapy with a
    PCA pump.
290
Q
1028. This fi ndings is characteristic of temporal arteritis:
A. Throbbing headache.
B. Markedly elevated ESR.
C. Tender temporomandibular joint.
D. Active arthritis.
E. Pulsatile, nontender temporal artery
A
  1. Answer: B
    Explanation:
    Headache is more commonly aching than throbbing. Jaw
    pain may occur with chewing, but TMJ tenderness is not
    usually present. The patient complains of joint pain and
    stiffness (polymyalgia rheumatica), but no active arthritis
    is found. The temporal artery is nonpulsative and
    frequently tender. ESR is usually markedly elevated.
    (American Journal of Medicine 67, pp. 839-845, 1972;
    Source: Neurology for the Psychiatry specialty Board
    Review By Leon A. Weisberg, MD
291
Q
1029. Sustained clonus at the ankle is most consistent with:
A. Peripheral neuropathy
B. Polyradiculopathy
C. Myelopathy
D. Anterior horn cell disease
E. None of the above
A
  1. Answer: C

Source: Wirght PD, Board Review 2004

292
Q
1030. Which of the following lie in the carpal tunnel?
A. Transverse carpal ligament
B. Radial artery
C. Flexor carpi radialis
D. Flexor pollicis longus
E. Palmar branch of median nerve
A
  1. Answer: D
    Explanation:
    The transverse (palmar) carpal ligament bounds the carpal
    tunnel, at its volar surface. This ligament is attached to the
    tubercle of the scaphoid and trapezium on the radial side
    and the hamate on the ulnar side. This canal transmits the
    median nerve, but not its palmar branch. Additionally, the
    canal transmits the fl exor digitorum superfi cialis x4,
    fl exor digitorum profundus x4, and fl exor policis longus.
    The radial artery and the FCR do not pass through the
    tunnel
    Source: Shah RV, Board Review 2004
293
Q
1031. Facet joint degeneration (osteoarthropathy) results
from:
A. Mechanical load and stress resulting from disk space
narrowing
B. Lumbar stenosis
C. Spine instability
D. Paget disease
E. All of the above
A
  1. Answer: A
    Explanation:
    Not all patients with back pain due to arthritic etiology
    have a herniated disk. There may be arthritic changes
    which occur in the superior and inferior articular facets
    that result in back pain.
    Source: Neurology for the Psychiatry specialty Board
    Review By Leon A. Weisberg, MD
294
Q
1032. If the patient has low back and hip pain and the pain can be exacerbated by external hip rotation, the most likely
source of the pain is:
A. L-4 radiculopathy
B. Sacro-iliac joint
C. Hip joint pathology
D. Lateral femoral cutaneous neuropathy
E. None of the above
A
  1. Answer: C
    Explanation:
    If back pain is exacerbated by stretch signs (straight-leg
    raising test), consider nerve root compression. If it is
    exacerbated by tenderness over the sacral-iliac joint,
    consider local bursitis. If pain is exacerbated by external
    rotation of the hip, consider hip pathology. Also, consider
    visceral pathology (kidney, stomach, pancreas, aorta,
    colon) as the cause of back pain.
    Source: Neurology for the Psychiatry specialty Board
    Review By Leon A. Weisberg, MD
295
Q
  1. Which represents a diagnostic “red fl ag” when assessing
    the patient with spine pain and/or sciatica?
    A. Periodic”on-off” periods of episodic pain
    B. Pain extending completely to the foot
    C. Progressive neurologic signs and defi cits
    D. All of the above
    E. None of the above
A
  1. Answer: C

Source: Giordano J, Board Review 2003

296
Q
1034. Erb’s Palsy results in:
A. Hypoesthesia in the C5 dermatome
B. Paralysis of scapular muscles
C. Hypoesthesia in the C6 dermatome
D. All of the above
E. None of the above
A
  1. Answer: D

Source: Wirght PD, Board Review 2004

297
Q
  1. A 31-year-old man develops left ankle pain after stepping
    off a curb. He treated the injury with ice overnight but the
    next day cannot walk due to the pain. On examination of
    the ankle, you notice that it is swollen and ecchymotic.
    The anterior and lateral aspects of the ankle are tender to
    palpation. Inversion of the ankle is painful. Which of the
    following is the most likely diagnosis?
    A. Ankle sprain
    B. Rupture of the Achilles tendon
    C. Metatarsal stress fracture
    D. Plantar fasciitis
    E. Tarsal tunnel syndrome
A
  1. Answer: A
    Explanation:
    (Tierney, 42/e, pp 801-802.)
    Ligament injuries of the ankle are common and may occur
    in sports requiring jumping and running. These injuries
    occur when the foot twists as it lands on the ground and
    can even be a consequence of walking on uneven ground.
    The medial ligament is typically injured with eversion and
    the lateral ligament (the ligament most commonly affected
    by injuries) with inversion. The lateral ligament is
    composed of three parts: the anterior talofi bular ligament,
    the calcaneofi bular ligament, and the posterior talofi bular
    ligament. The injured ligament is tender to palpation,
    ecchymotic, and swollen. Metatarsal stress fractures
    (march fractures) occur after long periods of running or
    walking; pain is typically in the middle of the forefoot.
    Rupture of the Achilles tendon may occur with running
    and jumping. It causes a palpable defect, swelling, and
    tenderness over the tendon. The Thompson test is positive
    (patient lies with knee fl exed to 90° and the examiner
    squeezes the calf muscle; if the Achilles tendon is ruptured,
    the foot will not move, but if the tendon is intact, the foot
    will plantarfl ex). Plantar fasciitis causes pain over the
    medial aspect of the plantar fascia. It usually starts slowly
    and is of long duration. The windlass test is positive (pain
    increases with ankle and great toe dorsifl exion). Tarsal
    tunnel syndrome occurs with entrapment of the posterior
    tibial nerve. The patient complains of burning and
    numbness that extends from the sole of the foot and toes
    to the medial malleolus.
298
Q
  1. Which of the following is true regarding anxiety
    disorders and their relationship to pain?
    A. Panic attacks are initiated by fear of movement (kinesiophobia).
    B. Agoraphobia is frequently experienced by patients with
    pain for > 6 months.
    C. Patients in pain are often depressed than they are anxious
    and worried.
    D. 80% of Vietnam vets with PTSD report pain.
    E. Chronic pain patient rarely suffer with anxiety.
A
  1. Answer: D

Source: Cole EB, Board Review 2003

299
Q
1037. What is the neurotransmitter involved in migraine ?
A. Dopamine
B. Acetylcholine
C. Serotonin
D. GABA
E. Norepinephrine
A
  1. Answer: C
    Explanation:
    It is believed that there is unstable serotonin
    neurotransmission in migraine, with increased raphe
    neuronal fi ring rates. During acute migraine attack,
    platelet serotonin levels fall and urinary serotonin
    increases. Serotonin transmission abnormalities in the
    gastrointestinal system explain prominent GI symptoms,
    and affective-mood disturbances are also due to unstable
    CNS serotonin changes. Drugs that treat migraine affect
    serotonin receptors.
    Source: Neurology for the Psychiatry specialty Board
    Review By Leon A. Weisberg, MD
300
Q
1038. Prophylactic medications for migraine are all EXCEPT:
A. Amitriptline
B. Topiramate
C. Verapamil
D. Isometheptene mucate
E. Atenolol
A
  1. Answer: D
    Explanation:
    Drugs used for preventive treatment of migraines are
    tricyclic antidepressants, beta blockers, anticonvulsants
    and calcium channel blockers. Some of the other drugs
    used are SSRI (class of antidepressants, NSAIDS, MAO
    (Monoamine oxidase) inhibitors.. Amitriptyline is a
    tricyclic antidepressant. Topiramate is an anticonvulsant.
    Verapamil is a calcium channel blocker. Atenolol is a beta
    blocker. Isometheptene mucate is used for abortive therapy
    and has no role in prophylactic therapy.
    Source: Chopra P, 2004
301
Q
  1. Which of the following is true with regards to phantom
    limb sensation?
    A. It is strongest in above elbow amputations
    B. More frequent in the non-dominant limb in double
    amputees
    C. Described as an unpleasant, burning sensation
    D. Requires aggressive treatment with medication and interventional
    therapy
    E. It is weakest in above knee amputations
A
  1. Answer: A
    Explanation:
    Ref: Hord and Shannon. Chapter 16. Phantom Pain. In:
    Practical Management Of Pain, 3rd Edition. Raj et al,
    Mosby, 2000, page 212-213
    Source: Day MR, Board Review 2003
302
Q
  1. A patient involved in a work related injury
    approximately 2 weeks ago complains of intractable low
    back and bilateral lower extremity pain. On examination,
    the patient presents with non-physiological signs, which
    included superfi cial tenderness and positive axial loading.
    These fi ndings indicate:
    A. Somatization disorder
    B. Malingering
    C. Conversion disorder
    D. Disc herniation
    E. Fibromyalgia
A
  1. Answer: B
    Explanation:
    Non-physiological signs described by Waddell include
    superfi cial tenderness, axial loading, positive distraction,
    and simulation.
    Overreaction has been removed, thus, 2 positive signs
    indicate non-physiological behavior which may also be
    described as malingering.
    Somatization disorder is different from malingering or
    conversion disorder.
    Non-physiological signs do not confi rm disc herniation or
    fibromyalgia
303
Q
  1. A young woman has a diagnosis of irritable bowel
    syndrome. She has a constitutional predisposition to
    respond physiologically to a situation in a particular
    way, has inadequate homeostatic restraints, and develops
    symptoms when exposed to activating situations. This
    etiological sequence in a psychophysiologic disorder
    follows the
    A. Specifi c-attitudes theory
    B. Diathesis-stress model
    C. Weak organ/system theory
    D. Individual response stereotypes
    E. Specifi c-response pattern model
A
  1. Answer: B
    Explanation:
    (Baum, pp 211-220.)
    · Psychophysiologic disorders were formerly referred to as
    psychosomatic illnesses. They are characterized by
    physical symptoms from organs of the body that have
    become dysfunctional through an interaction between psychological, biologic (including genetic), and
    sociocultural factors.
    · The most common psychophysiologic disorders are
    hypertension, bronchial asthma, dysmenorrhea, headache,
    neurodermatitis, peptic ulcer, irritable bowel syndrome,
    rheumatic arthritis, and ulcerative colitis. Diabetes, along
    with many other diseases, has a strong psychological
    component, but is not considered to be a
    psychophysiologic disorder.
    Source: Ebert 2004
304
Q
  1. Which of the following approaches provides the most
    consistent blockade of the brachial plexus?
    A. Interscalene
    B. Supraclavicular
    C. Infraclavicular
    D. Axillary
    E. Suprascapular
A
  1. Answer: B
    Explanation:
    (Raj, Pain Medicine Review, pages 236-238)
    A. The interscalene targets the roots and may miss C8 and
    even, C7.
    B. The upper, middle, and lower trunks of the brachial
    plexus are predictably pass over the fi rst rib, between the
    insertion of the anterior and middle scalene muscles onthe
    fi rst rib. This is the most compact area of the brachial
    plexus and hence, the supraclavicular approach is the most
    effi cient way to block the brachial plexus.
    C. The infraclavicular approach may block the plexus at
    the level of the musculocutaneous and axillary nerves high
    in the axilla: anesthesia is obtained from the shoulder to
    the hand. However, since the block is at the level of the
    formation of the musculocutaneous and axillary,
    typically inferior and lateral to the coracoid process,
    more cephalad and proximal neural branches may be
    missed.
    Since the brachial plexus is less compact at this level and
    since a neural stimulator is required, less consistent
    blockade may occur.
    D. The axillary approach is indicated for surgery of the
    forearm and hand.
    It primarily targets the radial, median, and ulnar nerves,
    but not the axilla nor musculocutaneous nerves.
    It does not reliably provide analgesia for procedures
    above the elbow.
    E. The suprascapular nerve block relieves afferent pain
    from the shoulder joint and causes motor blockade of the
    supra- and infraspinatus muscles.
    There is no suprscapular approach for brachial plexus
    blocks.
    Source: Shah RV, Board Review 2003
305
Q
  1. Compression of the L4 nerve root will result in all of the following findings except:
    A. Pain in the low back, anterior thigh, and sometimes medial aspect of the lower leg.
    B. Numbness in top medial aspect of the lower leg
    C. Weakness in the quadriceps and sometimes tibialis anterior
    D. Diminished ankle jerk refl ex
    E. Positive straight leg raise
A
  1. Answer: D
    Explanation:
    Ref: Chapter 37. Nerve Root Disorders and Arachnoiditis.
    In: Textbook of Pain, 4th Edition, Wall and Melzack,Churchill Livingston, 1999, page 857.
    Source: Day MR, Board Review 2003
306
Q
  1. A patient with tennis elbow has been refractory to
    conservative drug therapy. As a next step, you would like
    to splint the elbow. Your instructions for splinting are as
    follows:
    A. EF 90° WE 15°
    B. EF 50° WF 20°
    C. EF 70° WF 25°
    D. EF 10° WE 30°
    E. EF 60° WE 30°
A
  1. Answer: A
307
Q
1045. Extensive cord infarction caused by foraminal injection would most likely result from injection of particulate steroid directly into which of the following arteries:
A. Posterior radicular artery
B. Anterior radicular artery
C. Anterior segmental medullary artery
D. Anterior spinal artery
E. Posterior spinal artery
A
  1. Answer: C
    Explanation:
    Reference: Gray’s Anatomy, Thirteenth American Edition.
    Pages 964-971.
    Data from cases of extensive cord infarction after
    transforaminal injection of particulate steroid suggest that
    large portions of the cervical spinal cord can be infarcted
    by intra-arterial injection of particulate steroid into the
    anterior segmental medullary artery. The anterior
    segmental medullary arteries course through various
    neural foramina to connect to the anterior spinal artery
    which in turn delivers blood to the cord parenchyma.
    Injection of particulate steroid into this artery can disrupt
    spinal cord blood fl ow by occluding end-arterioles with
    microcrystal particles. These particles may exceed 20
    microns in diameter and, with intravascular coalescence
    and/or precipitation, much larger particles may be formed.
    The anterior and posterior radicular arteries supply blood
    to the anterior and posterior spinal nerve roots at every
    spinal level bilaterally. These arteries do not supply blood
    to large portions of the spinal cord parenchyma.
    The anterior and posterior spinal arteries are deep within
    the central spinal canal and are not directly accessible by
    intraforaminal injection.
    Source: Schultz D, Board Review 2004
308
Q
1046. A person with which of the following mental disorders
is most apt to seek medical help?
A. Major depressive disorder
B. Bipolar depressive disorder
C. Dysthymic disorder
D. Anxiety disorder
E. Obsessive-compulsive disorder
A
  1. Answer: D
    Explanation:
    (Fauci, pp 2486-2490.)
    Anxiety symptoms are very common in both medically ill
    patients and those otherwise well. Five to 20% of
    inpatients have anxiety symptoms and 5 to 20% of general
    medical outpatients suffer from anxiety states. Patients
    with anxiety disorders are more likely to seek help from
    general physicians and to use emergency room services
    than are patients with other types of mental disorders.
    Furthermore, it has been documented that over the past 15
    years, antianxiety medications have been the most
    frequently prescribed medication in the U.S. Also, primary
    physicians write over 80% of these prescriptions. In terms
    of other mental disorders, over 5% of the U.S. population
    suffers from mood disorders-including major depressive,
    bipolar, and dysthymic-yet they are less apt to seek medical
    help. Panic disorders occur in 1 to 2% of the population
    and 29% of these persons seek help from emergency room
    services. Obsessive-compulsive disorder usually begins in
    adolescence or young adulthood, but is not often
    recognized by general physicians. Help may not be sought
    because of the private nature of the disorder
309
Q
  1. Patients who have somatization disorder are diagnosed
    on the basis of their
    A. Having unexplained symptoms that persist after treatment
    B. Experiencing symptoms in multiple organ systems
    C. Having a history of past and present illnesses that have
    not responded to self-treatment
    D. Having a specifi c number of medically unexplained
    symptoms
    E. Demonstrating positive test results for several chronic
    illnesses at the same time
A
  1. Answer: D
    Explanation:
    (Sierles, pp 266-269. Ebert, pp 366-377.)
    Between 0.5 and 3% of the population experience many
    vague and fl uctuating symptoms in multiple organsystems
    over time. They are explored with medical tests and
    treated, but are never cured. The unexplained symptoms
    can start in childhood, are usually diagnosed by 25 years of
    age, and can continue many years undiagnosed. The
    Diagnostic and Statistical Manual of
    Mental Disorders, 4/e (DSM-IV) lists many relevant
    symptoms from multiple organ and psychological systems.
    A diagnosis of somatization disorder (SD) can be made if
    a patient experiences the following medically unexplained
    symptoms: 4 pain symptoms, 2 gastrointestinal symptoms,
    1 sexual symptom, 1 psychoneurologic symptom, and/or if
    the physical complaints and social or occupational
    impairments are in excess of the expected. This “lumping”
    of so many related and unrelated symptoms into one
    disorder has resulted in some disagreement among
    clinicians who argue that it is label-oriented and does not
    contribute to an understanding of causality or treatment.
    Nevertheless, it is a serious disorder that should receive
    more recognition and research.
    The symptoms have to concern the patient enough to take
    prescribed medication, to change behavior (e.g., to miss
    work), or to consult a physician. Episodes of symptoms,
    sometimes intense, typically last 6 to 9 months, with less
    intense, but continuing symptoms for 9 to 12 months.
    Generally, SD is a lifelong condition, and patients with SD
    consider themselves to be sick. Eighty-six percent report
    that their symptoms are so disabling that their work is
    limited. Seventy-fi ve percent are not employed full-time,
    as compared with 33% of patients with other psychiatric
    diagnosis. When compared with the general population,
    they are more likely to visit doctors, be hospitalized, and
    receive unnecessary surgery. Eighty to 90% report past
    depression, 27% have hysterectomies for non-cancerrelated
    causes, 17 to 25% have irritable bowel syndrome,
    and 12% experience chronic pain.
    SD patients are also at increased risk for panic disorder,
    phobias, general anxiety disorder, obsessive-compulsive
    disorder, and alcoholism; 47% have coexisting personality
    disorders (avoidant, paranoid, and histrionic). Female-tomale
    ratios between 2: 1 and 20: 1 have been reported.
    There is often an inability to identify and articulate their
    emotions, they have diffi culty habituating to stimuli, and
    they receive positive reinforcement from medical
    attention.No treatment cures SD,but patients can be
    taught about SD and taught a relaxation procedure.
    Patients should establish regular doctor visits (versus
    responding to symptoms). Physicians should direct I
    conversation to the patients personal life and a healthy
    lifestyle, while deemphasizing symptoms and praising
    tolerance for symptoms.
    Source: Ebert 2004
310
Q
  1. A 55-year old male presents himself with sudden pain
    and loss of function of the right shoulder fi ve days ago.
    Symptoms started after intense activity. The patient was
    holding the shoulder away from the body in 30° to 40°
    adduction. The pain was presented anteriorly. There
    was no history of recent injury. Aspirin helped his pain
    temporarily. X-were normal. The most likely diagnosis
    is:
    A. Subacromial bursitis
    B. Subcoracoid bursitis
    C. Calcifi c tendonitis
    D. Acromioclavicular joint arthritis
    E. Branchial neuritis
A
  1. Answer: A
    Explanation:
    Subacromial bursitis may occur as a primary disorder after
    a blow to the shoulder, but if, it most frequently occurs
    secondary to degenerative lesions of the rotator cuff and is
    part of the continuum of the many rotator cuff disorders.
    It may be viewed as a separate yet related pathologic
    condition to calcifi c tendonitis. Most of the body’s bursae
    exists in or around the shoulder complex, and they are
    listed up to 12. The most commonly present bursae
    locations include the subacromial and subdeltoid.
    The subdeltoid and subacromial bursae are really one but
    are separately named according to their adjacent anatomic
    structures.
    Bursitis will have a swift onset of extremely severe
    shoulder pain with dramatic tenderness localized to the
    insertion of the deltoid at the upper middle third of the
    anterolateral proximal arm. This is in contrast to more
    diffuse involvement found with impingement of the
    supraspinatus or biceps tendon or pain found adjacent to
    the coracoid process at the medial aspect of the shoulder
    in subcoracoid bursitis.
    The patient maintains the shoulder in an adducted
    position,which keeps the painful lesion away from the
    acromial undersurface. Elevation is hindered, abduction
    more so than forward fl exion, and a painful arc between
    50° and 130° is present whether the movement is active or
    passive.
    On palpation, the physician will fi nd exquisite local
    tenderness over the subacromial bursae, which may feel
    thickened as compared to the contralateral shoulder.
    Tenderness may also extend as far down as the bicipital
    groove. Tests for supraspinatus tendonitis and
    impingement will be positive in this condition.
    Source: Saidoff DC, McDonough AL. Critical Pathways in
    Therapeutic Intervention. Extremities and Spine. St.
    Louis,Inc., 2002
311
Q
1049. Decreased sensation from the nipple line inferiorly
would most likely suggest a lesion at:
A. T1
B. T2
C. T3
D. T4
E. T5
A
  1. Answer: D

Source: Wirght PD, Board Review 2004

312
Q
  1. Examination of a patient’s visual fi elds reveals complete
    blindness in the left eye. Ophthalmoscopic examination
    is normal. Which of the following lesions is most likely
    causing this abnormality?
    A. A lesion between the optic chiasm and the lateral geniculate
    body
    B. A lesion between the retina and the optic chiasm
    C. A lesion between the lateral geniculate body and the
    visual cortex
    D. A lesion at the medial longitudinal fasciculus
    E. A lesion of one occipital lobe
A
  1. Answer: B
    Explanation:
    (Seidel, 5/e, p 311.)
    When defects are detected in only one eye, the lesion must
    be anterior to the optic chiasm.
    Lesions at the optic chiasm produce a bitemporal
    hemianopsia because this is where the nasal retinal fi bers
    decussate. The medial longitudinal fasciculus (MLF) is
    involved with extraocular muscle contraction; a lesion to
    the MLF bilaterally will not allow either eye to look
    medially. Lesions between the geniculate body and the
    visual conex produce a contralateral upper homonymous
    quadrantanopsia. A lesion in the visual cortex (occipital
    lobe) produces similar defects in each eye. Bilateral lesions of the occipital lobes result in complete loss of vision, but
    pupillary refl exes (fi bers end in the midbrain) and
    extraocular muscle movements remain intact.
313
Q
  1. A 60-year old woman presents with a sharp, electric
    shock like, intermittent pain on the left side of her face.
    It is mostly over the cheek and her jaw. She has pain free
    intervals between attacks. She is unable to put on any
    makeup because the slightest touch of a brush sets of her
    pain. All of the following would be an appropriate initial
    medication EXCEPT:
    A. Baclofen
    B. Carbamazepine
    C. Lamotrigine
    D. Carisoprodol
    E. Lidocaine 5% patch
A
  1. Answer: D
    Explanation:
    The treatment of trigeminal neuralgia fi rst
    pharmacological and if this fails then surgical options are
    explored. Pharmacological management consists of
    carbamazepine, phenytoin, gabapentin, lamotrigine,
    baclofen.Carbamazepine is the initial drug of choice and is
    generally started as a single dose of 100mg daily. It is then
    increased every 2 or 3 days by 100mg until good relief is
    obtained. A common dose is at least 200mg per day.The
    common side effects of carbamazepine are
    agranulocytosis, dizziness and sedation. Carisoprodol
    is a muscle relaxant and has no role in the management of
    trigeminal neuralgia.
    The pathology of trigeminal neuralgia is usually at the
    ganglion and no benefi t is obtained using local anesthetics.
    Source: Chopra P, 2004
314
Q
  1. Diplopia following lumbar puncture with a 25-gauge,
    3½-inch needle is the result of
    A. Stretching the abducens nerve
    B. Pressure on the optic nerve
    C. Distortion of the oculomotor nucleus from collapse of
    the wall of the third ventricle
    D. The severity of the accompanying headache
    E. Compensatory cerebral swelling
A
  1. Answer: A
    Source: American Board of Anesthesilogy, In-trainnig
    examination
315
Q
  1. The following is characteristic of migraine with aura:
    A. Fortifi cation spectra.
    B. Headache preceding motor weakness.
    C. Headache preceding aphasia.
    D. Amaurosis fugax and scintillating scotoma.
    E. Headache precipitated by emotional stress.
A
  1. Answer: A
    Explanation:
    Fortifi cation spectra are the most characteristic visual
    disturbance of migraine. These consist of C-shaped
    serrated zig-zag arcs followed by scotoma (area of
    blindness). Visual disturbance recedes before headache
    develops. When headache precedes neurological
    disturbance, consider nonmigraine disorders. Amaurosis
    fugax is visual loss in one eye only and suggests severe
    carotid stenosis. Emotional stress may precipitate
    migraine. Migraine usually develops not at peak stress but
    during a period of relaxation (“let-down”). This is
    contrasted with tension headache, which correlates directly
    with severity of emotional stress. (Archives of Neurology
    36, p. 784, 1979;
    Source: Neurology for the Psychiatry specialty Board
    Review By Leon A. Weisberg, MD
316
Q
1054. Clinical features of carpal tunnel syndrome (CTS)
include:
A. Pain in the forearm
B. Positive Phalen sign
C. Weakness of thumb fl exion
D. Normal triceps refl ex
E. All of the above
A
  1. Answer: E
    Explanation:
    CTS may simulate C-7 cervical radiculopathy. In cervical
    radiculopathy, there would be neck pain and reduced
    triceps refl ex. In CTS, pain is usually in the wrist and
    thumb but may extend to the forearm.
    In CTS, Tinel sign (tapping over the demyelinated median
    nerve at the wrist) and Phalen sign (forced wrist fl exion
    causing sensory symptoms in median nerve distribution)
    are positive.
    Source: Neurology for the Psychiatry specialty Board
    Review By Leon A. Weisberg, MD
317
Q
  1. The following are characteristic of migraine without
    aura:
    A. Bilateral location of pain
    B. Thunderclap quality to pain onset.
    C. Shock or jolt quality of pain
    D. Photopsia and microscopia are present.
    E. Headache is associated with nasal congestion, lacrimation,
    and Horner syndrome
A
  1. Answer: C
    Explanation:
    Head shocks or jolts are quite characteristic of migraine.
    Pain begins as unilateral headache but later becomes
    bilateral. Thunderclap pattern or sudden increase to
    maximal pain severity suggests subarachnoid hemorrhage.
    Visual phenomena suggest migraine with aura, and
    autonomic features suggest cluster.
    Source: Neurology for the Psychiatry specialty Board
    Review By Leon A. Weisberg, MD
318
Q
1056.The spinal cord region responsible for refl exogenic
penile erection is the:
A. Parasympathetic center at S-2 to S-4
B. Sympathetic center at S-2 to S-4
C. Sympathetic center at T-10 to L-2
D. Somatic motor fi bers at S-2 to S-4
E. Hypothalamus
A
  1. Answer: A
    Explanation:
    Penile erection as well as bladder (micturition) and
    rectum (defecation) emptying are controlled by
    parasympathetic (PS) outfl ow through S-2 to S-4 (pelvic
    nerves). Acetylcholine is the primary postganglionic PS
    neurotransmitter. Sympathetic fi bers originating at T-10
    through L-2 play a central role in seminal emission and
    ejaculation and are involved in retention of urine and
    feces.
    Source: Neurology for the Psychiatry specialty Board
    Review By Leon A. Weisberg, MD
319
Q
1057. Sexual dysfunction occurs in these condition
A. Depressive illness
B. Diabetes mellitus
C. Multiple sclerosis
D. Lumbar sympathectomy
E. All of the above
A
  1. Answer: E
    Explanation:
    Sexual dysfunction occurs in all these conditions.
    Depression as well as medications used to treat depression
    should be considered as causal factors. MS causes spinal
    cord dysfunction and depression, and both conditions lead
    to sexual dysfunction. Diabetes may cause autonomic
    neuropathy with sexual dysfunction.
    Source: Neurology for the Psychiatry specialty Board
    Review By Leon A. Weisberg, MD
320
Q
1058.In patients with cauda equina compression, clinical
features usually include:
A. Asymmetric leg weakness
B. Absent ankle and knee refl exes
C. Bladder dysfunction
D. All of the above
E. None of the above
A
  1. Answer: D
    Explanation:
    With cauda equina compression, multiple nerve roots are
    involved. Findings are asymmetrical and autonomic
    dysfunction occurs late, since the spinal cord is not
    compressed. (Ref. 1, p. 449; Ref 2, pp. 593–594)
    Source: Neurology for the Psychiatry specialty Board
    Review By Leon A. Weisberg, MD
321
Q
1059. The following cranial structures are pain sensitive:
A. Venous sinuses.
B. Meningeal arteries.
C. Head and neck muscles.
D. Large cranial arteries.
E. All of the above.
A
  1. Answer: E
    Explanation:
    Pain-sensitive structures include:
    Proximal portion of large extra-and intracranial arteries.
    Large veins and venous sinuses.
    Meninges.
    Upper cervical nerve roots.
    Cranial nerves V, IX, and X.
    Brain parenchyma is pain-insensitive, as are ventricles and
    choroid plexus. Electrode stimulation of the
    periaqueductal gray (PAG) region and somatosensory
    thalamus may cause headache. The descending analgesic
    system includes the mid-brain PAG, medial medullary
    raphe nucleus, reticular formation, and dorsal horn
    neurons of the spinal cord.
    Source: Neurology for the Psychiatry specialty Board
    Review By Leon A. Weisberg, MD
322
Q
  1. The term spinal shock refers to:
    A. Depression of spinal refl ex activity below the level of
    injury
    B. Blood loss and hypovolemia following systemic injury
    C. Loss of motor function following spinal injury
    D. Loss of bladder function following spinal injury
    E. All of the above
A
  1. Answer: A
    Explanation:
    Immediately following spinal cord injury, there is
    electrical-chemical change which enhances inhibitory
    neurotransmission such that all refl exes are absent. Later,
    refl exes become hyperrefl exive.
    In adults, spine or brain traumatic injuries do not cause
    blood loss. With traumatic spinal cord injury, autonomic
    function occurs immediately, and these patients require
    catheterization.
    Source: Neurology for the Psychiatry specialty Board
    Review By Leon A. Weisberg, MD
323
Q
  1. Choose the accurate statement about Visual Analogue
    Scale.
    A. A Visual Analogue Scale consists of a list of adjectives
    describing different levels of pain intensity.
    B. A Visual AnalogueScale consists of a line, usually 10 cm
    long, whose ends are labeled as the extremes of pain (no
    pain to pain as bad as it could be).
    C. A Visual Analogue Scale involves asking patients to rate
    their pain from 0 to 10 on an 11 point scale.
    D. A Visual Analogue Scale employs pain intensity measure
    which is determined by photographs or line drawings
    that illustrate facial expressions of persons.
    E. A Visual Analogue Scale consists of pain intensity which
    describes from faint to very intense.
A
  1. Answer: B
    Explanation:
    A. A verbal rating scale consists of a list of adjectives
    describing different levels of pain intensity. An adequate
    VRS of pain intensity should include adjectives that refl ect
    the extremes of this dimension from no pain to extremely
    intense pain and suffi cient additional adjectives to capture
    the graduations of pain intensity that may be experienced.
    B. A Visual Analogue Pain Scale consist of a line, usually
    10 cm long, whose ends are labeled as the extremes of pain
    (e.g., “no pain” to “pain as bad as it could be”). A VAS may
    have specifi c points along the line that are labeled with
    intensity-denoting adjectives or numbers. Such a scale is
    called a graphic rating scale. Patients are asked to indicate
    which point along the line best represents their pain
    intensity. The distance from the no pain end to the mark
    made by the patient is that patient’s pain intensity score.
    C. A numeric rating scale involves asking patients to rate
    their pain from 0 to 10 (11 point scale), from 0 to 20 (21
    point scale), or from 0 to 100 (101 point scale), with the
    understanding that the 0 represents 1 end of the pain
    intensity continuum with no pain while 10, 20, or 100
    represents the other extreme of pain intensity. Verbal
    rating scales do not require paper and pencil. The patient
    is simply asked to verbally state his or her pain intensity on
    a 0 to 10 or one of the other scales. Nonetheless, a number
    of paper and pencil numeric rating scales exist. The
    validity of numeric pain rating scales has been well
    documented. They demonstrate positive and signifi cant
    correlations with other measures of pain intensity.
    D. Picture or face scales employ photographs or line
    drawings that illustrate facial expressions of persons
    experiencing different levels of pain severity. The patients
    are asked to indicate which one of the illustrations best
    represents their pain experience. Each face has a number
    representing the rank order of pain illustrated, and the
    number associated with the picture chosen by the patient
    represents that individual’s pain intensity score.
    E. Descriptor Differential Scale of pain intensity consists
    of a list of adjectives describing different levels of pain
    intensity. Patients are asked to rate the intensity of their
    pain as being more or less than each word on the list. If
    their experienced pain is greater than that described by the word, they place a check mark on the right of the word in
    proportion to how much greater their pain is. The DDS-1
    has many strengths because it is a multiple-item measure,
    it is possible to assess the internal consistency of the scale,
    and this consistency appears to be very high.
    Descriptor Differential Scale of pain intensity DDS-1
    consists of the words faint, moderate, barely strong,
    intense, weak, strong, very mild, extremely intense, very
    weak, slightly intense, very intense, and mild.
324
Q
  1. Which of the following statements concerning postspinal headache is true?
    A. Cerebrospinal fl uid leucocytosis occurs
    B. Intravenous caffeine therapy is more effective than epidural
    blood patch
    C. The incidence decreases with age
    D. The incidence is higher in males than in females of all
    ages
    E. The incidence is the same after single or multiple dural
    punctures
A
  1. Answer: C
    Source: Neurology for the Psychiatry specialty Board
    Review By Leon A. Weisberg, MD
325
Q
1063. The pathophysiological mechanism which initiate disk
herniation include:
A. Radial tear of annulus fi brosis
B. Prolapse of disk
C. Extrusion of disk
D. Biochemical changes within disk
E. All of the above
A
  1. Answer: A
    Explanation:
    Trauma-induced radial tears in the annulus appear to
    initiate disk herniation. These may be imaged with highresolution
    spinal MRI. With normal aging, disk
    desiccation may occur without disk herniation.
    Source: Neurology for the Psychiatry specialty Board
    Review By Leon A. Weisberg, MD
326
Q
  1. Which of the following is true with respect to carpal
    tunnel syndrome?
    A. Patients develop nocturnal pain and burning in their
    radial three fi ngers and wrist
    B. Phalen’s test is not often used in clinical practice
    C. Reverse Phalen’s test, unlike the Phalen’s test, alleviates
    pressure from the wrist
    D. Hypesthesia is present in the 5th digit
    E. Hypothenar muscle atrophy may be present
A
  1. Answer: A
    Explanation:
    (See musculoskeletal examination presentation by Shah)
    Patients develop hypesthesia of the radial 3 ½ digits.
    Phalen’s and Reverse Phalen’s tests are median nerve
    compression tests that are used in clinical practice.
    Patients develop nocturnal pain and have to shake their
    hands for relief. Thenar muscle atrophy may be present.
    Source: Shah RV, Board Review 2004
327
Q
  1. Which of the following nerves is most likely to be
    injured by fracture of the shaft of the humerus?
    A. Axillary
    B. Median
    C. Musculocutaneous
    D. Radial
    E. Ulnar
A
  1. Answer: B
    Source: American Board of Anesthesilogy, In-trainnig
    examination
328
Q
  1. A 39-year-old female patient presents with a 4-month
    history of sharp right lateral elbos pain after she suffered
    a fall to the outstreached hand at work. She complains
    of an intense sharp pain with use of right elbow on work
    activities. Upon examining the patient, you fi nd she had
    a (+) lateral pivot shift test. There was no evidence of
    fracture. The likely diagnosis is:
    A. Tendopathy of the extensor carpi radialis brevis
    B. Lateral ulanr collateral ligament instability
    C. Humeroradial joint degeneration
    D. Posterior interosseus nerve entrapment
    E. Laterl Epicondylitis
A
  1. Answer: B

Source: Sizer et al - Pain Practice - March & June 2004

329
Q
  1. Bupivacaine is more likely than lidocaine to cause
    refractory cardiac arrest because bupivacaine
    A. Has a lower rate of plasma clearance
    B. Has a secondary blocking effect on cardiac beta1-adrenergic
    receptors
    C. Dissociates more slowly from sodium channels in cardiac
    muscle
    D. Inhibits spontaneous space 4 decolorization in pacemaker
    cells
    E. Preferentially blocks calcium channels in Purkinje fi bers
A
  1. Answer: C
    Source: American Board of Anesthesilogy, In-trainnig
    examination
330
Q
  1. In normal tissue, which property of drugs has the
    greatest effect on the speed on onset of a local anesthetic?
    A. Amide structure
    B. Degree of protein binding
    C. Intrinsic vasoconstrictor activity
    D. pKa
    E. Potency
A
  1. Answer: D
    Source: American Board of Anesthesilogy, In-trainnig
    examination
331
Q
  1. The plasma concentration of equal doses of a local
    anesthetic is highest when the site of administration is
    A. Axillary brachial plexus
    B. Caudal
    C. Intercostal
    D. Lumbar epidural
    E. Subcutaneous
A
  1. Answer: C
    Source: American Board of Anesthesilogy, In-trainnig
    examination
332
Q

1070.Which of the following statements concerning
interscalene brachial plexus block is true?
A. The three trunks of the plexus are in the same fascial
plane as the internal jugular vein
B. Distal spread of anesthetic past the humeral head is accelerated
by adduction of the arm
C. Anesthetic solution can spread up the fascial sheaths to
involve the stellate ganglion
D. Ipsilateral diaphragmatic paralysis results from epidural
spread
E. Rich vascularity in the sheaths promotes rapid vascular
uptake of anesthetic

A
  1. Answer: C
    Source: American Board of Anesthesilogy, In-trainnig
    examination
333
Q
  1. Neurolytic block is most appropriate for
    A. Abdominal pain secondary to hepatic carcinoma
    B. Abdominal pain secondary to pancreatitis
    C. Persistent chest wall pain secondary to intercostal neuralgia
    following a thoracotomy for trauma.
    D. Reflex sympathetic dystrophy of the upper extremity
    with an excellent but transient response to a series of
    stellate ganglion blocks with local anesthetic
    E. A diabetic patient scheduled for surgical sympathectomy
    to relieve unilateral lower extremity pain secondary to
    severe peripheral vascular disease.
A
  1. Answer: A

Source: American Board of Anesthesiology, In-trainnig examination

334
Q
1072. To evaluate warm temperature sensation, the stimulus should be in which of the following temperature ranges?
A. 25 to 29°C
B. 30 to 35°C
C. 36 to 39°C
D. 40 to 45°C
E. 46 to 50°C
A
  1. Answer: D
    Explanation:
    To test warm temperature sensation, a glass or metal tube
    containing hot water with a temperature in the range of 40
    to 45°C (104 to 113°F) should be used.
    Temperatures higher than 45°C are perceived as painful.
    Source: Raj P
335
Q
1073. Injection of local anesthetic into the interspace between the 3rd and 4th toes may provide relief of which
condition
A. Morton’s neuroma
B. Metarsalgia
C. Plantar Fasciitis
D. Painful heel spur
E. Tarsal Tunnel Sundrome
A
  1. Answer: A
    Explanation:
    ( Raj, {Practical Mgmt of Pain, 3rd Ed., page 355-6, 371)
    A. Morton’s neuroma is the most common form of
    interdigital neuritis. It typically occurs between the third
    and fourth toes and rarely between the second and third
    toes. Pain can be produced between the metatarsal heads,
    which differentiates this condition from metatarsalgia, in
    which pain is elicited with pressure against the plantar foot
    under the metatarsal heads.
    B. Pain along the plantar surface of the metatarsal heads
    causes weight-bearing discomfort with each step and can
    be replicated with manual compression. About 80% of the
    weight is borne by the fi rst metatarsal head, but in
    pronation, weight is shifted over the second and third toes
    and painful repetitive trauma can accumulate. Pain is
    typically increased in combined pronation and eversion,
    and this gait
    C. Plantar fasciitis is commonly found in those who must
    stand on hard fl oors for long periods of time and is an
    infl ammation of tendon and fascia and as they insert into
    the calcaneal periosteum.Bone growth in the direction of
    pull is frequently found as a calcaneal spur. The examiner
    can elicit pain with plantar compression over the anterior
    calcaneus, but pain may radiate along the plantar fascia.
    D. Painful heel syndrome is often diagnosed in morbidly
    overweight people or those who stand or walk
    excessively.Degeneration of the normal heel compression
    allows injury to weight-bearing surfaces of the calcaneus.
    Frequently, symptoms are increased in the morning or
    after a prolonged rest. Examination fi ndings are similar to
    those in plantar fasciitis, but pain tends to be posterior and
    localized to the plantar calcaneus.
    E. The posterior tibial nerve is derived from L4 through
    S3 roots and may be compressed in the tarsal tunnel.Nerve
    conduction studies show prolongation of the distal motor
    and sensory latency of the tibial nerve.There may be EMG
    changes in the appropriate foot muscles. This syndrome is
    relatively uncommon.
    Source: Shah RV, Board Review 2004
336
Q
  1. A 19-year-old female whose roommate is being treated
    for depression decides that she is also depressed and
    secretly takes her roommate’s pills “ as directed on the
    bottle” for several days. One night, she makes herself a
    snack of chicken liver pate and bleu cheese, accompanied
    by a glass of red wine. She soon develops headache,
    nausea, and palpitations. She goes to the ED, where her
    blood pressure is found to be 200/110mmHg. What
    antidepressant did she take?
    A. Sertraline
    B. Phenelzine
    C. Nortriptyline
    D. Trazodone
    E. Fluoxetine
A
  1. Answer: B
    Explanation:Reference: Hardman, p 444.
    This patient ate tyramine-rich foods while taking an
    MAOIand went into hypertensive crisis.
    Tyramine causes release of stored catecholamines from
    presynaptic terminals, which can cause hypertension,
    headache, tachycardia, cardiac arrhymias, nausea, and
    stroke.
    In patients who do not take MAOIs,tyramine is inactivated
    in the gut by MAO, and patients taking MAOIs must be
    warned about the dangers of eating tyramine-rich foods.
    Source: Stern - 2004
337
Q
  1. Which of the following is the most important
    disadvantage of interscalene brachial plexus block
    compared with other approaches?
    A. Frequent sparing of the musculocutaneous nerve
    B. High incidence of pneumothorax
    C. Not suitable for operations on the shoulder
    D. Large volumes of local anesthetics required
    E. Frequent sparing of the ulnar nerve
A
  1. Answer: E
    Explanation:
    The major disadvantage of the interscalene block for
    hand and forearm surgery is that blockade of the inferior
    trunk (C8-T1) is often incomplete.
    Supplementation of the ulnar nerve is often required.
    The risk of pneumothorax is quite low, but blockade of
    the ipsilateral phrenic nerve occurs in up to 100% of
    blocks. This can cause respiratory compromise in patients
    with signifi cant lung disease.
338
Q
1076. The immune system is not an autonomous system, it can be altered by
A. Relaxation
B. Stress
C. Suppressed emotions
D. Conditioning
E. Diet
A
  1. Answer: D
    Explanation:
    (Baum, pp 169-173.)
    · Ader and Cohen discovered that the immune system
    could be conditioned by neutral taste stimuli. Follow-up
    studies reconfi rmed that immune system responses can be
    conditioned to neutral stimuli in both animals and
    humans.
    · Immunologists previously had assumed that the immune
    system was autonomous. Newer studies in this area have
    demonstrated that many immune components can be
    altered by behavioral factors such as stress, depression,
    isolation, relaxation, and bereavement.
    · All of the options listed in the question have some effect
    on the immune system, but the ability to modify the
    immune system by conditioning is most important.
    Source: Ebert 2004
339
Q
  1. The following statement is false regarding local
    anesthetic toxicity:
    A. Hyperventilation of a patient with a suspected overdose
    of local anesthetic will make seizures less likely.
    B. Lidocaine overdose causes seizures before cardiac depression.
    C. Toxic dose for direct intravascular injection of lidocaine
    is 300mg without epinephrine and 500mg when epinephrine
    is added.
    D. Intravenous benzodiazepines are recommended to treat
    local anesthetic-induced seizures
    E. Cardiac arrest from bupivacaine overdose is exceedingly
    diffi cult to treat and prolonged resuscitation with large
    doses of epinephrine may be required.
A
  1. Answer: C
    Explanation:
    References:
    Neural Blockade, Cousins and Bridenbaugh, Second
    Edition, Chapter 22 Complications of Local Anesthetic
    Neural Blockade, pp. 695-718.
    Waldman, Interventional Pain Management, Second
    Edition; Chapter 16 Local Anesthetics in Clinical Practice,
    pp. 214-218
    Elevation of pCO2 and acidosis tend to increase the toxic
    effects of local anesthetics by the following mechanisms:
    Elevated pCO2 causes cerebral vasodilation, delivering
    more local anesthetic to the brain
    Decrease in intracellular pH will convert more local
    anesthetic from the inactive base form to the cationic form
    which is active on the nerve membranes.
    Hypercarbia and acidosis decrease protein binding of
    local anesthetics increasing the portion of free drug
    available
    Conversely, decreases in pCO2 and elevations in pH tend
    to elevate the seizure threshold for local anesthetics by the
    same mechanisms.
    Lidocaine will almost invariable cause CNS effects prior to
    causing cardiac toxicity. First the patient may complain of
    dizziness, tinnitus and diffi culty focusing eyes. Increasing
    toxicity causes muscle twitching and tremors involving
    the face and distal extremities which progresses to grand
    mal seizures. Cardiac arrythmias occur late and only with
    massive overdose. The maximum dose for lidocaine soft
    tissue infi ltration is 300mg without epinephrine and
    500mg when epinephrine is added. This does not imply
    that 300mg can be given directly IV as a bolus dose. The
    toxic dose for direct intravenous injection of any local
    anesthetic is much lower than the toxic dose for tissue
    infi ltration. For instance, the tissue infi ltration maximum
    dose for bupivacaine is approximately 200mg whereas
    doses as small as 50mg have caused cardiac toxicity when
    administered directly IV.
    Bupivacaine has a much higher potential to cause lifethreatening
    cardiac arrhythmias than lidocaine. Whereas
    lidocaine is a fast-in/fast-out calcium channel blocker that
    reaches steady state block in one to two heartbeats. In
    contrast, bupivacaine is a fast-in/slow-out blocker
    manifesting a blocking action that increases with
    successive beats and with faster rates creating the potential
    for malignant re-entrant cardiac arrhythmias.
    Resuscitation from bupivacaine cardiac toxicity is diffi cult
    and may require prolonged efforts with high doses of
    epinephrine. There is no specifi c antidote or reversal agent
    for bupivacaine overdose.
    Benzodiazepenes increase the seizure threshold in the
    brain and are the treatment of choice for local anesthetic
    induced seizures.
    Source: Schultz D, Board Review 2004
340
Q
1078. The central anticholinergic syndrome is LEAST likely to occur after administration of
A. Atropine
B. Chlorpromazine
C. Diphenhydramine
D. Glycopyrrolate
E. Scopolamine
A
  1. Answer: D
    Source: American Board of Anesthesilogy, In-trainnig
    examination
341
Q
  1. A 32-year-old previously healthy man is brought to the
    emergency room after having a seizure. He has no family
    history of seizure and denies alcohol use, illicit drug
    use, or trauma. A family member states that recently
    the patient has been complaining of a headache and has
    been acting bizarre, which is a change in his personality.
    Physical examination reveals a temperature of 100.9°F.
    Blood pressure and heart rate are normal. During
    examination, the patient has a partial complex seizure.
    CT scan of the head reveals hemorrhagic necrosis of the
    temporal lobes. Which of the following is the most likely
    diagnosis?
    A. Lyme disease
    B. Cysticercosis
    C. Progressive multifocal leukoencephalopathy
    D. Herpes encephalitis
    E. Rabies
A
  1. Answer: D
    Explanation:
    (Tierney, 42/e, p 1305.)
    A. Lyme disease can produce an encephalitis or
    demyelination that mimics multiple sclerosis, but
    infection follows a tick bite. Waterhouse-Friderichsen
    syndrome is hemorrhagic infarction of the adrenal glands
    due to fulminant menigococcemia.
    B. Cysticercosis is characterized by multiple brain cysts
    produced by the larval form of the pork tapeworm (Taenia
    solium).
    C. Progressive multifocal leukoencephalopathy (PML) is
    a human papovavirus (JC virus) seen in patients with
    AIDS.
    Patients present with dementia, visual fi eld defects,
    weakness, and spasticity.
    D. Patients with herpes simplex encephalitis present with
    a subacute course consisting of personality changes, fever,
    headaches, and seizures.
    Temporal lobes are primarily affected, and the disease is
    fatal without treatment.
    E. Rabies causes personality changes, headache, dysphagia
    to even water (hydrophobia), and pharyngeal muscle
    spasm that makes patients appear to be frothing at the
    mouth.
342
Q
1080. The Triceps Refl ex best tests for what nerve root?
A. C5
B. C6
C. C7
D. C8
E. None of the above
A
  1. Answer: C

Source: Wright PD, Board Review 2004

343
Q
  1. A young adult reports that he has not been able to sleep
    for over two days and has been having strange reactions.
    These reactions are most apt to be caused by
    A. Feelings of excessive tiredness
    B. Increased levels of blood cortisol
    C. Physiologic stress in response to sleep deprivation
    D. Perceptual distortions
    E. The effects of the rebound phenomenon
A
  1. Answer: D
    Explanation:
    (Carlson, pp 259-267.)
    · The human research on sleep has demonstrated that after
    a few days of sleep deprivation people report perceptual
    distortions or, in a few cases, even hallucinations. These
    studies have documented statements such as “the fl oor
    seems wavy” or “steam seems to be rising from the fl oor,”
    indicating that sleep deprivation affects cerebral
    functioning.
    · Sleepiness can occur even without any activity and sleep
    deprivation does not appear to interfere with the ability to
    perform physical exercise. Likewise, there is no evidence of
    a physiologic stress response to sleep deprivation,
    indicated by little change in blood levels of cortisol and
    epinephrine.Sleep does appear to be necessary for the
    brain to function normally.
    · After a period of sleep deprivation a rebound
    phenomenon does occur. The individual will sleep longer
    and spend a much greater time in REM sleep, but will not
    regain the number of sleepless hours lost.
    Source: Ebert 2004
344
Q
  1. All of the following are true regarding uncontrolled
    post-operative pain except:
    A. Decreased chest wall and diaphragmatic excursion
    B. Increased myocardial oxygen consumption
    C. Increased cardiac work
    D. Decreased risk of thromboembolic complications
    E. Decreased gastrointestinal motility
A
  1. Answer: D
    Explanation:
    Ref: Crews. Chapter 14. Acute Pain Syndromes. In:
    Practical Management of Pain, 3rd Edition. Raj et al,
    Mosby, 2000, page 171.
    Source: Day MR, Board Review 2003
345
Q
1083. The preferred treatment of status epilepticus is
intravenous administration of
A. Chlorpromazine
B. Diazepam
C. Succinylcholine
D. Tranylcypromine
E. Ethosuximide
A
  1. Answer: B
    Explanation:
    Reference: Hardman, p 484.
    Intravenously administered diazepam is the drug of choice
    for treatment of status epilepticus.
    Diazepam increases the apparent affi nity of the inhibitory
    neurotransmitter GABA for binding sites on brain cell
    membranes. The effects of diazepam are short-lasting.
    Continuing therapy is usually with phenytoin.
    Other drugs suggested for use in status epilepticus are
    lorazepam and lidocaine.
    Chlorpromazine is an antipsychotic.
    Succinylcholine is a neuromuscular blocking agent.
    Tranylcypromine is an antidepressant.
    Ethosuximide is used in petit mal epilepsy.
    Source: Stern - 2004
346
Q
  1. In which of the following would you consider performing
    a spinal nerve denervation?
    A. Lumbar facet arthropathy
    B. Failed back surgery syndrome
    C. Severe spasticity and limb pain due to multiple sclerosis
    D. Intercostal neuralgia
    E. Sciatic nerve transaction
A
  1. Answer: C
    Explanation:
    (Raj, Practical Mgmt of Pain 3rd Ed. Page 802, Raj, Pain
    Medicine Review, 2nd
    Ed., page 314)
    This is not a benign procedure. Make sure you understand
    the difference between this procedure, dorsal rhizotomy,
    and dorsal ganglionectomy. In this procedure, one is
    targeting a mixed nerve. Spinal nerve more accurately
    more describes the entity referred to as a selective nerve
    root. The term, ‘selective nerve root’ should not be used
    since it lacks anatomic precision. Nonetheless, it may still
    be used in the exam.
    Hence, lesioning the spinal nerve can cause motor and
    sensory dysfunction. It should be used to treat pain related
    to spasticity due to central nervous system damage. It
    should be avoided in non-cancer conditions. Sciatic nerve
    transaction may lead to deafferentation pain and central
    sensitization. Denervation of those spinal nerves supplying
    the sciatic nerve may worsen this pain condition.
    Additionally, sciatic nerve transaction will not cause
    spasticity.
    Source: Schultz D, Board Review 2004
347
Q
  1. A 30-year-old secretary who is a single mother with two preschool children has frequent symptoms of anxiety,
    tension, headaches, and insomnia. Which of the following
    behavioral interventions could be the most effective in
    relieving her symptoms?
    A. Progressive muscle relaxation
    B. Psychoanalytic psychotherapy
    C. Hypnosis
    D. Selective biofeedback
    E. Interpersonal psychotherapy
A
  1. Answer: A
    Explanation:
    (Baum, pp 297-301.)
    · Progressive muscle relaxation, or a reasonable variation,
    can serve as a powerful therapeutic technique for treating
    generalized anxiety, insomnia, headaches, neck tension,and
    mild forms of agitated depression. It has also effectively
    been used to reduce pain.
    · Relaxation therapy is based on the premise and
    observation that muscle tension is a physiologic response
    to anxiety and stress. There is a signifi cant reduction in
    experienced anxiety if tense muscles can be relaxed.Muscle
    relaxation also can change the physiologic activation
    process.
    · Other effective methods of relaxation include systematic
    deep breathing, transcendental meditation, and yoga.
    Source: Ebert 2004
348
Q
  1. Which of the following is among the most useful
    procedures for the relief of unilateral cancer pain below
    the C5 dermatome?
    A. Midline myelotomy
    B. Lissauer tractotomy
    C. Percutaneous C1-2 radiofrequency unilateral cordotomy
    D. Post central gyrectomy
    E. Bilateral high cervical cordotomy performed at the same
    time
A
  1. Answer: C
    Explanation:
    (Raj, Pain Review, 2nd Ed., page 313)
    Lissauer tractotomy is the goal of the dorsal root entry
    zone (DREZ) procedure…but all dorsal horn lamina (I-V)
    may be affected. The DREZ lesion is classically indicated
    for central nervous system damage related pain: brachial
    plexus avulsion, stump pain, spinal cord injury pain.
    Midline or commissural myelotomy sections those
    midline fi bers just dorsal to the central canal of the spinal
    cord. The original intent was to lesion crossing
    spinothalamic neurons, which would eliminate pain, but
    preserve sensory function. However, pain relief extended
    caudally, without demonstrable caudal analgesia. This lead
    several investigators to postulate several alternate pain
    pathways. A multisynaptic short tract afferent pathway or
    an anterior tract located in between the posterior columns
    were proposed. The latter mediate pelvic and epigastric
    visceral pain. Nonetheless, myelotomy is indicated for
    bilateral pelvic and perineal pain of malignant origin.
    Post-central gyrectomy is not a primary neurosurgical
    procedure for pain relief and is used for central pain
    syndromes: thalamic or phantom pain. The post-central
    gyrus is the principal cortical area for the integration of
    sensory information.
    Bilateral high cervical cordotomy may be responsible for
    Ondine’s curse (sleep induced apnea) and is
    contraindicated unless performed in a staged fashion.
    Unilateral percutaneous cordotomy is among the most
    useful procedures for unilateral cancer pain below C5. It
    targets the spinothalamic tract. Radiofrequency energy is
    used. Electrical stimulation (sensory to obtain a feeling of
    warmth or coolness on the contralateral side and motor to obtain ipsilateral cervical muscles; ipsilateral contraction
    of muscles below the neck implies the probe is in the
    corticospinal tract) is used to identify the lesion target
    Source: Schultz D, Board Review 2004
349
Q
  1. Which of the following would not alter one’s decision to proceed with occipital neurectomy?
    A. Xrays showing spondylotic changes at C2 and C3
    B. History of brain tumor removal
    C. Positive response of occipital nerve blocks
    D. An MRI showing cerebellar descent into the spinal canal
    and a syrinx
    E. New onset posterior headache that is aggravated by
    coughing and is associated with vomiting
A
  1. Answer: A
    Explanation:
    (Raj, Pain Medicine Review, 3rd Ed., 313, Raj, Practical
    Mgmt of Pain, page 800)
    Poorly worded question. Nonetheless, expect a lot of
    questions to be worded poorly. Spondylotic changes are
    common as we age. This fi nding would support an upper
    cervicogenic etiology for occipital pain. The other choices
    would change your management strategy, specifi cally the
    presence of an Arnold-Chiari Malformation, posterior
    fossa tumors, or foramen magnum lesion.
    Source: Schultz D, Board Review 2004
350
Q
  1. Proper patient positioning for a subarachnoid alcohol
    block is:
    A. Painful side up with the patient tilted posteriorly 45°.
    B. Painful side down with the patient tilted posteriorly 45°.
    C. Painful side up with the patient tilted anteriorly 45°.
    D. Painful side down with the patient tilted anteriorly 45°.
    E. Painful side up with no tilt.
A
  1. Answer: C

Source: Day MR, Board Review 2005

351
Q
  1. The acupuncture point located between the fi rst and
    second metatarsal bones in the web is called
    A. Lieh Chuch
    B. Ho Ku
    C. Chih Tse
    D. Chien chen
    E. None of the above
A
  1. Answer: B
    Source: Raj P, Pain medicine - A comprehensive Review -
    Second Edition
352
Q
1090. An example of secondary gain is:
A. Pain behavior
B. Anxiety
C. Unconscious motivation
D. Projection
E. Work avoidance
A
  1. Answer: E

Source: Janata JW, Board Review 2005

353
Q
1091.Bilateral, compared to unilateral, surgical lumbar
sympathectomies pose the unique risk:
A. erectile dysfunction in men
B. ejaculatory dysfunction in men
C. genitofemoral neuralgia
D. spinal cord infarction
E. damage to the kidney or ureter
A
  1. Answer: B
    Explanation:
    (Raj, Practical Mgmt of Pain, 3rd Ed., page 803)
    Bilateral lumbar sympathectomies pose an undue risk of
    ejaculatory dysfunction. Lesions of S2, S3, and S4
    parasympathetics may cause erectile dysfunction.
    Genitofemoral neuralgia may be a complication of lumbar
    sympathectomy either unilaterally or bilaterally. A leftsided
    lumbar sympathectomy may inadvertently injury
    branches of the aorta (including the artery of
    Adamkiwiecz). Damage to the kidney or ureter can occur
    on either side
    Source: Schultz D, Board Review 2004
354
Q
1092. You diagnosed a patient with torn epidural catheter of 1.2 cm. Your choice of treatment is:
A. Do Nothing
B. MRI
C. Monthly Neurological Exam
D. Surgical Exploration
E. Antibiotics
A
  1. Answer: A
355
Q
  1. In assessing the characteristic pain patient, the clinician
    must appreciate that they are often:
    A. Emotionally debilitated and of low intellectual measure
    B. Severely agitated and aggressive
    C. Disabled, depressed and dependent
    D. All of the above
    E. None of the above
A
  1. Answer: C

Source: Giordano J, Board Review 2003

356
Q
1094. Which of the following is the best indication for the
intraventricular infusion of morphine?
A. Migraine Headaches
B. Meningitis
C. Failed back surgery syndrome
D. Oropharyngeal carcinoma
E. Diabetic peripheral neuropathy
A
  1. Answer: D
    Explanation:
    (Raj, Pain Review 2nd Ed. Page 310, Raj, Practical Mgmt
    of Pain 3rd Ed., pg. 795)
    The main indications for intraventricular infusion of
    opioids are (1) head and neck cancers and (2) failure of
    relief with intraspinal opioids in a patient with limited life
    expectancy,
357
Q
1095. Appropriate workup for classic migraine includes:
A. Cranial MRI
B. Head CT
C. Fundoscopic examination
D. All of the above
E. None of the above
A
  1. Answer: C

Source: Wirght PD, Board Review 2004

358
Q
  1. Maxillary nerve block is indicated for the diagnosis,
    treatment, or management of all of the following except:
    A. Temporo-mandibular joint problems
    B. Atypical facial pain
    C. Surgical anesthesia for removal of the upper incisors
    D. Trigeminal neuralgia
    E. Infiltrating tumor of the maxillary sinus
A
  1. Answer: A
    Explanation:
    (Raj, Pain Review, 2nd Ed., page 226)
    The temporo-mandibular joint is innervated by the
    auriculotemporal nerve a branch of V3. All the others are
    indications for a maxillary nerve block
    Source: Shah RV, Board Review 2003
359
Q
1097. A 49-year-old woman walks by moving her right leg
forward by abduction and circumduction. Choose correct
description of gait:
A. Ataxic gait
B. Parkinsonian gait
C. Spastic hemiplegic gait
D. Steppage gait
E. Scissor gait
A
  1. Answer: C
    Explanation:
    A. Ataxic gait is often characterized by clumsiness; when
    steps are taken, the advancing foot is lifted high.The foot is
    then brought down in a slapping or stamping manner.
    B. Parkinsonian gait is noted for the forward stoop of the
    head and shoulders, with arms slightly abducted and
    forearms partially fl exed; there is decreased arm swing as
    the feet shuffle.
    C. Spastic hemiplegic gait is the result of spasticity of the
    involved limb. The limb is moved forward by abduction
    and circumduction.
    D. Steppage gait occurs with footdrop (paralysis of the
    peroneal nerve); the affected foot is raised higher than
    normal to prevent dragging of the toe. Bilateral footdrop
    results in a gait resembling that of a high-stepping horse.
    E. Spastic diplegia gait or scissor gait occurs with
    extrapyramidal disorders. The patient uses short steps and
    drags the foot; the legs are extended and stiff and cross on
    each other.
    Source: Seidel, Sle, pp 791-792.
360
Q
  1. A patient loses consciousness in your pain clinic after
    a procedure. Ventricular fi brillation is apparent on
    the cardiac monitor. The patient’s airway is being well
    managed. An IV line is in place. A defi brillator is at the
    bedside. A precordial thump has been administered and
    chest compressions are ongoing. The most appropriate
    next intervention is:
    A. Intravenous vasopressin
    B. Intravenous epinephrine
    C. Intravenous lidocaine
    D. Intravenous amiodarone
    E. Stop chest compressions and cardiovert with 200 joules
A
  1. Answer: E
    Explanation:
    Reference:
    ACLS Provider Manual, 2000 Edition, American Heart
    Association
    Here is a pneumonic for cardiac resuscitation of
    ventricular fi brillation from the ACLS manual:
    Please Shock-Shock-Shock, EVerybody Shock, And Let’s
    Make Patients Better
    Source: Schultz D, Board Review 2004
361
Q
  1. All of the following are contra-indications to ultrasound except:
    A. An epiphyseal injury of a young athlete’s elbow.
    B. An acute muscle tear.
    C. A diabetic with peripheral neuropathy with a painful
    great toe.
    D. A contracture of a hip joint.
    E. A pregnant female with an abdominal strain.
A
  1. Answer: D

Source: Malanga G, Board Review 2003

362
Q
  1. A 44-year-old man presents with facial asymmetry. On
    physical examination, touching the cornea of either eye
    with a cotton swab results in blinking of only the left eye.
    The patient states that he feels the cotton swab touch
    in both eyes. Which of the following is the most likely
    diagnosis?
    A. Left trigeminal palsy
    B. Right trigeminal palsy
    C. Right facial nerve palsy
    D. Left facial nerve palsy
    E. Left oculomotor nerve palsy
A
  1. Answer: C
    Explanation:
    (Seidel, 51e, p 785.)
    The corneal refl ex is normal when touching the cornea
    (trigeminal nerve provides sensation) causes bilateral eye
    closure (facial nerve provides motor).
    This refl ex will not occur on the side of a facial nerve
    paralysis.
363
Q
1101. Pars Interarticularis defects are effectively diagnosed
by:
A. Ultrasound
B. CT
C. Plain radiographs
D. Upper GI series
E. MR
A
  1. Answer: B
    Explanation:
    Although plain fi lms may reveal pars defects, CT is the
    imaging modality which best confi rms and characterizes
    the abnormality
    Source: Bieneman B, Board Review 2005
364
Q
  1. A 45-year old, slim, heavy smoker, presents with low back pain that radiates into right buttock and to the leg slightly below
    the knee. The pain started somewhat suddenly for this episode but he suffered with chronic pain on and off for 20 years.
    Most recently, the pain was associated after driving continuously for 8 hours in a pickup truck. The physical examination
    was grossly normal except for tenderness in the lumbar spine and exacerbation of pain with hyperextension. Patient
    brought plain x-ray picture fi lms with him and refuses to undergo any further investigations. Based on the plain x-ray
    films shown below (pg.180), your radiologic diagnosis is as follows:
    A. Grade I spondylolisthesis and spondylolysis of L5 on S1
    B. Grade I spondylolisthesis of L5 on S1 with bilateral pars defects
    C. Grade II spondylolysis of L5 on S1
    D. Grade III spondylolisthesis of L5 on S1 with bilateral pars defects
    E. Extensive facet joint arthritis with spinal stenosis
A
  1. Answer: E
    Explanation:
    (Tierney, 42/e, pp 797-798.)
    The patient most likely has carpal tunnel syndrome (CTS),
    which is compression of the median nerve by the
    transverse volar ligament of the wrist. Patients complain
    of pain and paresthesias of the ‘hand and weakness and
    atrophy of the thenar muscles. The Tinel sign (tapping the
    median nerve at the wrist) and Phalen sign (forced wrist
    fl exion) intensify the symptoms. Risk factors for CTS
    include pregnancy, diabetes mellitus, hypothyroidism,
    rheumatoid arthritis, amyloid infi ltration as seen in
    patients with multiple myeloma, acromegaly, and
    repetitive trauma. Ulnar nerve paralysis causes a claw hand
    deformity Radial nerve palsy causes wristdrop. Erb-
    Duchenne palsy (C5-C6) causes weakness of the shoulder
    and elbow and results in the waiter’s tip position (arm
    dangles at the side with palm in a backward position with
    fi ngers fl exed). Klumpke-Dejerine palsy (C8-Tl) is a triad
    of claw hand deformity, absent triceps refl ex, and Horner
    syndrome. Patients with cervical radiculopathy (C6 or C7
    root) complain of neck pain that radiates to the arm
    (radicular pain), dermatomal sensory loss, and decreased
    refl exes.
365
Q
  1. A 35-year old man presented with constant low back pain that radiated to the left or right upper buttock region with
    occasional radiation to the thigh and calf posteriorly with tingling sensation in the left heel. The symptoms started
    approximately a year ago when he lifted a heavy box which caused the gradual onset of low back pain at the time with
    increasing intensity in a week. His motor examination was grossly within normal limits. However, he had a positive left
    straight leg raising at 50°. There was decreased sensation to pin prick on the lateral side of the foot on the left side. The
    following MRI shows (pg.180):
    A. L4/5 disc herniation
    B. L5/S1 disc herniation
    C. Large osteophyte pressing on L5 nerve root
    D. Large osteophyte pressing on L4 nerve root
    E. Facet joint arthritis causing spinal stenosis
A
  1. Answer: B

Source: Giordano J, Board Review 2003

366
Q
  1. A 38-year old man presented with low back pain of 3 months’ duration which started following a lifting incident. Pain was present in the low back with radiation into lower extremity associated with signifi cant stiffness. He failed to respond to non-steroidal anti-infl ammatory medication, an aggressive exercise program, intramuscular and oral corticosteroids. Patient was referred to you for a transforaminal epidural steroid injection. You will perform the transforaminal epidural steroid injection in this patient at the following level (pg181):
    A. Right L5 transforaminal epidural steroid injection
    B. Left L5 transforaminal epidural steroid injection
    C. Right L4 transforaminal epidural steroid injection
    D. Left L4 transforaminal epidural steroid injection
    E. Left S1 transforaminal epidural steroid injection
A
  1. Answer: A

Source: Wirght PD, Board Review 2004

367
Q
  1. A 44-year old white male presents with a history of spontaneous onset of low back pain with radiation into lower extremity
    associated with numbness, tingling, and a positive straight leg raising test at 60°. A surgeon recommended discectomy, however,
    the patient refused and wanted to try conservative management with interventional techniques. Non-steroidals, oral and
    intramuscular steroids, and physical therapy failed to provide him any signifi cant relief. Your diagnosis in this patient based on
    the following MRI (pg 181)is:
    A. Right L3/4 disc herniation with pressure on L4 nerve root
    B. Right L4/5 disc herniation with pressure on L5 nerve root
    C. Right L5/S1 disc herniation with pressure on L5 nerve root
    D. Left L3/4 disc herniation with pressure on L4 nerve root
    E. Left L4/5 disc herniation with pressure on L5 nerve root
A
  1. Answer: E
    Explanation:
    (Raj, Pain Medicine Review, 2nd Ed., page 236-238)
    An interscalene block is performed with the patient supine
    and their head rotated away from the operator. The C6
    level is palpated by identifying the cricoid cartilage. The
    non-dominant hand’s index fi nger is used to gently palpate
    the posterior border of the sternocleidomastoid muscle.
    The fi nger is moved further posteriorly to identify the
    groove between the anterior and middle scalenii. The
    patient may be asked to deep breathly to accentuate this
    groove. The ext. jugular vein may run across this location.
    A 22g needle is advanced medial, posterior, and slightly
    caudad (perpendicular to skin). Paresthesias should be
    elicited in the shoulder, or hand is obtained. 40-50 cc of
    local may be instilled incrementally.
    The block is performed at the level of the cervical roots
    and is most likely to miss the C8 spinal nerve. Thus, one
    may get incomplete analgesia of the ulnar aspect of the
    hand. All the others are covered by the C5, C6 areas
    primarily.
    Source: Shah RV, Board Review 2003
368
Q

1106.Ganglion impar neurolytic block after successful
diagnostic block for rectal pain may be performed by
using:
1. 25 ml of 50% alcohol
2. 10 ml of absolute alcohol
3. 4 ml of 25% phenol in glycerol
4. 4 ml of 6% phenol

A
  1. Answer: D (4 Only)

Source: Racz G. Board Review 2003

369
Q
  1. Anterior Spinal Artery Syndrome usually results in:
  2. Unilateral impairment of position sense
  3. Normal motor function below the lesion
  4. Brain stem stroke
  5. Bilateral impairment of pain and temperature
A
  1. Answer: D (4 Only)

Source: Wirght PD, Board Review 2004

370
Q
1108.During EMG testing, abnormal electrical activity
includes the following:
1. Fibrillation potentials
2. Fasciculation potentials
3. Myokymic discharges
4. Miniature endplate potentials
A
  1. Answer: A ( 1, 2, & 3)

Source: Wirght PD, Board Review 2004

371
Q
  1. A 42 year-old man develops excruciating pain extending
    from his buttocks to the dorsum of his foot. Which
    of the following would confi rm the presence of a disc
    protrusion?
  2. The Lasegue’s sign
  3. Thomas test
  4. Milgram Test
  5. Fabere Test
A
  1. Answer: B (1 & 3)
    Explanation:
    (Raj, Practical Mgmt of Pain, 3rd Ed., page 350 and 358)
  2. Straight Leg Raising Test: Lasègue’s Sign: This
    maneuver can test for sciatic irritation (pain radiating to
    the ankle of the tested leg),but sciatic nerve irritation must
    be differentiated from hamstring tightness (pain
    descending the posterior thigh only). The patient lies
    supine, with the examiner stabilizing the heel in one palm
    and helping to maintain knee extension with the other.
    Elevation is performed to 70 to 90 degrees at the hip, and
    radiating pain to the ankle confi rms the test.
  3. Thomas Test.The Thomas test evaluates fl exion
    contracture of the hip. The patient lies supine with the
    pelvis level, allowing a T to form between the vertebral
    spine and the pelvic brim. The hip is passively fl exed with
    the examiner’s other hand beneath the small of the back,
    feeling the point where the lumbar curve is lost. The thigh
    is then placed against the abdominal wall. The other leg is
    fl exed in similar fashion and allowed to descend to the
    examination table. If compensation is attempted by
    arching
  4. MILGRAM TEST: The patient lies supine and attempts
    to hold the heels about 2 inches off the table for 30
    seconds.
    Intrathecal pressure is elevated, and if a mass lesion or
    herniated disk is present, the patient lowers the affected
    side to the table.
  5. Fabere (Flexion-Abduction-External Rotation [-
    Extension]) Test: This maneuver tests for sacroiliac or hip
    pathology. The patient is supine with the knee and hip
    fl exed and the heel on the opposite knee, allowing the
    femur to lower to the examination table. This position
    results in hip abduction and external rotation. Groin pain suggests hip pathology, and sacroiliac pain suggests a
    problem with this joint
    Source: Shah RV, Board Review 2004
372
Q
  1. A positive Froment’s sign indicates
  2. Weakness of the adductor pollicis
  3. Weakness of the flexor pollicis brevis
  4. Weakness of the first dorsal interosseous
  5. Weakness of the hypothenar muscles
A
  1. Answer: A (1,2, & 3)
    Explanation:
    Froment’s sign is a sign of ulnar nerve palsy. It refl ects a
    reduction in the ability to pinch between the thumb and
    adjacent digits. Specifi cally, a patient cannot pinch a piece
    of paper between the ulnar side of the thumb and radial
    side of the index fi nger. The patient compensates by using
    median nerve muscles: FPL, FDS, Index FD. Muscles in
    choices A,B,C are involved
    Although the hypothenar muscles are affected by the ulnar
    nerve palsy, they are not part of Froment’s sign
    Source: Shah RV, Board Review 2004
373
Q
  1. Which of the following are late responses?
  2. H-refl ex
  3. A-wave
  4. F-wave
  5. M-wave
A
  1. Answer: A ( 1, 2, & 3)

Source: Wirght PD, Board Review 2004

374
Q
  1. Which statements are false regarding spinal injection
    and bleeding complications:
  2. Aspirin and other NSAIDS, in and of themselves, do
    not signifi cantly increase the risk of epidural hematoma
    and need not be discontinued prior to spinal injection.
  3. Spinal injection may be safely performed on a patient
    who has been off Coumadin for 4 days and has an INR
    of 1.4
  4. Cox 2 inhibitors such as Celebrex and Vioxx do not inhibit
    platelets and do not affect coagulation.
  5. Bleeding time predicts hemostatic compromise in patients
    taking anti-platelet drugs.
A
  1. Answer: D (4 Only)
    Explanation:
    Reference:
    Horlocker, et. Al.
    Regional Anesthesia in the Anticoagulated Patient:
    Defi ning the Risks (The Second ASRA Consensus
    Conference on Neuraxial Anesthesia and Anticoagulation)
    At the 1998 Consensus Conference on Neuraxial
    Anesthesia and Anticoagulation, it was concluded that
    NSAIDs, in and of themselves, did not appear to present
    signifi cant risk to patients for developing spinal-epidural
    hematomas. There is no recommendation to discontinue
    aspirin or other NSAIDS prior to spinal injection in the
    ASRA guidelines.
    Normal clotting requires 40% or greater clotting factor
    activity. An INR value of 1.5 indicates approximately 40%
    activity of clotting factors and essentially normal ability to
    clot. This value has been derived from studies correlating
    hemostasis with clotting factor activity levels. INR below
    1.5 is considered safe for spinal injection.
    Cyclooxygenase (COX) exists in 2 forms. COX-1
    regulates constitutive mechanisms, while COX-2 mediates
    pain and infl ammation. NSAIDs inhibit platelet COX 1
    and COX 2 and prevent the synthesis of thromboxane A2.
    NSAIDS inhibit platelet function. Celecoxib (Celebrex)
    and Rofecoxib (Vioxx) are anti-infl ammatory agents that
    primarily inhibit COX-2,an inducible enzyme which is not
    expressed in platelets. Therefore COX 2 inhibitors do not
    cause platelet dysfunction. Platelets from patients who
    have been taking COX 2 inhibitors have normal platelet
    adherence to subendothelium and normal primary
    hemostatic plug formation. After single and multidosing
    regimens, there have not been fi ndings of signifi cant
    disruption of platelet aggregation, nor is there a history of
    undesirable bleeding events.
    It has been suggested that the Ivy bleeding time is the most
    reliable predictor of abnormal bleeding in patients
    receiving antiplatelet drugs. However, there is no evidence
    to suggest that a bleeding time can predict hemostatic
    compromise and this test is not recommended to
    determine safety of spinal injection in the setting of
    platelet inhibition.
    Source: Schultz D, Board Review 2004
375
Q
1113. In assessing a possible C8 radiculopathy, the following
muscle(s) would be benefi cial:
1. Triceps
2. Flexor carpi ulnaris
3. Abductor policis brevis
4. Trapezius
A
  1. Answer: A ( 1, 2, & 3)

Source: Wirght PD, Board Review 2004

376
Q
  1. Complication of C6 transverse process stellate ganglion
    block include:
  2. Seizure from injection into vertebral artery
  3. Total spinal with subarachnoid injection
  4. Spinal cord trauma
  5. Nerve injury
A
  1. Answer: E (All)

Source: Racz G. Board Review 2003

377
Q
  1. In the rehabilitation of a chronic low back pain patient,
    which of the following has been scientifi cally validated as
    effective treatment?
  2. A long course of hot packs, ultrasound and electrical
    stimulation.
  3. Manipulation and other manual treatments.
  4. Daily traction combined with cryotherapy.
  5. An active exercise program.
A
  1. Answer: D (4 Only)

Source: Malanga G, Board Review 2003

378
Q
  1. A 70 year old woman with spinal stenosis and lumbar
    radiculopathy is hospitalized for TIA episodes. She is
    placed on intravenous low molecular weight heparin
    (LMWH) because she is at high risk for stroke. You are
    asked by the neurologist to perform a lumbar epidural
    injection prior to hospital discharge to treat her
    radiculopathy. The patient stops heparin and clotting
    returns to normal as measured by pre-procedure aPTT.
    You then perform an atraumatic L3-4 epidural steroid
    injection. She is re-heparinized 2 hours after injection.
    24 hours later she begins complaining of increased back
    pain and increasing numbness and weakness in her lower
    extremities. You are concerned about spinal hematoma.
    The following statement(s) are true:
  2. Epidural hematoma is unlikely because 24 hours have
    passed since injection.
  3. Surgical decompression of spinal hematoma has good
    outcome if performed within 24 hours of onset of
    symptoms.
  4. Severe back pain is the most common presenting complaint
    of patients with epidural hematoma.
  5. The best test to order fi rst is an emergency MRI scan of
    the spine.
A
  1. Answer: D (4 Only)
    Explanation:
    Reference:
    Horlocker, et. Al.
    Regional Anesthesia in the Anticoagulated Patient:
    Defi ning the Risks (The Second ASRA Consensus
    Conference on Neuraxial Anesthesia and Anticoagulation)
    Spinal hematoma, defi ned as symptomatic bleeding within
    the spinal neuraxis, is a rare and potentially catastrophic
    complication of spinal or epidural anesthesia. The actual
    incidence of neurologic dysfunction resulting from
    hemorrhagic complications associated with central neural
    block is unknown. In an extensive review of the literature,
    Tryba identifi ed 13 cases of spinal hematoma following
    850,000 epidural anesthetics and 7 cases among 650,000
    spinal techniques. Based on these observations, the
    calculated incidence is approximated to be less than 1 in
    150,000 epidurals and less than 1 in 220,000 spinal
    anesthetics. Hemorrhage into the spinal canal most
    commonly occurs in the epidural space, most likely
    because of the prominent epidural venous plexus.
    Between 1993 and 1998, there were 45 cases of spinal
    hematoma associated with LMWH, 40 of which involved a
    neuraxial anesthetic.Severe radicular back pain was not the
    presenting symptom; most patients complained of new
    onset numbness, weakness, or bowel and bladder
    dysfunction. Neurologic compromise presented as
    progression of sensory or motor block (68% of patients)
    or bowel/bladder dysfunction. Approximately half of
    patients reported neurological defi cits 12 hours or more
    following spinal procedure. Median time interval between
    initiation of LMWH therapy and neurologic dysfunction
    was 3 days, while median time to onset of symptoms and
    laminectomy was over 24 hours. Only 38% of patients had
    partial or good neurological recovery and spinal cord
    ischemia tended to be reversible in patients who
    underwent laminectomy within 8 hours of onset of
    neurological dysfunction. Early diagnosis by MRI scanning
    is therefore of paramount importance.
    Source: Schultz D, Board Review 2004
379
Q
  1. True entrapment of the ulnar nerve at as described
    by Guyon at Guyon’s canal could lead which of the
    following?
  2. Loss of sensation on the dorsum of the 5th digit
  3. Wrist pain that radiates to the forearm
  4. Paralysis of the hypothenar
  5. A positive Froment’s sign
A
  1. Answer: C (2 & 4)
    Explanation:
    (Dawson, Entrapment Neuropathies,
    The American Academy of Physical Medicine and
    Rehabilitation–
    http://www.aapmr.org/education/archive/emg0102e.htm.)
    Okay, a Trick Question. Entrapment at the proximal aspect
    of Guyon’s canal before the deep motor branch sends a
    hypothenar motor branch could cause weakness of the
    hypothenar and intrinsics. However, even, then
    hypothenars are often mildly involved and can be
    overlooked. Froment’s sign occurs with ulnar palsies as
    described earlier
    Source: Shah RV, Board Review 2004
380
Q
1118. Sleep Studies (polysomnography) involve measurements
of following parameters:
1. O2 saturation
2. Heart Rate
3. Eye movements
4. Respiratory movement
A
  1. Answer: E (All)

Source: Wirght PD, Board Review 2004

381
Q
  1. Which of the following statements is most accurate
    concerning our current understanding of medication
    overuse headache (MOH), formerly referred to as
    analgesic rebound headache syndrome? Pick one of the
    following:
  2. Medication overuse headache is likely to occur over
    time when short acting pain relieving medications or
    compounds are, on average, used more frequently than
    two days a week to relieve headache.
  3. Long acting preparations of opioid analgesics do not
    generally result in MOH even when used daily over
    years in the control of headache or other pain.
  4. Prophylaxis of headache will fail in the face of excessive
    (too frequent) use of short acting abortive medication
    over time no matter the type, dose or combination of
    prophylactic medication used.
  5. The majority of those who overuse short acting ‘over the
    counter’ and / or controlled substances for the management
    of headaches tend to have addictive personalities
A
  1. Answer: A

Source: Goodwin J, Board Review 2005

382
Q
1120. Mechanism of action of TENS pain relief is thought to
be:
1. Stimulates corticospinal system
2. Stimulates reticulospinal system
3. Inhibits large A fi ber activity
4. Inhibits C fiber activity
A
  1. Answer: D (4 Only)
    Explanation:
    TENS is thought to involve the gate control theory;
    activation of large fi bers presumably inhibits C fi ber
    activity.
383
Q
  1. Spinal stenosis rehabilitation includes the following:
  2. A lumbar fl exion program
  3. Modifi ed abdominal strengthening
  4. Bicycling
  5. Downhill walking
A
  1. Answer: A (1, 2, & 3)
    Explanation:
    Therapeutic exercises are benefi cial and should include a
    lumbar fl exion program, modifi ed abdominal
    strengthening, trunk and lower extremity fl exibility,
    bicycling, and uphill treadmill walking.
384
Q
  1. A 33 year-old female pricks her index fi nger and
    progressively develops pain involving her upper limb.
    Which of the following is required for a diagnosis of
    complex regional pain syndrome?
  2. diffuse pain in the upper limb
  3. pain that becomes worse with light touch or exposure
    to cold
  4. swelling of the hand
  5. tremor of the hand
A
  1. Answer: A (1,2, & 3)
    Explanation:
    (Shah, et. al. Recurrence and spread of CRPS, accepted to
    American Journal of Orthopedics)
    Complex regional pain syndrome (CRPS) describes a
    constellation of sensory, motor, autonomic, and trophic
    disturbances, with spontaneous pain and hyperalgesia
    being the most persistent signs (Birklein). The term,
    ‘CRPS’, was introduced by a consensus group in 1996, to
    describe a variety of painful conditions that follow injury
    (Stanton-Hicks). They are characterized by spontaneous
    pain or hyperalgesia, a distal regional predominance,
    variable progression over time, impairment of motor
    function, and a magnitude and duration exceeding the
    expected clinical course of the inciting event; temperature,
    skin color, edematous, and sudomotor abnormality are or
    have been present (Stanton-Hicks 1998). The two subtypes
    include all the foregoing features but either exclude
    (CRPS I) or include (CRPS II) a peripheral nerve injury
    (Stanton-Hicks).
    Motor dysfunction is not required for diagnosis, but is
    often present in most patients with CRPS
    Source: Shah RV, Board Review 2004
385
Q
  1. Which of the following are included in a complete
    electrodiagnostic evaluation?
  2. Electromyography and late response studies
  3. Peripheral nerve conduction studies of motor and sensory
    nerves
  4. Somatosensory evoked potentials
  5. Muscle biopsy
A
  1. Answer: A (1, 2 & 3)
    Explanation:
    Reference: Bonica, p 629.
    The electromyogram (EMG), peripheral nerve conduction
    studies (NCSs), late response studies, and somatosensory
    evoked potentials (SEPs) help to characterize the nature
    and location of the abnormality being studied.
    Determination of the cause of the abnormality can occur
    only after integration of the information obtained fromthe
    physical examination, history, and electrodiagnostic and
    radiologic studies.
    Muscle biopsy is not a component of electrodiagnostic
    evaluation.
    Source: Kahn and Desio
386
Q
  1. Operant interventions are specifi c and targeted. These are all of the below:
  2. Patient sets goals and has a predefi ned award for attainment
    of goal.
  3. Goals are increasingly more diffi cult in an attempt to
    maximize function.
  4. Medical staff and family members are asked to acknowledge
    adaptive behaviors by the patient.
  5. Medical staff and family members are instructed to support
    the patient’s pain behavior
A
  1. Answer: A (1, 2, & 3)

Source: Raj, Pain Review 2nd Edition

387
Q
  1. A patient presents with an acute onset of upper extremity
    pain. On examination, there was decrease in sensation in
    the lateral arm. The patient may be suffering with the
    following condition(s):
  2. C5/6 disc herniation
  3. C4/5 disc herniation
  4. C6 radiculopathy
  5. C5 radiculopathy
A
  1. Answer: C (2 & 4)
    Explanation:
    The C5 neurological level supplies sensation to the lateral
    arm, from the summit of the shoulder to the elbow. The
    purest patch of axillary nerve sensation lies over the lateral
    portion of the deltoid muscle. This localized sensory area
    within the C5 dermatome is useful for indicating specifi c
    trauma to the axillary nerve, as well as general trauma to
    the C5 nerve root.
    Source: Hoppenfeld S. Orthopaedic Neurology. A
    Diagnostic Guide to Neurologic Levels.
    Philadelphia,LWW, 1997.
388
Q
1126. The most common cause of acquired fl atfoot in the adult
population is one of the following:
1. Tarsal tunnel syndrome
2. Posterior tibialis dysfunction
3. Plantar fasciitis
4. Spring ligament failure
A
  1. Answer: B (1 & 3)

Source: Sizer et al - Pain Practice - March & June 2004

389
Q
  1. Coccygodynia has been reported to be treated with:
  2. cryoneurolysis of S5
  3. radiofrequency thermocoagulation of S5
  4. sacral nerve root stimulation – transsacral
  5. retrograde dual electrode placement to S3
A
  1. Answer: E (All)

Source: Racz G. Board Review 2003

390
Q
  1. Which of the following treatments is relevant to
    managing cervical whiplash pain?
  2. Prolonged immobilization of the neck
  3. Non-steroidal anti-infl ammatory drugs
  4. Benzodiazepines
  5. Cervical medial branch radiofrequency neurotomy
A
  1. Answer: C (2 & 4)
    Explanation:
    Short-term immobilization of the neck maybe appropriate
    to provide relief. However, prolonged immobilization
    leads to weakening of the cervical spine musculature. Nonsteroidal
    anti-infl ammatory medications provide analgesia
    and reduce infl ammation may be benefi cial in the shortterm.
    Benzodiazepines are not analgesics despite their
    purported effi cacy in reducing muscle spasm. Rather,
    biofeedback and psychological counseling may be more
    appropriate. Other less addicting and sedating ‘muscle
    relaxants’ maybe more appropriate, e.g., baclofen or
    tizanidine. Cervical medial branch neurotomy in those
    selected with placebo controlled diagnostic blocks may benefi t patients with whiplash.
    Source: Shah RV: 2003 (Bonica, 3rd Ed., page 1010)
391
Q
  1. Which of the following percutaneous procedures
    compares favorably to re-operation in the management
    of failed back surgery syndrome?
  2. Spinal cord stimulation
  3. Peripheral nerve stimulation
  4. Radiofrequency denervation of the lumbar facet joints
  5. Translaminar epidural steroid injections
A
  1. Answer: B (1 & 3)
    Explanation:
    The term failed back surgery syndrome refers to persistent
    or recurrent chronic pain after one or more surgical
    procedures on the lumbosacral spine. Management is
    mired in controversy.
    However, radiofrequency denervation of the lumbar facets
    compares favorably to re-operation in long-term followup.
    Spinal cord stimulation also compares favorably to reoperation.
    Although SCS is best described for radicular
    pain, it may also be useful for axial pain. Peripheral nerve
    stimulators are usually reserved for patients with a well
    defi ned single peripheral nerve injury and complex
    regional pain syndrome. Placement must be proximal to
    the injured nerve. In radiculopathy, this would require
    placement in a retrograde approach parallel to the
    descending root, i.e., in the lateral recess and out towards
    the foramen. This technique, although described, is not
    widely practiced. Translaminar epidural steroids have been
    mired in controversy in patients without previous back
    operations. Only a few studies have demonstrated short
    term benefi t in failed back surgery syndrome and they too,
    probably won’t escape the attack of the evidence-based
    axe.
    Source: Shah RV: 2003(Bonica, 3rd Ed., pages 1544-1547)
392
Q
  1. Single fi ber EMG is most useful in evaluating:
  2. Carpal tunnel syndrome
  3. Multiple sclerosis
  4. Charcot Marie Tooth Disease
  5. Myasthenia gravis
A
  1. Answer: D (4 Only)

Source: Wirght PD, Board Review 2004

393
Q
  1. Which of the following is true about cerebrospinal
    fl uid?
  2. CSF is reabsorbed by the choroidal plexi in the ventricles
  3. CSF passes from the lateral to the 3rd ventricle via the
    foramen of Lushcka
  4. CSF is formed by arachnoid villi
  5. The total volume of CSF is 150 ml., of which 25-30 cc are
    in the spinal subarachnoid space
A
  1. Answer: D (4 only)
    Explanation:
    (Raj, Practical Management of Pain, 3rd Ed., page 632)
    Cerebrospinal fl uid (CSF) is a clear, colorless ultrafi ltrate
    of blood formed by the choroid plexuses in the ventricles
    of the brain. CSF passes through the interventricular
    foramen of Monro into the third ventricle, then through
    the cerebral aqueduct to the fourth ventricle. It exits the
    fourth ventricle by way of the lateral and median foramina
    of Luschka and Magendie to reach the subarachnoid space.
    CSF is then absorbed by arachnoid villi that project from
    the subarachnoid space. The total volume of CSF is about
    150ml, with about 25 to 35ml contained within the spinal
    subarachnoid space.
    Source: Shah RV, Board Review 2005
394
Q
  1. The major strength of the McGill Pain Questionnaire
    (MPQ) is that it is organized as a list of words, which
    are rated on a common intensity scale. It is possible to
    compare diagnosis and treatment with various pain
    syndromes by calculating the score obtained by the
    patients’ responses. All of the following are part of the
    evaluation
  2. The number of words chosen
  3. The total score based on each subclass intensity scale
  4. Rating of the common intensity scale
  5. Rating of the patient’s depression scale
A
  1. Answer: A (1,2, & 3)

Source: Raj, Pain Review 2nd Edition

395
Q
  1. The McGill Pain Questionnaire is designed to measure
    which of the following components of pain?
  2. Societal
  3. Sensory
  4. Quantitative
  5. Evaluative
A
  1. Answer: C (2 & 4)
    Source: Raj P, Pain medicine - A comprehensive Review -
    Second Edition
396
Q
1134. Which of the following symptoms are the earliest
indication of lithium intoxication?
1. Impaired consciousness
2. Myoclonus
3. Seizures
4. Coarse tremor
A
  1. Answer: E
    Explanation:
    The early signs and symptoms of lithium toxicity include
    coarse tremor, dysarthria, and ataxia; the later signs and
    symptoms include impaired consciousness, muscular
    fasciculations, myoclonus, seizures, and coma. The higher
    the lithium levels (and the longer they have been elevated),
    the worse the symptoms of lithium toxicity.
    Source: Laxmaiah Manchikanti, MD
397
Q
  1. Factitious disorder is differentiated from malingering
    by which of the following characteristics?
  2. The production of physical signs is under voluntary
    control
  3. The presence of a serious organic disorder as a comorbid
    factor
  4. The primary motivation of the patient is to assume the
    sick role.
  5. The absence of secondary gain
A
  1. Answer: D
    Explanation:
    1.In factitious disorder, the patient intentionally produces
    physical or psychological signs or symptoms that are
    under voluntary control and are not explained by any
    other underlying physical or mental disorder.
    2.There is no serious organic disorder.
    3.The primary motivation of the behavior is to assume the
    sick role. In factitious disorder however, there is no
    secondary gain such as economic benefi t or avoidance of
    legal responsibilities. In malingering, the patient has an
    obvious recognizable secondary gain in producing their
    signs and symptoms such as avoiding work or
    prosecution, or obtaining fi nancial gain.
    4.Absence of secondary gain is the main feature that
    differentiates factitious disorder from malingering.
    Source: Laxmaiah Manchikanti, MD
398
Q
  1. The following statements are true regarding
    contraindications and side effects for TENS:
  2. TENS should not be used in patients with cardiac pacemakers
  3. TENS should not be used in the vicinity of peripheral
    arteries
  4. TENS should not be used in the anterolateral neck
  5. TENS should not be used for more than one hour at a
    time
A
  1. Answer: B (1 & 3)
    Explanation:
    Reference:
    Bonica’s Management of Pain, Third Edition, Chapter 98,
    Transcutaneous Electrical Nerve Stimulation
  2. TENS should probably be avoided in patients with
    cardiac pacemakers because of the risk of electrical
    interference with pacemaker function.
    2..TENS can be used in the vicinity of other arteries.
  3. TENS electrodes should not be placed over the
    anterolateral neck because the carotid sinus may be
    stimulated causing bradycardia, hypotension and syncope.
  4. It is safe to use TENS for many hours.
    The most common side effect is skin irritation at the site
    of the patches.
    Source: Schultz D, Board Review 2004
399
Q
1137. Pain originating from which of the following viscera can
be treated with a celiac plexus block?
1. Pancreas
2. Gall bladder
3. Ascending colon
4. Sigmoid colcon
A
  1. Answer: A (1, 2 & 3)

Source: Day MR, Board Review 2005

400
Q
  1. Which of the following are true about deep brain
    stimulation?
  2. Good results are obtained with Dejerine-Roussy syndrome
  3. Short term pain relief exceeds 60%
  4. Periaqueductal gray mediated anti-nociception does not
    depend on non-opioid analgesic systems
  5. Poor results are obtained with complete spinal cord
    injury pain
A
  1. Answer: C (2 & 4)
    Explanation:
    (Raj, Pain Review 2nd Ed., page 311, Bonica 3rd Ed., pages
    130-2)
    Poor results are obtained with central thalamic and complete spinal cord injury pain. Peri-aqueductal grey
    mediated analgesia depends on both opioids and nonopioid
    systems. Short-term pain relief is 61-80%, but long
    term relief drops to 50-63%.
    Source: Schultz D, Board Review 2004
401
Q
  1. Regarding temporomandibular Disorders (TMD), which
    of the following statements have empirical support?
  2. The male to female ratio of symptomatic TMD is 1:2
  3. MRI is the imaging study of choice because effusion into
    a temporomandibular synovial joint (TMJ) correlates
    well with pain, and because disc displacement and soft
    tissues are well visualized.
  4. The most common source of pain from TMD is myofascial.
  5. The most common source of pain is degeneration
    or displacement of the TMJ’s disc brought about by
    malocclusion, bruxism and/or severe and chronic jaw
    clenching.
A
  1. Answer: A

Source: Goodwin J, Board Review 2005

402
Q
  1. Effects of spinal cord stimulation on circulation
    include:
  2. Increase in capillary density within stimulated region
  3. Increase in red cell volume within microcirculation at
    stimulation target
  4. Increase in number of capillaries perfused within stimulated
    region
  5. Decrease in diastolic blood pressure
A
  1. Answer: A ( 1, 2, & 3)
    Explanation:
    Spinal cord stimulation causes vasodilation in the
    stimulated region. This effect is the result of
    microvascular changes occurring at the capillary level
    most likely as a result of central inhibition of sympathetic
    neurons. Spinal cord stimulation does not cause
    macrocirculatory changes and blood pressure and pulse
    remain unchanged.
    Reference:
    Krames, Interventional Pain Management, Second Edition;
    Chapter 53 Mechanisms of Action of Spinal Cord
    Stimulation
    Source: Schultz D, Board Review 2004
403
Q
  1. Where a nerve root is permanently injured, on physical
    examination one might observe:
  2. fi brillations in muscles served by that root
  3. increased refl exes involving that root
  4. numbness in at least two adjacent dermatomes due to
    overlapping innervation
  5. fasciculations and atrophy of the muscles innervated
    by that root
A
  1. Answer: D

Source: Goodwin J, Board Review 2005

404
Q
1142. Which of the following central nervous system changes
occurs with age?
1. Reduced Sympathetic function
2. Low-frequency hearing loss
3. Increased muscle tone
4. Decrease in primitive refl exes
A
  1. Answer: B

Source: Day MR, Board Review 2006

405
Q
  1. Which of the following is or are true regarding
    Pseudotumor Cerebri (PTC) a.k.a. Benign Intracranial
    Hypertension (BIH)? Choose only one:
  2. It is strongly associated with obesity at all ages.
  3. It is probably, in part, due to overproduction of CSF
    rather than poor drainage of CSF
  4. It can occur without evidence of papilledema
  5. It is usually self-limited with only transient headaches
    that are commonly mistaken for the presence of a supratentorial
    tumor, ruled out by MRI hence the designations
    ‘pseudotumor’ and ‘benign’.
A
  1. Answer: A

Source: Goodwin J, Board Review 2005

406
Q
  1. Which of the statements concerning cluster headache are
    currently felt to be true? Choose one:
  2. The pain is always unilateral but can occur on one side
    or the other in any given individual over time
  3. Unlike migraine, cluster headaches do not induce nausea
  4. The only abnormal physical sign seen between attacks is an ipsilateral partial Horner’s syndrome (a minor
    degree of ptosis and myosis).
  5. The stabbing pain of cluster headaches occurs most
    commonly in the area innervated by the 3rd branch
    (V3) of the trigeminal nerve
A
  1. Answer: B

Source: Goodwin J, Board Review 2005

407
Q
  1. The visual analog scale is characterized by all of the
    following
  2. It is a progression of the numeric pain scale
  3. It uses a 10-cm line with 0 on one side and 10 on other
  4. The patient is asked to place a mark along the line to
    denote the level of pain
  5. It is a multidimensional pain scale
A
  1. Answer: A (1,2, & 3)

Source: Raj, Pain Review 2nd Edition

408
Q
  1. A 70 year old woman with spinal stenosis and lumbar
    radiculopathy is hospitalized for TIA episodes. She is
    placed on intravenous low molecular weight heparin
    (LMWH) because she is at high risk for stroke. You
    are asked by the neurologist to perform a lumbar
    epidural injection prior to hospital discharge to treat
    her radiculopathy. Which of the following statements
    regarding heparin and spinal injection are true?
  2. LMWH should be stopped for a minimum of 24 hours
    prior to spinal injection
  3. Low dose, subcutaneous heparin (5000 units every 12
    hours) creates minimal increased risk for spinal hematoma
    with spinal injection
  4. There is a higher incidence of spinal hematoma associated
    with LMWH than with unfractionated (standard)
    heparin
  5. An epidural catheter may be safely removed in a fully
    heparinized patient
A
  1. Answer: A ( 1, 2, & 3)
    Explanation:
    Reference:
    Horlocker, et. Al.
    Regional Anesthesia in the Anticoagulated Patient:
    Defi ning the Risks (The Second ASRA Consensus
    Conference on Neuraxial Anesthesia and Anticoagulation)
    When heparin is administered in the setting of spinal
    injection, there is increased risk for spinal bleeding in
    certain situations. The concurrent use of medications such
    as oral anticoagulants antiplatelet medications that affect
    other components of the clotting mechanism may increase
    the risk of bleeding complications for patients receiving
    standard heparin. Guidelines for spinal injection in the
    patient receiving standard, unfractionated heparin were
    established over 2 decades ago and are well outlined in the
    ASRA article listed above.
    Placement of a needle into the spinal canal and
    withdrawing a catheter from the spinal canal are both possible causes for epidural hematoma in the heparinized
    patient and recommendations for performing either spinal
    injection or catheter removal are similar. The following
    table illustrates the relative risk of spinal injection in
    various scenarios involving heparin:
    During SC heparin (mini-dose) prophylaxis, there is no
    contraindication to the use of neuraxial techniques. The
    risk of neuraxial bleeding may be reduced by delay of the
    heparin injection until after the block and may be
    increased in debilitated patients after prolonged therapy.
    Because heparin-induced thrombocytopenia may occur
    during heparin administration, patients receiving
    subcutaneous heparin for greater than 4 days should have
    a platelet count assessed prior to neuraxial block and prior
    to catheter removal.
    With unfractionated heparin, administration should be
    delayed for 1 hour after needle or catheter placement. In
    patients receiving heparin pre-procedure, spinal injection
    and/or catheter removal shouldbe performed after heparin
    cessation only after clotting status has returned to normal
    as determined by activated partial thromboplastin time
    (aPTT). Typically spinal injection should be delayed for at
    least 4 hours after the last heparin dose and indwelling
    neuraxial catheters should be removed 2 to 4 hours after
    the last heparin dose. Re-heparinization after spinal
    intervention should be delayed for 1 hour or longer.
    The biochemical and pharmacologic properties of LMWH
    differ from those of unfractionated heparin and patients
    receiving LMWH heparin are considered to be higher risk
    for spinal hematoma. During the fi rst 5 years of LMWH
    use in the United States, some 60 epidural hematomas
    were reported, prompting a “black box” warning label by
    the FDA.The most relevant differences with LMWH are
    its’ pro-longed half-life, its’ irreversibility with protamine
    and the lack of monitoring of the anticoagulant response
    with standard lab testing. Prolonged LMWH therapy may
    be associated with an accumulation of anti-Xa activity and
    fi brinolysis.
    Patients on preoperative lower dose LMWH for
    thromboprophylaxis can be assumed to have altered
    coagulation. In these patients, needle placement should
    occur at least 10 to 12 hours after the last LMWH dose.
    Patients receiving higher doses of LMWH for
    anticoagulation, such as enoxaparin 1 mg/kg every 12
    hours, enoxaparin 1.5 mg/kg daily, dalteparin 120 U/kg
    every 12 hours, dalteparin 200 U/kg daily, or tinzaparin
    175 U/kg daily will require delays of at least 24 hours to
    assure normal hemostasis at the time of needle insertion.
    Source: Schultz D, Board Review 2004
409
Q
  1. A patient undergoes a left stellate ganglion block without
    fl uoroscopic guidance. One week later the patient develops
    intermittent fever and increasingly severe posterior neck
    pain. The following statements are true:
  2. The patient should be treated with a 10 day course of
    oral antibiotics.
  3. Discitis is more likely after a left-sided vs. a right sided
    stellate block.
  4. Symptoms are most likely unrelated to the stellate ganglion
    block.
  5. The most appropriate immediate work up includes ESR,
    C-reactive protein and cervical MRI scan.
A
  1. Answer: C (2 & 4)
    Explanation:
    Reference:
    Pyogenic cervical epidural abscess and discitis following
    stellate ganglion block
    Vadodaria B.S., Bridgens J. and Richmond M.
    Anaesthesia 2001 56:9 (871-873)
    Disc space infection (commonly called discitis) is a rare but potentially catastrophic complication of injection into
    the spinal region. Any time a needle enters an
    intervertebral disc there is a potential for this
    complication. Epidural abscess and/or vertebral body
    osteomyelitis are common consequences of untreated disc
    space infection. Symptoms may include increasingly severe
    neck pain worse with movement, fevers and general
    malaise. Rapid diagnosis and aggressive treatment are of
    vital importance to prevent severe neurological
    consequences and life-threatening infection. The most
    important early diagnostic maneuvers include laboratory
    analysis with ESR and C-reactive protein and emergent
    MRI scan. ESR is a somewhat nonspecifi c test and Creactive
    protein is more specifi c and sensitive for discitis.
    The MRI may be relatively normal early on in the course
    of discitis and repeat MRI scans may be used to reevaluate
    and track progression or regression of disc space
    infection.
    MRI is also important to rule out epidural abscess which
    may require surgical intervention. Irradication of disc
    space infection in the relatively avascular disc is diffi cult
    and treatment typically includes intravenous antibiotics
    for a period of many weeks.
    Discitis is most commonly associated with discography
    but can occur after stellate ganglion block since the needle
    is in close proximity to the disc and disc penetration can
    occur. In the cervical region, the esophagus is a left-sided
    structure and any needle traversing the esophagus and
    entering a cervical disc has the potential to cause disc
    space infection. For this reason, cervical discography
    is always
    performed from the right side. Left-sided stellate ganglion
    block therefore has an inherently higher risk of discitis.
    Source: Schultz D, Board Review 2004
410
Q
  1. A 50-year old typist complains of numbness of 6 weeks
    in duration in her entire right hand that is relieved by
    placing her hand under cold running water. Numbness
    and tingling are prominent to the index fi nger when
    driving her car or typing. She had entered menopause
    fi ve years previously. She is also a non-insulin dependent
    diabetic for approximately 10 years. There was no
    evidence of diabetic neuropathy. The true statements
    regarding this patient’s condition include:
  2. She suffers with carpal tunnel syndrome
  3. Nerve conduction studies showed a prolong residual
    latency and normal conduction velocity in the forearm
  4. Treatment includes non-steroidal anti-infl ammatory
    agents, a cock-up volar splint with wrist is loose-packed
    position (10° to 30° dorsifl exion) to be worn during day
    and night and tendon-gliding exercises.
  5. Her diagnosis is C6 radiculopathy caused by cervical
    spondylosis.
A
  1. Answer: A (1,2, & 3)
    Explanation:
    Carpal tunnel syndrome is one of the most common, best
    defi ned, and most carefully studied entrapment
    neuropathies. It affects middle-aged females between 40
    and 60 years of age, that is menopausal women, a
    characteristic suggestive of a hormonal aberration as a
    causative development of this disorder. The most
    common cause of carpal tunnel syndrome is an idiopathic
    non-specifi c fl exor tenosynovitis that may simply arise
    from chronic repetitive occupational stress, both in males
    and females. Carpal tunnel syndrome may occur acutely
    after lunate bone dislocation or from a Colles’ fracture and
    requires immediate medical attention as to prevent acute
    nerve ischemia.
    * Carpal tunnel syndrome may be subdivided into one of
    the four categories.
    - An increase in volume or tunnel content secondary to
    non-specifi c tenosynovitis of the fl exor tendons within the
    carpal tunnel
    - Thickening or fi brosis of the transverse carpal
    ligament
    - Alteration of the osseous modus of the carpus caused by fractures, dislocations or arthritic joint changes
    - Tumor or systemic disease
    * The median nerve has both sensory and motorbranches.
    During median nerve compression at the carpal tunnel
    sensory, abnormalities usually occur fi rst only to progress
    to motor involvement as the pathology evolves.
    * Clinical fi ndings are proportional to the degree of nerve
    damage, which in turn is related to the severity of
    compression and not to the duration of compression.
    * The differential diagnosis includes C6 radiculopathy
    with refl ex changes and EMG studies showing denervation
    out of the median nerve territory and sensory loss of the
    6th cervical dermatome.
    * Other diagnosis include:
    - Pronator syndrome referring to compression of the
    median nerve by pronator muscle as it passes through the
    heads of that muscle and to a lesser extent,by fi brous
    bands near the origin of deep fl exor muscles known as the
    lacertus fi brosis and fl exor digitorum superfi cialis arcade,
    and even less commonly by the ligament of Struthers, an
    analomous structure found in about 1% of the population.
    Pronator syndrome may also be expressed with expressed
    with median nerve paresthesias mimicking those of CTS,
    it differs in several aspects. Night pain, symptoms brought
    on by wrist movement, intrinsic weakness of opponents
    and abduction movements, as well as positive Phalen and
    Tinel wrist signs are not common to this condition.
    - Other conditions include anterior interosseous
    syndrome.
    - Carpal tunnel is diagnosed with positive Phalen’s test
    or Tinel’s sign where the median nerve is easily
    depolarized when mechanically stimulated by direct
    tapping over the palmaris longus tendon over the fl exor
    retinaculum. However, positive fi ndings occur only in
    approximately 45% of all cases.
    Source: Saidoff DC, McDonough AL. Critical Pathways in
    Therapeutic Intervention. Extremities and Spine.
    St. Louis,Inc., 2002
411
Q
  1. Regarding Nerve Conduction Studies:
  2. Slowing of conduction velocity most often implies demyelination
  3. Diminished amplitude of the action potential implies
    axonal damage
  4. Prolonged distal latencies are seen in entrapment neuropathies
  5. Needle electrodes are used only in morbidly obese individuals
    due to attenuation of the AP signal secondary
    to adipose tissue transduction blockade
A
  1. Answer: A

Source: Goodwin J, Board Review 2006

412
Q
  1. With audio amplifi cation, a ‘dive bomber’ sound is
    characteristic of which phenomenon?
  2. A post synaptic, decremental response-generating condition
    such as myesthenia gravis
  3. Myokymia
  4. Pronged but attenuating fi brillation potentials
  5. Myotonia
A
  1. Answer: D

Source: Goodwin J, Board Review 2006

413
Q
  1. Which of the following statements is true?
  2. In compression neuropathies, sensory nerve conduction
    tests are more sensitive than motor.
  3. Plexopathies usually involve diminished SNAP’s
  4. In radiculopathies, SNAP’s are usually undiminished
  5. The absence of paraspinal muscle fi brillations rules out
    radiculopathy
A
  1. Answer: A

Source: Goodwin J, Board Review 2006

414
Q
1152. Which of the following are common fi ndings on MRI
with an epidural abscess?
1. Discitis
2. Dural enhancement
3. Vertebral osteomyelitis
4. Epidural fl uid collection
A
  1. Answer: E (All)

Source: Bieneman B, Board Review 2005

415
Q
  1. Which of the following are true regarding multiple
    myeloma?
  2. Most common primary bone tumor
  3. Bone scan is normal in majority of cases
  4. MRI more sensitive than plain radiographs
  5. Long bones more often involved than axial skeleton
A
  1. Answer: A (1,2, & 3)

Source: Bieneman B, Board Review 2005

416
Q
  1. Which of the following is or are true with respect to post
    lumbar puncture headaches (PLPH?
  2. Sprotte or Whitacre needles increase the risk of PLPH as
    compared to Quincke needles because they cut rather
    than spread apart the longitudinal fi bers of the dura
    mater.
  3. While strict bed rest, IV caffeine and IV theophyline
    may help reduce or stop a PLPH, the quickest and most
    effective method is a blood patch.
  4. Lying prone for 3 hours after a lumbar puncture reduces
    the incidence of PLPH by 30-50%.
  5. Cranial nerve 6 is the most likely cranial nerve to be
    affected by low CSF levels because it is the longest one
    exposed to low CSF levels and can be stretched over the petrous ridge of the temporal bone when the CSF
    levels fall.
A
  1. Answer: C

Source: Goodwin J, Board Review 2005

417
Q
1155. Which of the following hepatic metabolic pathways
dcrease with age?
1. Conjugation
2. Microsomal hydroxylation
3. Oxidation
4. Demethylation
A
  1. Answer: D

Source: Day MR, Board Review 2006

418
Q
  1. A 38-year-old white male with chronic low back pain
    and history of alcoholism, on total of 200 mg of morphine
    per day, was admitted to the emergency room because he
    was found by his neighbors to be acting agitated and
    confused. ER physician notifi es you of his admission.
    Which of the following identifi es delirium tremens in
    differential diagnosis of this patient’s condition?
  2. Clear sensorium
  3. Prominent tremor
  4. Auditory hallucination
  5. Dilated pupils with slow reaction to light
A
  1. Answer: B
    Explanation:
    1.There is diffi culty sustaining attention, disorganized
    thinking, and perceptual disturbances.
    2.Acute alcoholic hallucinosis may start without a drop in
    blood alcohol concentration, and without delirium,
    tremor, or autonomic hyperactivity
    3.Hallucinations are usually auditory and paranoid and
    may last more than 10 days.
    4.In delirium tremens, the patient is confused, with
    prominent tremor and psychomotor activity, disturbed
    vital signs, autonomic dysfunction with dilated pupils, and
    a slow reaction to light. Hallucinations are usually of the
    visual type
    Source: Laxmaiah Manchikanti, MD
419
Q
1157. A massage technique that applies gentle taps or blows
would be classifi ed as a
1. stroking
2. petrissage
3. friction vibration
4. percussion
A
  1. Answer: D (4 only)
    Explanation:
    (Raj, Practical Mgmt of Pain 3rd Ed., page 538-539)
    Massage
    Massage is the scientifi c application of force by the hands
    to soft tissue, usually the skin, fascia, muscles, tendons, and
    ligaments, to produce a therapeutic effect. Several types
    exist
    Stroking or Effl eurage
    Kneading and Pétrissage
    Friction Massage
    Percussion, Tapotement, or Clapping
    Stroking and Vibration
    Source: Shah RV, Board Review 2005
420
Q
  1. The criteria for diagnosing hypochondriasis include all
    of the following:
  2. Preoccupation with the fear of having a serious disease
  3. Persistent preoccupation despite medical reassurance
  4. The preoccupation has a duration of 6 months or more
  5. The preoccupation is delusional
A
  1. Answer: A ( 1, 2, & 3)

Source: Janata JW, Board Review 2005

421
Q
  1. Psychophysiologic assessment might include:
  2. Malingering indices
  3. Depression scales
  4. The MMPI
  5. Biofeedback assessment
A
  1. Answer: D (4 only)

Source: Janata JW, Board Review 2005

422
Q
1160. Complex pain presentations are generally optimally
treated using:
1. A single medical specialist
2. Biofeedback
3. Polypharmacy
4. Interdisciplinary treatment
A
  1. Answer: D (4 only)

Source: Janata JW, Board Review 2005

423
Q
  1. The Beck Depression Inventory may slightly overestimate
    depression in pain populations because it includes:
  2. Family history of depression
  3. A malingering scale
  4. An interpersonal distress index
  5. Somatic symptoms
A
  1. Answer: D (4 only)

Source: Janata JW, Board Review 2005

424
Q
1162. Which of the following describes a method of heat
transfer
1. Conduction
2. Convection
3. Conversion
4. Radiation
A
  1. Answer: E
    Explanation:
    (Raj, Practical Mgmt of Pain, 3rd Ed., page 530)
    Conduction is the transfer of thermal energy between two
    bodies in direct contact.
    Convection uses movement of a transfer medium such as
    air or water to convey the change in temperature.
    Conversion is the transformation of energy in one form,
    such as sound, into another, such as heat.
    Radiation is the thermal energy given off by any object
    whose surface temperature is above absolute zero.
    Source: Shah RV, Board Review 2005
425
Q
1163. A pain psychologist might use the Millon Behavioral
Health Inventory to assess:
1. Basic coping style
2. Psychogenic attitudes
3. Psychosomatic correlates
4. Personality Disorders
A
  1. Answer: A ( 1, 2, & 3)

Source: Janata JW, Board Review 2005

426
Q
1164. The visceral afferent fi bers of the heart are transmitted
through what nerves
1. Vagus
2. Middle and inferior cervical ganglia
3. Thoracic cardiac nerves
4. Thoracic ganglia 3-6
A
  1. Answer: A (1, 2, and 3)
    Explanation:
    Reference: Raj. Chapter 43. Thoracoabdominal Pain. In:
    Practical Management of Pain.3rd Edition. Raj et al,
    Mosby, 2000. page 618
    Source: Day MR, Board Review 2005
427
Q
  1. On T2-weighted images of the lumbar spine
  2. Intervertebral disc height is usually greatest at L4/5
  3. Disc signal intensity is greatest at its central aspect
  4. The nuclear cleft is normal in most cases
  5. The conus usually ends at or above L1
A
  1. Answer: A (1,2, & 3)
    Explanation:
    The conus usually ends at or above L2. (reference:
    Renfrew; Atlas Spine Imaging, Saunders, 2003, page 1)
    Source: Bieneman B, Board Review 2005
428
Q
  1. Conus Medullaris Syndrome differs from Cauda Equina
    Syndrome in that the former:
  2. is less likely to be painful, but if present, is a relatively
    mild perineum and thigh pain
  3. results in earlier and more severe sphincter dysfunction
  4. generally presents with symmetrical and bilateral sensory
    defi cits
  5. generally presents with radicular pain
A
  1. Answer: A

Source: Goodwin J, Board Review 2005

429
Q
  1. Subacute combined degeneration due to vitamin B12
    defi ciency typically produces which of the following
    spinal cord changes on imaging?
  2. Atrophy of the cord on CT scan
  3. MRI T2-weighted hyperintensity of dorsal columns
  4. Hyperintensity of ventral cord on T2-weighted MRI
  5. Mild cord enlargement with abnormal signal in dorsal
    cord
A
  1. Answer: C (2 & 4)

Source: Bieneman B, Board Review 2005

430
Q
  1. Acute infl ammatory demyelinating polyradiculipathy
    (Guillain-Barre Syndrome) is characterized by which of
    the following?
  2. Diffuse enhancement of cauda equina and conus medularis
  3. Atrophy frequently present in images of anterior cord
  4. Best seen on MRI scan
  5. Readily visualized on CT scans
A
  1. Answer: B (1 & 3)
    Explanation:
    The most common cause of acute paralysis in the western
    world, also known as ascending paralysis. An
    infl ammatory demyelinating disease involving peripheral
    nerves, nerve roots and cranial nerves. Believed to occur
    most commonly after viral illness, campylobacter infection
    or with autoimmune responses, such as with vaccination
    Source: Bieneman B, Board Review 2005
431
Q
  1. Regarding demyelinating diseases of the spinal cord
  2. Cord edema and atrophy are common in the acute setting
  3. T1 weighted sequence with contrast is best for diagnosing
    MS
  4. All actively demyelinating lesions enhance
  5. ADEM and MS may have an identical appearance
A
  1. Answer: D (4 Only)

Source: Bieneman B, Board Review 2005

432
Q
  1. Which items are safe for MRI?
  2. Internal orthopedic hardware
  3. Intrauterine device
  4. Epilepsy depth electrodes
  5. Drug infusion pumps
A
  1. Answer: E (All)
    Explanation:
    It is alwatys important o refer to an up-to-date reference of
    MRI compatible devices before imaging patients, however,
    these devices are usually safe.
    Source: Bieneman B, Board Review 2005
433
Q
1171. Spray and stretch is a therapeutic cold technique that
uses
1. Ice massage
2. Cold water immersion
3. Cold Packs
4. Ethyl chloride
A
  1. Answer: D (4 only)
    Explanation:
    (Raj, Practical Mgmt of Pain, 3rd Ed., page 532-533)
    Vapocoolant Spray
    Topical anesthetics like ethyl chloride and fl uori-methane
    are used in the technique of spray-and-stretch to treat the
    many myofascial and musculoskeletal pain syndromes
    typically characterized by the presence of trigger points.
    The trigger point and its referral zone are sprayed in
    unidirectional parallel sweeps while the muscle is
    maintained at a passive stretch.
    Source: Shah RV, Board Review 2005
434
Q
  1. With regards to NCS’s:
  2. Slowing of conduction velocity most often implies demyelination.
  3. Diminished amplitude of the action potential implies
    axonal damage
  4. Prolonged distal latencies are seen in entrapment neuropathies
  5. Needle electrodes are used only in morbidly obese individuals
    due to attenuation of the AP signal secondary
    to adipose tissue transduction blockade
A
  1. Answer: A

Source: Goodwin J, Board Review 2005

435
Q
  1. Relative and absolute contraindications to MRI include:
  2. hemodynamic instability
  3. implanted pacemaker or defi brillator
  4. signifi cant claustrophobia
  5. Severe contrast reaction
A
  1. Answer: A (1,2, & 3)
    Explanation:
  2. Patients who are hemodynamically unstable should not
    be placed in an MRI, as there is limited access to the
    patient.
  3. Implanted pacemaker or defi brillator is a
    contraindication for MRI.
  4. Claustrophobia is a relative contraindication and
    pacemakers are an absolute contraindication.
  5. Contrast reaction is not a contraindication to MRI.
    It is a contraindication to contrast administration.
    Source: Bieneman B, Board Review 2005
436
Q
1174. Which of the following modalities does not use ionizing
radiation:
1. MRI
2. CT scan
3. Ultrasound
4. Radiography
A
  1. Answer: B (1 & 3)
    Explanation:
    (See lecture notes)
    Source: Bieneman B, Board Review 2005
437
Q
  1. MR signal:
  2. Is based on electron spins
  3. Fluid is hypointense on T1
  4. Water is hyperintense on T2
  5. The nucleus pulposus is dark on T2
A
  1. Answer: A (1,2, & 3)

Source: Bieneman B, Board Review 2005

438
Q
  1. Immediately following complete transection of the
    spinal cord, deep tendon refl exes and muscle tone below
    the level of the lesion are most likely to:
  2. Increase
  3. Fluctuate according to the presence or absence of dysautonomia
  4. Remain unchanged
  5. Decrease
A
  1. Answer: B

Source: Goodwin J, Board Review 2005

439
Q
  1. On axial MRI images of the lumbar spine
  2. Ventral rami of L5 nerves lie on the ventral sacrum
  3. Nucleus pulposus demonstrates T2 prolongation compared
    to the annulus
  4. The disc constitutes the anterior border of the intervertebral
    foramen
  5. The superior articular process lies anterior to the inferior
    process
A
1177. Answer: E (All)
Explanation:
All of the above (reference: Renfrew; Atlas Spine Imaging,
Saunders, 2003, page 5)
Source: Bieneman B, Board Review 2005
440
Q
  1. FDA approved indications for spinal cord stimulation
    include
  2. Angina
  3. Peripheral vascular disease
  4. Chronic pelvic pain
  5. Neuropathic leg pain associated with multiple lumbar
    spine surgeries
A
  1. Answer: D (4 Only)
    Explanation:
    There is literature support for spinal cord stimulation to
    treat angina, peripheral vascular disease and chronic pelvic
    pain but none of these indications are FDA approved at
    present.
    Source: Schultz D, Board Review 2004
441
Q
  1. Which of the following reduce the rate of post-dural
    puncture headache, actually or theoretically?
  2. using a Whitacre needle instead of a Quincke-Babcock
  3. using a 25 gauge needle instead of a 24 gauge
  4. advancing a Quincke-Babcock needle parallel to the
    dura instead of perpendicular
  5. using a 6 inch needle instead of a 4 inch needle
A
  1. Answer: A (1,2, & 3)
    Explanation:
    (Raj, Practical Management of Pain, 3rd Ed., page 633)
    Needle length, as well as needle tip shape, may affect the
    length of time you have to wait before recognizing thatyou
    are subarachnoid, i.e., it takes longer for CSF to drip out of
    a longer, as compared to shorter needle.Spinal needles vary
    with regard to length, inside and outside diameters, as
    well as the shape of their tip. The latter affects the size and
    shape of the hole made in the dura as well as the speedwith which CSF appears in the hub after dural puncture. The
    incidence of postdural puncture headache appears to be
    directly related to the size of needle used and the
    orientation of the needle in performing spinal anesthesia.
    A spinal needle oriented parallel to the dura separates the
    fi bers rather than cutting them, as a perpendicularly
    oriented needle does, and produces a smaller defect in the
    dura.
    All spinal needles come with a removable stylet, which
    must be close-fi tting to prevent coring of the skin and the
    resultant obstruction of the needle and contamination of
    spinal space with epidermal tissue and skin bacteria.
    Several spinal needle types and sizes are commercially
    available, although only two different needle tip points are
    available. The tip points can have either a beveled cutting
    point or a noncutting, rounded pencil point.
    The commonly used spinal needle with a cutting point is
    the Quincke-Babcock, which has a short bevel with cutting
    edges and a rounded heel. The cutting-point spinal needles
    appear to be associated with a high incidence of postspinal
    headache even when smaller needles are used. Spinal
    needles with a noncutting, rounded, pencil tip seem to
    cause less trauma to the dura mater and appear to be
    associated with a lower incidence of postspinal headache
    when larger-caliber needles are used.
    The Greene, Sprotte, Whitacre, and Huber needles have a
    noncutting, rounded, pencil tip. The Sprotte and Whitacre
    needles have completely rounded non-cutting bevels with
    solid tips, and the opening on their side approximately 2
    mm proximal to the tip. These are currently the most
    widely used needles for spinal anesthesia as a result of their
    association with a reduced incidence of postdural
    puncture headache.
    Note: the Greene, Sprotte, Whitacre, and Huber needles do
    not have to be advanced parallel to the dura, since they are
    ‘pencil point’ needles.
    Source: Shah RV, Board Review 2005
442
Q
1180. Indications for spinal imaging in pediatric patients with
back pain include
1. Neurologic fi ndings
2. Decreased function
3. Chronic pain
4. Fever
A
  1. Answer: E (All)

Source: Bieneman B, Board Review 2005

443
Q
  1. Which of the following statements is true regarding
    peripheral nerve stimulation:
  2. Positive response to TENS is a reliable predictor for
    positive response to peripheral nerve stimulation
  3. Pain relieving effects are caused by local anesthetic-like
    blockade of neural conduction within the peripheral
    nerve.
  4. Pain due to nerve root injury often responds well to
    PNS
  5. The best indication is pain in the distribution of a single
    traumatized peripheral nerve
A
  1. Answer: D (4 Only)
    Explanation:
    Reference:
    Heavner, Interventional Pain Management, Second
    Edition; Chapter 57 Peripheral Nerve Stimulation:
    Current Concepts
    The pain relieving effects of PNS are similar to those of
    SCS and are thought to be mediated by stimulation of Abeta
    fi bers within the peripheral nerve with subsequent
    activation of local inhibitory circuits within the dorsal
    horn. Peripheral nerve stimulation is best used to treat
    pain caused by trauma to a single peripheral nerve
    although two peripheral nerves within the same region can
    successfully be treated with a single stimulation system (2 leads and 1 pulse generator or receiver). Pain due to nerve
    root injury or to spinal mechanisms usually does not
    respond to PNS. Patients who have a positive response to
    TENS may be somewhat more likely to respond to PNS
    although TENS response is not a reliable predictor of PNS
    effect and a negative response to TENS does not mean that
    PNS should not be tried.
    Source: Schultz D, Board Review 2004
444
Q
  1. AIDS-induced vacuolar myelopathy, involving the
    posterior columns of the spinal cord, results in the loss of
    which sensory modalities?
  2. Pain and temperature sensation contralateral to and
    below the side of the lesion
  3. Proprioception and vibratory sensation of the lower and
    upper extremities
  4. Inability to detect a full bladder
  5. Two point discrimination
A
  1. Answer: C

Source: Goodwin J, Board Review 2005

445
Q
  1. Which of the following about hypophysectomy is true?
  2. Stereotactic instillation of phenol is the most commonly
    described method
  3. Gamma knife stereotactic radiotherapy of the hypothalamus
    is administered percutaneously
  4. The analgesic mechanism is thought to be due to interruption
    of limbic pathways
  5. One of the best described indications is diffuse pain due
    to bone metastases from breast or prostate carcinoma
A
  1. Answer: D (4 Only)
    Explanation:
    (Raj, Pain Review 2nd Ed., page 309)
    Percutaneous stereotactic instillation of alcohol is the best
    described technique. Other percutaneous methods for
    hypophysectomy include the use of radiofrequency
    thermocoagulation, cryotherapy, or radioactive seeds.
    Gamma knife radiotherapy is a noninvasive method for
    creating hypophyseal lesions. The analgesic mechanism is
    unknown, but limbic system or psychological effects are
    unlikely to be the reasons for pain relief. Hypophysectomy
    is recommended in the treatment of metastatic prostate
    and breast cancer, irrespective of the hormonal
    responsiveness of the tumors.
    Source: Schultz D, Board Review 2004
446
Q
1184. What are the main types of cervical spine pathology
found in rheumatoid arthritis?
1. Cranial settling
2. Atlanto-axial subluxation
3. Erosion or fusion of the facet joints
4. Multilevel subluxations
A
  1. Answer: E (All)
    Explanation:
    (reference Renfrew page 354)
    Source: Bieneman B, Board Review 2005
447
Q
  1. For the following non-organic signs, which method of
    assessment is a reasonable?
  2. Numbness around torso: use 128Hz tuning fork to see if
    vibration is felt above the line
  3. Astasia-abasia: refer to ENT to rule out a problem with
    the inner ear before jumping to conclusions
  4. Paralysis of one leg: use Hoover’s maneuver to check for
    reciprocal extension
  5. Loss of consciousness: squeeze nipples really hard to see
    if the patient ‘wakes up’
A
  1. Answer: B

Source: Goodwin J, Board Review 2005

448
Q
  1. Which of the following is not a deep heating method?
  2. Ultrasound
  3. Phonopheresis
  4. Diathermy
  5. Hot packs
A
1186. Answer: D (4 only)
Explanation:
(Raj. Practical Mgmt of Pain, 3rd Edition, page 530-532)
Superficial                   Deep
Hot Packs                    Ultrasound
Paraffin                         Diathermy
Heat Lamps                 Phonophoresis
Hydrotherapy
Fluidotherapy
Source: Shah RV, Board Review 2005
449
Q
  1. Pain behaviors can include all of the following:
  2. Reduced activity level
  3. Verbal behavior
  4. Nonverbal behavior
  5. Pain beliefs
A
  1. Answer: A ( 1, 2, & 3)

Source: Janata JW, Board Review 2005

450
Q
  1. Which items are not safe for MRI?
  2. Chest ports for chemotherapy
  3. Oxygen tanks
  4. All types of cerebral aneurysm clips
  5. Metal in the eye
A
  1. Answer: C (2 & 4)
    Explanation:
    It is always best to refer to an up-to-date reference of MRI
    compatible devices before imaging patients, however,
    foreign body metal and metallic objects unrelated to
    patients are not safe. Current cerebral aneurysm clips are
    MRI compatible, however, older clips may not be- again
    check the operative notes to determine if a clip is non ferromagnetic
    Source: Bieneman B, Board Review 2005
451
Q
  1. A patient presents with an acute onset of upper extremity
    pain with numbness in the little fi nger. Physical
    examination showed weakness with fi nger fl exors. Most
    likely, diagnosis in this patient is:
  2. C5 nerve root involvement
  3. C6 nerve root involvement
  4. C7 nerve root involvement
  5. C8 nerve root involvement
A
  1. Answer: D (4 Only)
    Explanation:
    Flexor digitorum superfi cialis is supplied by a median
    nerve, C8. Similarly, fl exor digitorum profundus are also
    supplied by medial and ulnar nerves, C8. Lumbricals are
    supplied by median and ulnar nerve, C8 and T1.The fl exor
    digitorum profundus, which fl exes the distal
    interphalangeal joint, and the lumbricals,which fl ex the
    metacarpal phalangeal joint, usually receive innervation
    from the ulnar nerve on the ulnar side of the hand and
    from the median nerve on the radial side. If there is any
    injury to the C8 nerve root, the entire fl exor digitorum
    profundus becomes weak, with secondary weakness in all
    fi nger fl exors. If, however, there is a peripheral injury to
    the ulnar nerve, weakness will exist only in the ring and
    little fi ngers. The fl exor digitorum superfi cialis, which
    fl exes the proximal interphalangeal joint, has only median
    nerve innervation, and is affected by root injury to C8 and
    peripheral nerve injuries to the median nerve.
    C8 supplies sensation to the ring and little fi ngers of the
    hand and the distal half of the forearm. The ulnar side of
    the little fi nger is the purest area for sensation of the ulnar
    nerve, which is predominantly C8, and is most effi cient
    location for testing.
    Source: Hoppenfeld S. Orthopaedic Neurology. A
    Diagnostic Guide to Neurologic Levels. Philadelphia,
    LWW, 1997.
452
Q
  1. Subdural blockade is typically characterized by:
  2. immediate onset of sensory and motor block
  3. motor paralysis and sensory preservation
  4. upper motor neuron signs
  5. occurrence following the injection of a small volume of
    local anesthetic
A
  1. Answer: D (4 only)
    Explanation:
    (Raj, Practical Mgmt of Pain, 3rd Edition, page 648)
    The subdural space is a potential space between dura and
    arachnoid mater. Injection of drugs into this space can
    cause extensive and erratic spread. The onset of a block
    after a subdural injection characteristically is slower (5 to
    10 minutes) than after an intrathecal block (3 to 5
    minutes) but signifi cantly faster than an epidural injection
    (10 to 20 minutes). Moreover, a profound patchy sensory
    block with mild motor block may develop.
    Diagnosis can often be made by subsequent injection of a
    radiopaque dye in case an epidural catheter was threaded
    into the subdural space. A small volume of dye (e.g., 5 ml)
    shows an extensive spread of a very thin fi lm of the dye that is confined within the subdural space. The incidence of
    subdural injections is between 0.3% and 1%.
    Motor paralysis and sensory preservation is typically due
    to anterior spinal artery syndrome, whereas motor
    preservation and sensory loss is associated with subdural
    blockade.
    Source: Shah RV, Board Review 2005
453
Q
  1. Loss of resistance to air or saline describes a method to
    access the epidural space. What are the characteristics of
    the ligamentous structure that offers ‘resistance’?
  2. A ligament that is weaker than the supraspinous ligament
  3. A ligament that is composed of 20% elastin
  4. A ligament that spans from the anterior surface of the
    caudad lamina to the posterior surface of the cephalad
    lamina
  5. A ligament that is referred to as the yellow ligament
A
  1. Answer: D (4 only)
    Explanation:
    (Raj, Practical Management of Pain, 3rd Ed., page 638)
    The ligamentum fl avum, which consists of more than 80%
    elastin, is the toughest of the three ligaments. It usually is
    easy to identify by its increased resistance to advancement
    of the epidural needle and ability to inject air or saline
    solution. It spans from the anterior surface of the cephalad
    lamina of an adjacent pair of vertebrae to the posterior
    aspect of the lower lamina. The ligamenta fl ava arise
    embryonically from two separate laminae. They fuse to a
    variable degree in the midline. Sometimes the fusion is
    incomplete, which may unintentionally lead to a dural
    puncture. The right and left halves meet at an angle of less
    than 90 degrees. The lateral edges wrap anteriorly around
    the facet joints.
    Source: Shah RV, Board Review 2005
454
Q
  1. Which of the following are true regarding segmentation
    abnormalities of the spine?
  2. Vertebral column abnormalities due to congenital malformations
  3. Classic fi nding is scoliosis with deformed vertebral
    bodies
  4. Posterior element defects best seen on axial CT views
  5. Most common form is an indeterminate (transitional)
    vertebra
A
  1. Answer: E (All)

Source: Bieneman B, Board Review 2005

455
Q
  1. Which of the following is true regarding neurolytic
    blocks?
  2. Target is the ventral root
  3. Lumbar subarachnoid neurolysis is performed at the
    vertebral level corresponding to the level of desired
    blockage.
  4. Lower potential for motor defi cits using the epidural
    approach
  5. Less predictable spread of the neurolytic agent with an
    epidural approach
A
  1. Answer: D (4 only)

Source: Day MR, Board Review 2005

456
Q

1194.Regarding polysomnography (PSG), which of the
following is true?
1. This is a good test to order prior to seeing a sleep specialist
given the information it can generate
2. It is a poor choice of testing if the patient has trouble
sleeping because the data will be invalid unless he or she
gets at least 4 hours of sleep.
3. The strength of the test is that two night’s testing results
are averaged for accuracy
4. With a history of restless legs syndrome, periodic leg
movements of sleep will likely show up

A
  1. Answer: D

Source: Goodwin J, Board Review 2005

457
Q
  1. Regarding headaches associated with tumors, which of
    the following are consistent with clinical experience?
  2. Slow growing tumors and those that compress the brain
    from outside are more likely to present with seizures
    than with headache.
  3. Infratentorial tumors are more likely to present with
    headache than supratentorial tumors.
  4. 90% of childhood tumors will cause headache at some
    point while the corresponding number in adults is
    60%.
  5. Supratentorial tumors tend to refer pain anteriorly to
    the frontotemporal region
A
  1. Answer: E

Source: Goodwin J, Board Review 2005

458
Q
  1. Which of the following actions (or lack thereof) could
    lead to severe complications, put the patient into a
    persistent vegetative state or prove fatal in the case of
    certain intracranial infections?
  2. Failure to consider subdural empyema in the face of
    classic signs and symptoms, failing in the meantime to
    get the necessary neurosurgical consultation.
  3. Delaying the L.P and / or wide spectrum antibiotics
    when imaging studies are delayed or unavailable having
    been requested to rule in or out a supratentorial mass
    that could, if present, lead to uncal herniation via CSF
    withdrawal by lumbar puncture.
  4. Failure to use IV acyclovir at the same time as other
    antibiotics since Herpes simplex is the only directly treatable life threatening viral infection of the brain parenchyma,
    the diagnosis of which may take some time.
  5. Failing to check for Brudzinski and Kernig signs when
    doing the neurologic evaluation following a quick and
    incomplete history.
A
  1. Answer: E

Source: Goodwin J, Board Review 2005

459
Q
1197. Cervical traction would be useful in which of the
following conditions
1. Arnold-chiari malformation
2. cervical myelopathy
3. rheumatoid arthritis
4. cervical disc herniation
A
  1. Answer: D (4 only)
    Explanation:
    (Raj, Practical Mgmt of Pain, 3rd Ed., page 540)
    TRACTION
    In the therapy of traction, the soft tissues of the body
    (cervical or lumbar spine) are stretched by a pulling
    (traction) force. This force can be applied either manually
    or mechanically. Factors that determine the amount of
    separation (and thus pain reduction) include the position
    of the spine, the angle of pull, and the amount of force
    applied.Traction, when applied properly, may prevent
    adhesion formation, subdue painful muscle spasm, relieve
    pain, maintain anatomic alignment, and prevent or correct
    a deformity.
    Contraindications to the use of these techniques include
    acute trauma, infl ammation, hypermobility, increasing
    pain, and any spinal condition in which movement is to be
    avoided.
    Source: Shah RV, Board Review 2005
460
Q
  1. The following are advantages of a surgically implanted
    “paddle” lead over a percutaneous wire lead:
  2. Positional stimulation is less common
  3. There is less risk of lead migration
  4. There is dorsal shielding with less stimulation of dorsal
    structures
  5. The lead is easier to implant
A
  1. Answer: A ( 1, 2, & 3)
    Explanation:
    Reference:
    Bedder, Interventional Pain Management, Second Edition;
    Chapter 55 Implantation Techniques for Spinal Cord
    Stimulation
    Surgical paddle leads over the following advantages:
    · Broader surface area with less risk of migration
    · Dorsal shielding with less uncomfortable stimulation of
    dorsal spinal structures
    · Less positional stimulation
    · Increased effi ciency and less power requirement
    The leads are more diffi cult to implant because they
    require a laminotomy to insert into the spinal canal.
    Source: Schultz D, Board Review 2004
461
Q
  1. Indications for a trial of spinal cord stimulation
    include:
  2. Complex regional pain syndrome of bilateral extremities
  3. Refractory angina
  4. Post-laminectomy syndrome with neuropathic back
    and leg pain
  5. Severe intractable ibromyalgia
A
  1. Answer: A ( 1, 2, & 3)
    Explanation:
    Krames, Interventional Pain Management, Second Edition;
    Chapter 54 Spinal Cord Stimulation: Patient Selection
    Spinal cord stimulation is effective for neuropathic pain. It
    is most likely to be effective for single or bilateral
    extremity neuropathic pain but has a reasonable chance
    for success in cases of intractable neuropathic back and leg pain. It is not indicated for axial somatic, nociceptive pain except in the case of intractable angina where it has proven
    to be effi cacious.
    Source: Schultz D, Board Review 2004
462
Q
  1. Regarding Evoked Potentials in general, which of the
    following is true?
  2. The BSAEP’s anatomic pathway is the middle ear, 8th
    cranial nerve, brainstem and auditory cortex
  3. The P300 waveform latency in CP’s can be delayed by
    autism, Schizophrenia and dementia
  4. The most commonly used peripheral nerve in SSEP testing
    is the posterior tibial
  5. The most important peak of electrical activity in VEP’s
    is the P120 with a maximum latency of 100 msec
A
  1. Answer: A

Source: Goodwin J, Board Review 2005

463
Q
  1. Which of the following factors are capable of inducing
    pain in visceral structures?
  2. Abnormal distention and contraction of hollow visceral
    structures
  3. Rapid stretching of the capsule of solid visceral organs
  4. Ischemia of visceral musculature
  5. Traction or compression of ligaments, vessels, or mesentery
A
  1. Answer: E (ALL)
    Explanation:
    Reference: Raj and Patt. Chapter 11. Visceral Pain. In:
    Pain Medicine: A
    Comprehensive Review, 2nd Edition, Raj, Mosby, 2003,
    page 95
    Source: Day MR, Board Review 2005
464
Q
  1. The MMPI is a psychological test that assesses:
  2. Pain tolerance
  3. Beliefs and attitudes about pain
  4. Psychotic process
  5. Personality
A
  1. Answer: D (4 only)

Source: Janata JW, Board Review 2005

465
Q
  1. Which of the following is/are true regarding intravenous
    Propofol?
  2. Decreased mean arterial pressure
  3. Decreased heart rate
  4. Increased venodilation
  5. Increased systemic vascular resistance
A
  1. Answer: B (1 & 3)

Source: Day MR, Board Review 2005

466
Q

1204.Potential complications with aggressive percutaneous
thermal lesioning of the trigeminal ganglion include the
following:
1. corneal keratitis
2. weakness of ocular abduction
3. diffi culty chewing
4. palsy of the 4th cranial nerve

A
  1. Answer: E (All)
    Explanation:
    (Raj, Pain Review 2nd Ed., page 311)
    Aggressive heat lesions may provide better pain relief, but
    also increase the risk of complete hemifacial anesthesia
    and motor weakness of ipsilateral masticatory muscles
    (pterygoids, temporalis, masseter). Cranial nerves in the
    cavernous sinus (III, IV, VI) may also become injured due
    to excessive heating and cause weakness of some or all
    ocular movements.
  2. It may cause corneal keratitis.
  3. It may cause weakness of muscles of ocular abduction.
  4. It may cause motor weakness of masticatory muscles
    including masseter, temporalis and pterygoids
  5. It may cause paralysis of oculomotor (III), trochlear
    (IV), and trigeminal nerve (VI) by injury in cavernous
    sinus.
    Source: Shah RV, Board Review 2005
467
Q
1205. The defi nition of pyogenic spondylitis includes which of
the following structural fi ndings?
1. Discitis
2. Osteomyelitis
3. Endplate erosions
4. Epidural
A
  1. Answer: A (1,2, & 3)

Source: Bieneman B, Board Review 2005

468
Q
1206. Which of the following is/are a component/s of a
Horners Syndrome?
1. Mydriasis
2. Ptosis
3. Facial anhidrosis
4. Enophthalmos
A
  1. Answer: C (2 & 4)

Source: Day MR, Board Review 2005

469
Q
  1. Which of the following is most likely to cause respiratory
    compromise?
  2. Unilateral percutaneous C1-2 cordotomy in a patient
    with a contralateral pneumonia
  3. Ipsilateral C1-2 cordotomy and contralateral C5-6
    cordotomy
  4. Bilateral C1-2 percutaneous cordotomy
  5. Stereotactic mesencephalectomy with ipsilateral diaphragmatic
    paralysis
A
  1. Answer: B (1 & 3)
    Explanation:
    (Raj, Pain Review 2nd Ed. Page 313, Raj Practical Mgmt of
    Pain 3rd Ed., page 801)
    In patients with diaphragmatic paralysis, pneumonectomy,
    pneumonia, extensive pulmonary carcinoma, contralateral
    high cervical cordotomies or mesencephelectomies can
    reduce ventilatory drive and cause respiratory demise.
    Bilateral high cervical cordotomies can lead to sleepinduced
    apnea (Ondine’s curse). A staged high cervical
    cordotomy or a combination of a high and low cervical
    cordotomy can avoid this problem. Others have assessed
    the absence of the 2-3 fold rise in minute volume to CO2
    as a marker for the development of sleep induced apnea.
    Source: Schultz D, Board Review 2004
470
Q
  1. Regarding NMJ disease:
  2. Lambert-Eaton Myesthenic Syndrome (LEMS) is a presynaptic
    condition
  3. Myesthenia gravis (MG) is a post synaptic phenomenon
  4. MG results in a decremental response to repetitive
    stimulation
  5. LEMS results in an incremental response to repetitive
    stimulation
A
  1. Answer: E

Source: Goodwin J, Board Review 2005

471
Q
  1. Congenital spinal stenosis
  2. May result in neurogenic claudication
  3. Is known as” short pedicle” syndrome
  4. Results in decreased anterioposterior canal narrowing
  5. Often is associated with acquired (degenerative) spinal
    stenosis
A
  1. Answer: E (All)

Source: Bieneman B, Board Review 2005

472
Q
  1. Assumptions underlying cognitive-behavioral therapy
    include:
  2. Cognitions interact with emotions, sensations and
    behavior
  3. Individuals must be active participants in treatment
  4. The interaction between an individual and the environment
    is reciprocal
  5. Behavior is infl uenced by expectations of outcomes and consequences
A
  1. Answer: E (All)

Source: Janata JW, Board Review 2005

473
Q
  1. Which of the following are resistive exercises?
  2. Isometric
  3. Active assisted
  4. Isokinetic
  5. Proprioceptive neuromuscular facilitation
A
  1. Answer: B (1 & 3)
    Explanation:
    (Raj, Practical Mgmt of Pain, 3rd Ed., page 536-538)
    All of the above are forms of therapeutic exercise.
    Therapeutic exercise may be broken down into Range of
    Motion, Resistive, Endurance Activities, Desensitization,
    Breathing Exercises, Relaxation, Coordination Training,
    and Proprioceptive Neuromuscular Facilitation. The goals
    of therapeutic exercise include:
    Strengthening the muscles.
    Improving fl exibility of muscles and tendons.
    Increasing endurance.
    Reinstating the normal pattern of motion to the affected
    muscles and to the body in general.
  2. Isometric Resistive Exercise
    Isometric exercise is a static form of motion performed
    by contraction against an immovable object.
  3. Active Assisted Range-of-Motion Exercises
    Active assisted movement is movement through a ROM
    by means of a muscular contraction supplemented by an
    external force either manually or mechanically
  4. Isokinetic Resistive Exercise
    Isokinetic exercise is a form of dynamic motion in
    which the velocity of muscle shortening or lengthening
    and thus the velocity of the body part is controlled by a
    rate-limiting device
  5. Proprioceptive Neuromuscular Facilitation
    In general, proprioceptive neuromuscular facilitation
    (PNF) is used to promote or hasten the response
    of the neuromuscular mechanism of the proprioceptors.
    It employs total patterns of movement, specifi c patterns of
    facilitation, and techniques for expediting motor learning
    Source: Shah RV, Board Review 2005
474
Q
  1. Secondary gain can include:
  2. Financial compensation
  3. Responsibility avoidance
  4. Reinforcement from family
  5. Reinforcement from friends
A
  1. Answer: E (All)

Source: Janata JW, Board Review 2005

475
Q
  1. Transcutaneous electrical stimulation:
  2. Is based on the gate control theory of pain
  3. Mechanistically activates large diameter afferent fi bers,
    in order to suppress afferent small fi ber input into the
    spinal cord
  4. High intensity, low frequency stimulation is thought to
    work via a naloxone reversible mechanism
  5. Low frequency, high pulse duration cause strong muscle
    contractions
A
  1. Answer: E (All)
    Explanation:
    (Raj, Practical Mgmt of Pain, 3rd Ed., pages 534-535)
    1.The gate control theory explains the mechanisms of pain
    relief associated with TENS treatment for many
    conditions.
  2. Simply stated, this theory proposed the existence of a
    gating mechanism in the dorsal horns of the spinal cord,
    where there is an interaction between the small-diameter,
    unmyelinated C fi bers, which mediate the transmission of
    pain, and the larger-diameter, myelinated A fi bers, which
    mediate sensation of light touch and pressure.
  3. High-intensity, low-frequency stimulation (frequently
    referred to as “acupuncture-like TENS”) also appears to
    offer pain relief, the effects of which can be reversed with
    naloxone, an opiate antagonis
  4. Central to the discussion of the rationale of TENS
    therapy are its various stimulation parameters. Lowfrequency
    and high-pulse [width] energy cause strong,
    rhythmic muscle contractions.
    Source: Shah RV, Board Review 2005
476
Q
1214.There is increased risk of depression in chronic pain
secondary to all of the following :
1. The aversive nature of pain
2. Sense of loss of control
3. Disrupted sleep patterns
4. Disability income
A
  1. Answer: A ( 1, 2, & 3)

Source: Janata JW, Board Review 2005

477
Q
  1. Chronic pain syndrome includes all of the following:
  2. Disrupted activity levels
  3. Minor to moderate pathophysiology
  4. Excessive reliance on medications
  5. Minor to moderate pain complaining
A
  1. Answer: A ( 1, 2, & 3)

Source: Janata JW, Board Review 2005

478
Q
1216. Interventional pain management techniques used to
treat pelvic pain include:
1. Splanchnic Nerve Block
2. Hypogastric Plexus Block
3. Celiac Plexus Block
4. Ganglion Impar Block
A
  1. Answer: C (2 and 4)
    Explanation:
    Reference: Raj, Chapter 17. Visceral Pain. In: Practical
    Management of Pain. 3rd Edition. Raj et al, Mosby, 2000,
    page 236.
    Source: Day MR, Board Review 2005
479
Q
  1. Which of the following statements are true regarding
    atypical facial pain?
  2. The pain is constant, non-paroxysmal and poorly localized,
    even if generally unilateral.
  3. It is usually resistant to uni-modal approaches to care
    whether medical, surgical or behavioral.
  4. Patients appear less distressed than one might expect
    from the descriptors of pain which include ‘crushing’
    or ‘ripping’
  5. Depression is believed to be the underlying cause
A
  1. Answer: A

Source: Goodwin J, Board Review 2005

480
Q
  1. Contact thermography
  2. Is more reliable than infrared thermography
  3. It best performed at normal room temperatures
  4. Can picture the entire body
  5. Is less expensive than infrared thermography
A
  1. Answer: D (4 Only)
    Source: Raj P, Pain medicine - A comprehensive Review -
    Second Edition
481
Q
  1. Spinal cord stimulation is most effective in:
  2. Brachial plexopathy
  3. Phantom limb pain
  4. Post herpetic neuralgia
  5. Acute lumbar radiculopathy
A
  1. Answer: A (1,2, & 3)
482
Q
  1. During which of the following upper extremity motions
    is scapular posterior tilting most prevalent?
  2. Abduction elevation
  3. Extension elevation
  4. Internal rotation
  5. Flexion elevation
A
  1. Answer: D (4 Only)

Source: Sizer Et Al - Pain Practice March & June 2003

483
Q
  1. A 42-year-old female patient presents with an 12 month
    history of elbow pain on the lateral aspect, resulting from
    repetitive strain at work. Symptoms were worse at night.
    Physical examination revealed mild provocation with
    resisted dorsal extension, but signifi cant provocation
    with resisted forearm supination.
  2. Tendopathy of the extensor carpi radialis brevis
  3. Tendopathy of the fl exor carpi ulnaris
  4. Humeroradial joint chondropathy
  5. Posterior interosseus nerve entrapement
A
  1. Answer: D (4 Only)

Source: Sizer et al - Pain Practice - March & June 2004

484
Q
  1. Twenty years ago, a patient was informed by her dentist
    that she was allergic to local anesthetics. True statements
    include:
  2. The local anesthetic solution most likely contained
    methylparaben
  3. The antigenic local anesthetic was most likely an amide
  4. Skin testing is unreliable in confi rming the diagnosis
  5. Enzyme-linked immunosorbent assay (ELISA) will confi
    rm the diagnosis
A
  1. Answer: D (4 Only)
    Source: American Board of Anesthesilogy, In-trainnig
    examination
485
Q
  1. Landmarks for stellate ganglion block include the
  2. Cricoid cartilage
  3. Mastoid process
  4. Transverse process of C6
  5. Hyoid bone
A
  1. Answer: D (4 Only)
    Source: American Board of Anesthesilogy, In-trainnig
    examination
486
Q
  1. What are the advantages of performing a stellate
    ganglion block at C7 compared to C6?
  2. Easier to identify anatomic landmarks by palpation
  3. Increased risk of recurrent laryngeal nerve palsy
  4. Decreased risk of pneumothorax
  5. Reduced volume of local anesthetic is needed
A
  1. Answer: D (4 Only)
    Explanation:
    (Raj, Pain Review 2nd Ed.)
    Since the stellate ganglion is located at C7-T1, a C7
    approach requires less volume. The risk of recurrent
    laryngeal nerve palsy is less at C7. There is an increased
    risk of pneumothorax due to the dome of the lung. The
    landmarks at C6 are easier to identify by palpation. The C6
    transverse process is easier to identify than C7; the C6
    transverse process, specifi cally the tubercle, is known as
    Chassaignac’s tubercle. There is an increased risk of
    pneumothorax at C7 as is the case for a supraclavicular
    approach to the brachial plexus.
    Source: Shah RV, Board Review 2003
487
Q
  1. Each of the following items describes pain in the
    abdominal viscera:
  2. Pain is transmitted via the vagus nerve
  3. The nerve fi bers are type C versus A-delta
  4. Pain is in a dermatomal distribution
  5. Pain is characterized by a dull aching or burning sensation
A
  1. Answer: C (2 & 4)
    Explanation:
    1, 2. Virtually all pain arising in the thoracic or abdominal
    viscera is transmitted via the sympathetic nervous system
    in unmyelinated type C fi bers.
  2. Visceral pain is caused by any stimulus that excites
    nociceptive nerve endings in diffuse areas.
    Distention of a hollow viscus causes a greater sensation
    of pain than does the highly localized damage produced by
    transecting the gut.
  3. Visceral pain is dull, aching, burning, and non-specifi c.
    Source: Hall and Chantigan
488
Q
1226. Which of the following blocks can be performed by both
intra-oral and extra-oral approach?
1. Sphenopalatine ganglion block
2. Glossopharyngeal nerve block
3. Infra-orbital nerve block
4. Greater palatine nerve block
A
  1. Answer: A (1, 2, & 3 )
    Explanation:
    (Raj, Pain Medicine Review 2nd Ed., ) The sphenopalatine
    ganglion and greater palatine nerve block can be
    performed intra-orally through the greater palatine
    foramen (on the hard palate). The greater palatine nerve
    block, however, can only be performed intra-orally. A
    glossopharyngeal nerve block can be performed extra- or
    intra-orally. The infra-orbital nerve block can be
    performed extra-orally or intra-orally
    Source: Shah RV, Board Review 2003
489
Q
  1. A patient with history of chronic low back pain of several
    years starts complaining of lower extremity pain with
    radiation into lateral foot. Examination showed loss of
    refl ex of Achilles tendon with reduced sensation on the
    lateral foot with weakness on foot eversion. The true
    statements with reference to EMG fi ndings with lumbar
    spine evaluation are as follows:
  2. S1 nerve root involvement shows fi brillation or sharp
    waves in peroneus longus and brevis
  3. L4 nerve root involvement shows fi brillation or sharp
    waves in tibialis anterior
  4. L5 nerve root involvement shows fi brillation or sharp
    waves in extensor hallucis longus
  5. S1 nerve root involvement shows fi brillation or sharp
    waves in extensor hallucis longus
A
  1. Answer: A (1, 2, & 3)
    Source: Hoppenfeld S. Orthopaedic Neurology. A
    Diagnostic Guide to Neurologic Levels. Philadelphia,
    LWW, 1997.
490
Q
  1. Which of the following is correct about the Global
    Assessment of Function (GAF) indicated on Axis V?
  2. GAF scores, like VAS scores, are reported as whole numbers
    between 0 and 10.
  3. Lower GAF scores correlate with lower levels of daily
    functioning for patients.
  4. GAF scores are objectively determined through the
    Mental Status Examination.
  5. GAF scores involved severity of symptoms and level of
    functioning
A
  1. Answer: C (2 & 4)

Source: Cole EB, Board Review 2003

491
Q
  1. The Sternocleidomastoid (SCM) muscle can cause:
  2. Headaches
  3. Hearing loss
  4. Vertigo
  5. Nystagmus
A
  1. Answer: A (1, 2, & 3 )
    Explanation:
    SCM pathology can cause headaches, vertigo, ear pain,
    hearing loss, but not nystagmus.
    Travel J, Simmons D. The Trigger Point Manual
    Source: Trescot AM, Board Review 2003
492
Q
  1. The following are the methods of achieving hypnotic
    pain control
  2. Alter the perception of pain
  3. Substitute the painful sensation with a different or less
    painful sensation
  4. Move the pain to another area of the body
  5. Distortion of time
A
  1. Answer: E (All)
    Source: Raj P, Pain medicine - A comprehensive Review -
    Second Edition
493
Q

1231.Which of the following statements is (are) true
concerning thermogrpahy?
1. It is useful in localizing trigger points in myofascial pain
syndrome
2. It uses infrared radiation from the body for diagnostic
purposes
3. It is useful for revealing dysfunction in microcirculation
4. It is usually associated with abnormal laboratory studies

A
  1. Answer: C (2 & 4)
    Source: Raj P, Pain medicine - A comprehensive Review -
    Second Edition
494
Q
  1. Which of the following is (are) the most common
    causative organism(s) implicated in the genesis of pelvic
    infl ammatory disease?
  2. Neisseria gonorrhoeae
  3. Staphylococcus epidermitis
  4. Chlamydia trachomatis
  5. Herpes simplex virus
A
  1. Answer: B (1 & 3)
    Source: Raj P, Pain medicine - A comprehensive Review -
    Second Edition
495
Q
1233. Potential complications associated with an ophthalmic
nerve block include:
1. Orbital Perforation
2. Bradycardia
3. Ptosis
4. Miosis
A
  1. Answer: A (1, 2, & 3 )
    Explanation:
    (Raj, Pain Review 2nd Ed.,)
    The opthalmic nerve block or the retrobulbar block is
    indicated for intraocular surgery and rarely, refractory eye
    pain.
    The goal of the technique is to block the ophthalmic
    branch of V1, the oculomotor nerve, abducens nerve, and
    trochlear nerve. The ciliary ganglion (located in the
    intracone area) will also be blocked.
    If this latter parasympathetic output is blocked, then one
    will see papillary dilatation (mydriasis) and not
    contraction (miosis).
    Cardiac arrhythmias have been reported with this block
    including the oculo-bradycardic refl ex. Globe perforation
    is possible with needle entry. Lid ptosis occurs since the
    oculomotor nerve supplies the superior levator palpebra.
    Recall that the sympathetics supply the superior tarsal muscle (elevates the eyelid) and may be
    spared…nonetheless, the block of the oculomotor will
    cause ptosis
    Source: Shah RV, Board Review 2003
496
Q
  1. Which of the following is true about performing a
    ‘carpal tunnel’ injection?
  2. It is indicated for analgesia of the 5th digit
  3. The target nerve lies medial to the palmaris longus
    tendon
  4. The injection is superfi cial to the fl exor retinaculum
  5. The target nerve lies medial the fl exor carpi radialis
    tendon
A
  1. Answer: D (4 Only)
    Explanation:
    (Raj, Pain Review 2nd Ed.)
    The median nerve is the target in carpal tunnel injections
    and is the compressed nerve in carpal tunnel syndrome.
    However, the nerve (with the hands in supinated anatomic
    position) is located LATERAL to the palmaris longus and
    MEDIAL to the fl exor carpi radialis. It is not indicated for
    analgesia of the 5th digit. The median nerve that traverses
    the carpal tunnel provides sensory innervation to the
    radial 3 ½ digits on the palmar side, but distally on the
    radial side. The palmar branch of the median nerve does
    not go through the carpal tunnel and innervates the radial
    palm. The roof of the carpal tunnel is formed by the fl exor
    retinaculum; hence the injection should be deep and not
    superfi cial to this structure.
    Source: Shah RV, Board Review 2003
497
Q
  1. Thermography can be used for the following purposes
    :
  2. Documents the locations of myofascial trigger points
  3. Evaluate sympathetic blockade after stellate ganglion
    block
  4. Support the diagnosis of refl ex sympathetic dystrophy
  5. Prove the presence of psyschogenic pain syndromes
A
  1. Answer: A (1, 2, & 3)
    Explanation:
    In clinical practice, thermography is useful only as a
    means to measure skin temperature over a wide body area.
  2. Thermography can also document locations of
    myofascial trigger points.
  3. It has also been used to evaluate the degree of
    sympathetic blockade after stellate ganglion, lumbar
    sympathetic, or epidural blocks.
  4. It has been used to diagnose refl ex sympathetic
    dystrophy, entrapment neuropsthies, spinal nerve root
    irritation, vascular disease, joint disease, and fractures.
  5. Psychogenic pain syndrome is not proven by
    thermography.
    Source: Ramamurthy
498
Q

1236.Which of the following electrodiagnostic studies is
typically used to assess radicular pain involving the spine
and related extremities?
1. Selective tissue conductance tests
2. Nerve conduction velocity studies
3. Somatosensory evoked potentials
4. Needle EMG recordings

A
  1. Answer: C (2 & 4)
    Source: Raj P, Pain medicine - A comprehensive Review -
    Second Edition
499
Q
  1. If you use the inion as a point of reference and march
    anteriorly, you will encounter several nerves innervating
    the cranium. Which of the following sequences would be
    correct?
  2. Greater occipital nerve, Least occipital nerve, Lesser occipital
    nerve, Greater auricular nerve
  3. Lesser occipital nerve, Greater occipital nerve, Least occipital
    nerve, Auriculotemporal nerve
  4. Least occipital nerve, Lesser occipital nerve, Greater
    auricular nerve, Auriculotemporal nerve
  5. Greater occipital nerve, Least Occipital Nerve, Auricolotemporal,
    Greater auricular nerve
A
  1. Answer: B (1 & 3)
    Explanation:
    (Raj, Pain Review 2nd Ed.)
    The correct order is greater occipital nerve (C2), least
    occipital nerve (C3), lesser occipital (C2-3), greater
    auricular nerve (C2-3), auriculotemporal (V3),
    supraorbital (V1), and supratrochlear(V1).
    Source: Shah RV, Board Review 2003
500
Q
  1. The true statements with regards to EMG fi ndings with
    cervical nerve root involvement.
  2. With C7 nerve root irritation, fi brillation or sharp waves
    are detected in triceps
  3. With C8 involvement, fi brillation or sharp waves are
    detected in intrinsic hand muscles
  4. With C6 involvement, fi brillation or sharp waves are
    detected in biceps
  5. With C5 involvement, fi brillation or sharp waves are
    detected in biceps and triceps
A
  1. Answer: A (1, 2, & 3)
    Explanation:
    1, 2, 3. With C7 nerve root irritation, fi brillation or sharp
    waves are detected in triceps. With C8 involvement,
    fi brillation or sharp waves are detected in intrinsic hand
    muscles. With C6 involvement, fi brillation or sharp
    waves are detected in biceps
  2. With C5 involvement, fi brillation or sharp waves are
    detected in deltoid and biceps.
    Source: Hoppenfeld S. Orthopaedic Neurology. A
    Diagnostic Guide to Neurologic Levels. Philadelphia,
    LWW, 1997.
501
Q
  1. An ulnar nerve injury would most likely produce:
  2. Numbness of part of the 4th and all of the 5th digit of
    the hand
  3. Claw hand deformity
  4. Weakened fl exion of the wrist
  5. Numbness of thumb
A
  1. Answer: A ( 1, 2, & 3)

Source: Wirght PD, Board Review 2004

502
Q
1240. Which of the following may cause referred pain to the
inguinal/thigh region?
1. femur fracture
2. osteonecrosis of the femoral head
3. inguinal hernia
4. Dermatomyositis
A
1240. Answer: E (All)
Explanation:
(Raj, Practical Mgmt of Pain 3rd Ed., Box 24-3)
COMMON SOURCES OF INGUINAL OR THIGH PAIN
FRACTURE TO FEMUR
Traumatic
Pathological
Stress
MUSCLE
Strain
Fever-related myalgias
Dermatomyositis
Polymyositis
VASCULAR
Sickle cell crisis
Iliofemoral venous thrombosis
Avascular necrosis of femoral head
REFERRED
Inguinal or femoral hernia
Inguinal or femoral lymphadenitis
Degenerative arthritis of the hip joint (severe)
Source: Shah RV, Board Review 2004
503
Q
  1. The treatment of trochanteric bursitis or gluteal fasciitis
    includes the following:
  2. Non-steroidal anti-infl ammatory drugs
  3. Physical therapy and exercise program
  4. Local corticosteroid injection
  5. Correction of mechanical abnormality
A
  1. Answer: E (All)

Source: Cole & Herring. Low Back Pain Handbook.

504
Q
  1. Regarding Axis I of the DSM-IV-TR Multiaxial diagnostic
    methodology, which of the following is correct?
  2. Use Axis I for reporting all signifi cant personality disorders.
  3. Medical disorders should be reported on Axis I, but the
    principal psychiatric diagnosis should be listed fi rst.
  4. When no Axis I disorder is present, note the Global Assessment
    of Function (GAF) as “>100.”
  5. The principal diagnosis or reason for the visit will be assumed
    on Axis I unless the Axis II diagnosis is followed
    by a qualifying phrase (“reason for visit”).
A
  1. Answer: D (4 Only)

Source: Cole EB, Board Review 2003

505
Q
  1. Each of the following is associated with an increased
    incidence of headache after spinal anesthesia:
  2. Young age
  3. Female gender
  4. Pregnancy
  5. Large needle size
A
  1. Answer: E (All)
    Explanation:
    Other factors that affect the incidence of spinal headache
    include the number of dural punctures and the position of
    the needle bevel.
    The incidence of spinal headache increases as the number of dural punctures increases.
    The incidence of headache has been shown to be less when
    the dural fi bers are split longitudinally rather than when
    they are cut while the needle is held in a transverse
    direction.
    The timing of ambulation relative to dural puncture has
    not been shown to affect the incidence of postspinal
    headache.
506
Q
  1. A 42-year-old male patient presents with anterior knee
    pain. Pain started at work. Physical examination showed
    positive resisted knee extension. The conditions included
    in differential diagnosis include the following:
  2. Meniscal anterior horn lesion
  3. Prepatellar bursitis
  4. Patellofemoral joint pathology
  5. Infrapatellar bursitis
A
  1. Answer: B (1 & 3)

Source: Sizer et al - Pain Practice - March & June 2004

507
Q
  1. Which is true of a superior hypogastric plexus?
  2. When blocking the SHP, the needle ideally should be
    anterior to the L5-S1 disc
  3. It is composed of parasympathetic and sympathetic
    fi bers
  4. It is indicated for pelvic malignancy and chronic interstitial
    cystitis
  5. It receives branches of the sacral nerves
A
  1. Answer: B (1 & 3)
    Explanation:
    (Raj, Pain Review, 2nd Ed., page 267-269)
    The SHP block is indicated for chronic pains in the pelvis.
    It is composed of sympathetic fi bers, unlike the inferior
    hypogastric plexus. The IHP receives parasympathetic
    branches of the sacral nerve: nervi erigentes. The ideal
    placement should have the needle anterior to the L5-S1
    disc, with dye spread inferiorly along the sacral
    promontory. Needle entry is initially at L4-5, but this
    procedure can be done transdiscally through L5-S1.
    Source: Shah RV, Board Review 2003
508
Q
  1. Which of the following complications may be attributable to unrelieved constipation?
  2. Chronic abdominal Pain
  3. Headache
  4. Back Pain
  5. Lower extremity weakness
A
  1. Answer: A (1, 2, & 3)

Source: Reddy Etal. Pain Practice: Dec 2001, march 2002

509
Q
  1. Somatization Disorder is best characterized by which of
    the following statements?
  2. It generally develops in early life.
  3. Pain is rarely described by these patients.
  4. Physical complaints are in excess of what would be expected
    based upon the history, physical examinations
    and laboratory studies.
  5. Symptoms are intentionally produced or feigned.
A
  1. Answer: B (1 & 3)

Source: Cole EB, Board Review 2003

510
Q
1248. Sacral nerve root stimulation for rectal pain is achieved
by:
1. bilateral stim-cath to S2 nerves
2. bilateral stim-cath to S3 nerves
3. bilateral S5 nerves with stim-cath
4. bilateral S4 nerves with stim-cath
A
  1. Answer: D (4 Only)

Source: Racz G. Board Review 2003

511
Q
  1. With seizures following lidocaine injection,
    hyperventilation with 100% O2 is recommonded. The
    rationale for this therapy is to
  2. Decrease delivery of lidocaine to the brain
  3. Prevent hypoxia
  4. Hyperpolarize the nerve membranes
  5. Convert of lidocaine to the protonated (ionized) form
A
  1. Answer: A (1, 2, & 3)
    Explanation:
  2. Hyperventilation causes cerebral vasoconstriction and
    decreased delivery of local anesthetic to the brain.
  3. Administration of 100% O2, during a seizure helps to
    prevent hypoxia in a patient who otherwise might not be
    breathing.
  4. Hyperventilation includes hypokalemia and respiratory
    alkalosis, both of which result in hyperpolarization of
    nerve membranes and elevation of the seizure threshold.
  5. Hyperventilation also raises the patient’s pH
    (respiratory alkalosis) and converts lidocaine into the
    nonionized (nonprotonated) from, which crosses the
    membrane easily. This has no beneficial effect.
512
Q
  1. The multiaxial distinction among Axis I, II and III
    disorders implies theimportance given to which of the
    following statements?
  2. Mental disorders are related to physical or biological
    factors, or that general medical conditions are related to
    behavioral or psychosocial factors.
  3. General medical conditions are rarely the direct etiological
    causes for the development or worsening of
    mental symptoms.
  4. Enhanced communication among healthcare providers
    of different specialties is essential for the care of
    patients with pain.
  5. Axis I disorders are not psychological reactions to an
    Axis III general medical conditions.
A
  1. Answer: B (1 & 3)

Source: Cole EB, Board Review 2003

513
Q

1251.The incidence of postdural puncture headache is
increased in which of the following situations?
1. Pregnancy
2. Young age
3. Use of large-bore spinal needle
4. Use of paramedian instead of midline approach

A
  1. Answer: A (1, 2, & 3)
    Explanation:
    1, 2. Patients who are at increased risk of headache after
    dural puncture include parturients and young patients.
  2. Use of large-bore needles and glucose-containing local
    anesthetics also can raise the risk of spinal headache.
    Spinal headaches result from leakage of CSF through the
    dural sheath.
    The headache is typically frontal or occipital in location
    and is worsened by sitting or standing up.
  3. There is some evidence that the incidence of spinal
    headache is less after a dural puncture made through the
    paramedian approach.
    Source: Hall and Chantigan
514
Q
  1. A patient with constant supraorbital pain comes into
    your offi ce. Possible etiologies include:
  2. Frontal sinusitis
  3. SCM pathology
  4. Supraorbital neuralgia
  5. Spinal accessory neuralgia
A
  1. Answer: E (All)
    Explanation:
    Frontal sinusitis, SCM spasms, and supraorbital neuralgia
    all can cause supraorbital pain. The spinal accessory nerve
    innervates the SCM and therefore could cause SCM
    pathology.
    Source: Trescot AM, Board Review 2003
515
Q
  1. Which of the following are advantages of intravenous
    PCA over conventional IM therapy?
  2. Decreased work load for health care personnel
  3. Avoids excess drowsiness
  4. Rapid pain relief
  5. Equipment cost
A
  1. Answer: A (1, 2, & 3 )
    Explanation:
    Ref: Rosenberg, Porter, Lupatkin. Chapter 11. Patientcontrolled
    Analgesia. In: Pain Management and Regional
    Anesthesia in Trauma. Rosenberg, Grande, Bernstein. W.B.
    Saunders, 2000, page 165.
    Source: Day MR, Board Review 2003
516
Q
  1. Treating patients with painful conditions and underlying
    Personality Disorders is made complicated because of
    which of the following?
  2. The more bizarre patients appear to be the more likely
    they are to be assaultive.
  3. There are no objective means to confirm compliance
    with treatment provided.
  4. Insurance companies rarely provide defined benefits for
    “dual diagnosed” patients.
  5. Borderline personality disorders are prone to exaggerated
    complaints, inappropriate attachments and impulsivity
    making their care very difficult.
A
  1. Answer: D (4 Only)

Source: Cole EB, Board Review 2003

517
Q
  1. Constipation can be lead to?
  2. Nausea
  3. Increased Pain
  4. Delirium
  5. Hypercalcemia
A
  1. Answer: A (1, 2, & 3)

Source: Reddy Etal. Pain Practice: Dec 2001, march 2002

518
Q
1256. A patient on tramadol (Ultram) and sertraline (Zoloft)
develops confusion. Your diagnosis is:
1. Drug abuse
2. Drug withdrawal
3. Cardiac side effects
4. Serotonin syndrome
A
  1. Answer: D (4 Only)
    Explanation:
    Serotonin syndrome is a toxic hyperserotonergic state that
    develops soon after initiation or dosage increments of the
    offending agent. Patients may differ in their susceptibility
    to the development of serotonin syndrome. The (+)
    enantiomer of tramadol inhibits serotonin uptake.
    Tramadol is metabolized to an active metabolite, M1, by
    the CYP2D6 enzyme. If this metabolite has less
    serotonergic activity than tramadol, inhibition of CYP2D6 by sertraline may cause increased levels of serotonin in the
    synaptic cleft.
519
Q

1257.True statement/s regarding peripheral nerve blocks
is/are:
1. Frequently used as a component of multimodal analgesia
2. Interrupts the transmission component of the nociceptive
process
3. Provide more selective anesthesia and analgesia than
central neural blockade techniques, i.e. subarachinoid
or epidural neural blockade
4. Femoral nerve block (3-in-1 block) is effective for anesthesia
and analgesia of the lower leg.

A
  1. Answer: A (1, 2, & 3 )
    Explanation:
    Ref: Crews. Chapter 14. Acute Pain Syndromes. In:
    Practical Management of Pain. 3rd Edition, Raj et al,
    Mosby, 2000, page 177.
    Source: Day MR, Board Review 2003
520
Q
  1. The anatomic features contributing to the development
    of carpal tunnel syndrome, include:
  2. Abnormalities of the hamate hook
  3. Capitate exostosis
  4. Size and proximal insertion of the lumbricle muscles
  5. Extensor digitorum communis tendon hypertrophy
A
  1. Answer: A (1, 2, & 3)

Source: Sizer et al - Pain Practice - March & June 2004

521
Q
  1. A young woman in her forties has a 20-year history of
    Crohn’s disease, presents with the acute onset of right
    ankle and left knee pain. She recalls a worsening of her
    gastrointestinal symptoms a few days before the joint
    symptoms developed. Radiographs of the knee and ankle
    demonstrate soft tissue swelling and small effusions but
    no bone destruction. The true statements include:
  2. The patient is HLA- B27 positive
  3. The patient is experiencing the most common extraintestinal
    manifestation of infl ammatory bowel disease
  4. Controlling the intestinal symptoms will eliminate the
    knee and ankle arthritis
  5. The patient will go on to develop bone erosion and destruction
    of the knee and ankle
A
  1. Answer: B (1 & 3)
    Explanation:
    (Tierney, 42/e, pp 825-829)
    · HLA-B27 diseases are easy to remember with the
    mnemonic PAIR (Psoriasis, Ankylosing spondylitis,
    Infl ammatory bowel disease, and Reiter syndrome). These
    are called the seronegative spodylarthropathies. Reiter
    syndrome preceded by a bacterial infection (Yersinia,
    Salmonella, or gonococcus) has a high association with a
    positive HLA-B27. Ankylosing spondylitis has a 90%
    association with HLA-B27; overall, Reiter syndrome and
    infl ammatory bowel disease (IBD) have an 80% HLA-B27
    association.
    Patients with IBD (Crohn’s disease and ulcerative colitis)
    may dev op a nonerosive oligoarthritis of the large
    peripheral joints that is usually eliminated after
    controlling the gastrointestinal symptoms. Arthritis is the
    second most common extraintestinal manifestation in
    patients the IBD (anemia is the most common
    extraintestinal manifestation). NSAIDs must be used with
    caution in patients with IBD.
522
Q
  1. After a cervical plexus block, it is noted that the patient
    is unable to elevate the shoulder. Following nerves were
    blocked during the cervical plexus block
  2. Thoracodorsal
  3. Anterior cervical
  4. Supraclavicular
  5. Accessory
A
  1. Answer: C (2 & 4)
    Explanation:
    Blockade of the accessory nerve (CN XI) is useful for
    trapezius muscle block as an adjunct to interscalene nerve
    blocks of the brachial plexus for surgery on the shoulder.
    The accessory nerve traverses the posterior triangle of
    the neck in a very superfi cial location. It emerges from the
    body of the sternocleidomastoid muscle at the junction of
    the superior and middle third of the posterior border of
    the muscle and therefore is frequently unintentionally
    blocked when a superfi cial cervical plexus block is
    performed.
    Source: Kahn and Desio
523
Q
  1. Which of the following statements about cachexia in the
    cancer patient is NOT true?
  2. Cachexia in cancer patients may be managed by increasing
    caloric intake.
  3. Cachexia is found in a majority of cancer patients and
    is a major contributing factor of death in 50% of these
    patients.
  4. The main cause of cathexia in the cancer patient is depression
    leading to food aversion and apathy.
  5. Corticosteroids may stimulate the appetite and decrease
    nausea in these patients.
A
  1. Answer: B (1 & 3)

Source: Reddy Etal. Pain Practice: Dec 2001, march 2002

524
Q
  1. The initial treatment for leg pain associated with sickle cell crisis should include:
  2. NSAIDs
  3. Opioids
  4. Hydration
  5. Hydroxyurea
A
  1. Answer: A (1,2, & 3)
    Explanation:
    Hydroxyurea stimulates fetal hemoglobin synthesis and
    will not provide acute relief of pain.
525
Q
  1. Post lumbar puncture headaches
  2. Usually occur immediately following dural puncture
  3. Are relieved 8 to 12 hours after an epidural blood patch
    is performed
  4. Occur more frequently in nonpregnant compared with
    pregnant patients
  5. Can be associated with neurologic defi cits
A
  1. Answer: D (4 Only)
    Explanation:
    Postspinal headaches are characterized by frontal or
    occipital pain, which worsens with sitting and improves
    with reclining.
    The etiology of postspinal headaches is unclear;
    however, they are believed to be caused by a
    reduction in CSF pressure and resulting tension on
    meningeal vessels and nerves (which result
    from leakage of CSF through the needle hole in the dura
    mater).
  2. Usually occurs 24-48 hrs after lumbar puncture.
  3. Conservative therapy for a postspinal headache include
    bed rest, analgesics, and oral and intravenous hydration.
    If conservative therapy is not successful after 24 to 48
    hours, it is recommended that an epidural
    “blood patch” with 10 to 20 mL of the patient’s blood be
    performed. An epidural “blood patch” provides prompt
    relief of the postspinal headache.
  4. Factors associated with an increased incidence of
    postspinal headaches include pregnancy, size and type of
    needle used to perform the block, age of the patient, the
    number of dural punctures.
  5. Postspinal headaches may be associated with neurologic
    symptoms such as diplopia, tinnitus, and reduced hearing
    acuity.
    Source: Hall and Chantigan
526
Q
  1. Dyspnea, a common symptom in patients with advanced cancer may be caused by:
  2. Pleurl effusion
  3. Psychological distress
  4. Pneumonia
  5. Intracranial metastases
A
  1. Answer: A (1, 2, & 3)

Source: Reddy Etal. Pain Practice: Dec 2001, march 2002

527
Q
1265. Which of the following is/are signs and symptoms of a
myelopathy?
1. Positive Babinski
2. Positive Hoffman’s sign
3. Clonus
4. Hyporefl exia
A
  1. Answer: A (1, 2, & 3 )
    Explanation:
    Ref: Simon. Chapter 15. Physical Examination. In: Pain
    Medicine: A Comprehensive Review, 2nd Edition. Raj,
    Mosby, 2003, page 132.
    Source: Day MR, Board Review 2003
528
Q
  1. Appropriate indications for intrathecal opioids include:
  2. Post laminectomy syndrome
  3. CRPS
  4. Cancer pain
  5. Spasticity from spinal cord injury
A
  1. Answer: A ( 1, 2, & 3)
    Explanation:
    Post laminectomy syndrome pain and cancer pain are well
    recognized indications for intrathecal opioids. CRPS is
    considered an indication if the trial gives good relief.
    Although MSO4 intrathecally may decrease some of the
    spasticity, intrathecal baclofen is probably more
    appropriate
    Source: Trescot AM, Board Review 2004
529
Q
  1. The true statements regarding management of low back
    pain secondary to spondylosis or spondylolisthesis are as
    follows:
  2. Physical therapy initially using slight fl exion bias with
    neutral spine position
  3. Flexibility training program to improve hamstrings
  4. Strength training to help to maintain segmental spinal
    mechanics
  5. Spine extension program
A
  1. Answer: A (1, 2, & 3)
    Explanation:
    Recommended management is as follows:
    1.Bracing based on isthemic spondylosis in adolescent
    2.Medication
    3.Physical therapy
    A.Education
    B.Modalities to control pain and muscle spasm
    i.fl exibility training program
    Initially use slight fl exion bias with neutral spine position
    because this position decreases stress on the posterior
    elements and may help to decrease pain, particularly
    hamstrings
    4.Strength training
    A.Initially use a slight fl exion bias with neutral spine
    positron because this position decreases stress on the
    posterior elements and may help to decrease pain
    B.Helps to maintain
    i.segmental spinal mechanics and lower extremity kinetic
    chain strength balance
    5.Home program
    6.Fluoroscopically guided epidural or transforaminal
    injections for associated discogenic or radicular
    symptoms
    7.Facet joint injections if indicated
    8.Surgery
530
Q
  1. In terminal abdominal cancer pain, celiac plexus
    neurolytic block is:
  2. An accepted procedure
  3. Performed with 50 percent alcohol 25 ml bilaterally
  4. Performed with absolute alcohol 12 ml transaortic
    route
  5. Its effectiveness has been shown to be 100%
A
  1. Answer: A (1, 2, & 3)
    Explanation:
  2. Neurolytic celiac plexus block is an accepted procedure
    in terminal carcinoma of abdomen.
  3. Neurolytic celiac plexus block is performed with 50
    percent alcohol 25 ml bilaterally.
  4. Neurolytic celiac plexus block is performed with
    absolute alcohol 12 ml transaortic route.
  5. Effectiveness has been reported to be as high as 90% in
    some studies.
    Source: Racz G. Board Review 2003
531
Q
1269. Potential complication(s) of a stellate ganglion block
include
1. Recurrent laryngeal nerve paralysis
2. Subarachnoid block
3. Brachial plexus block
4. Pneumothorax
A
  1. Answer: E (All)
532
Q
1270. Sacral nerve root stimulation for coccygodynia is
achieved by:
1. bilateral stim-cath to S2 nerves
2. bilateral S4 nerves with stim-cath
3. bilateral S5 nerves with stim-cath
4. bilateral stim-cath to S3 nerves
A
  1. Answer: D (4 Only)

Source: Racz G. Board Review 2003

533
Q
1271. Spinal cord stimulation has been demonstrated to
produce which of the following changes?
1. Temperature increase
2. Peripheral vasodilation
3. Increased peripheral blood fl ow
4. Blockade of noxious pain sensations
A
  1. Answer: A (1,2, & 3)
    Explanation:
    SCS does not block afferent small fi ber, high threshold,
    nocieptive input.
    Ref: Bonica’s Management of Pain, 3rd edition, page 1860.
534
Q
  1. The true statements regarding conversion disorder are:
  2. An alteration in physical functioning occurs as a consequence
    of psychological conflict.
  3. Limb paralysis and blindness can be symptoms of conversion
    disorder.
  4. Sexual dysfunction is a common conversion symptoms
    encountered clinically.
  5. The patient is conscious of the connection between the
    physical dysfunction and the psychological stress at the
    time it occurs.
A
  1. Answer: A (1, 2, & 3 )
    Explanation:
    * Conversion disorder is the loss or alteration of physical
    functioning that is temporarily associated with
    psychosocial stressor. The patient is not conscious of
    intentionally producing the physical symptom in response
    to the psychic stressor.
    * Paralysis and blindness are often described as classic
    symptoms of conversion disorder.
    * Sexual dysfunction is common.
    * Pain is the least common conversion symptom
    encountered clinically. Patient may not be able to connect.
535
Q
  1. Rehabilitation exercises recommended for lumbar
    spondylolysis and spondylolisthesis include the
    following:
  2. Stretches to reduce impairments of trunk mobility, hip
    fl exors, hamstrings, quadriceps, and calves
  3. Modalities including ultrasound and electrical stimulation
    have been shown to improve symptoms and are
    generally of great value
  4. Improving back and abdominal strength can help decrease the discomfort associated with the lumbar spine
    instability
  5. Exercises to improve back and abdominal strength can
    be very painful and increase lumbar spine instability
A
  1. Answer: B (1 & 3)
536
Q
  1. Which of the following is (are) true of osteochondritis
    (Scheuermann’s Disease)?
  2. Abnormality at junction of vertebral body and disc
  3. Irregularity of ossifi cation and endochondral growth
  4. Thoracic spine involvement in teenagers
  5. Anterior wedging and kyphosis
A
  1. Answer: E (All)

Source: Boswell MV, Board Review 2004

537
Q
  1. MRI fi ndings in carpal tunnel syndrome include
  2. Increased diameter of median nerve proximal to entrapment
  3. Flattening of nerve deep in carpal tunnel
  4. Increased signal on T2 images
  5. Decreased signal on STIR images
A
  1. Answer: A (1,2, & 3)

Source: Bieneman B, Board Review 2005

538
Q
  1. Which of the following are included in the diagnosis of
    Major Depressive Episode?
  2. Diminished interest or pleasure in activities
  3. Negative symptoms such as affective fl attening
  4. Weight loss or weight gain when not dieting
  5. Catatonic or disorganized behavior
A
1276. Answer: B (1 & 3)
Explanation:
Flat affect and disorganized behavior are criteria for
schizophrenia
Source: Boswell MV, Board Review 2004
539
Q
  1. The true statements concerning neurolytic nerve blocks
    include.
  2. There is little difference in the effi cacy between alcohol
    and phenol
  3. Destruction of peripheral nerves can be followed by
    a denervation hypersensitivity that is worse than the
    original pain
  4. Neurolytic blocks should be reserved for patients with
    short life expectancies
  5. Neurolytic blockade with phenol is permanent
A
  1. Answer: A (1, 2, & 3)
    Explanation:
    Alcohol and phenol are similar in their ability to cause
    nonselective damage to neural tissues.
    Neural tissue will regenerate; therefore, neurolytic blocks
    are never “permanent” and neurolysis can lead to a
    denervation hypersensitivity, which can be extremely
    painful.
    Source: Hall and Chantigan.
540
Q
1278. A nerve is undergoing Wallerian degeneration, but has preservation of the epineurium. This nerve injury could
be classified as:
1. Seddon’s axonotmesis
2. Sunderland class 1
3. Sunderland class 4
4. Sunderland class 5
A
  1. Answer: B (1 & 3)
    Explanation:
    Wallerian degeneration occurs following axonal loss
    hence, Sunderland class 1 (conduction block) is false.
    Sunderlan class 5 implies complete nerve transection,
    which is false.
    Other truisms would be Sunderland class 2,3,4—the
    epineurium is intact in all of these. Seddon’s axonotmesis
    would be true, because Wallerian degeneration would
    occur and the epineurium, endoneurium, and
    perineurium are intact.
    Source: Shah RV, Board Review 2004
541
Q
  1. Advantages of intrathecal opioids include:
  2. Absence of sympathetic blockade
  3. Absence of hypotension
  4. Avoidance of cardiovascular effects
  5. Lack of tolerance
A
  1. Answer: A (1, 2, & 3)
    Explanation:
    Intrathecal opioids do not cause sympathetic blockade,
    hypotension, or cardiovascular effects. Intrathecal opioids
    do develop tolerance.
    Source: Trescot AM, Board Review 2004
542
Q
  1. Opioids that are commonly used intrathecally include:
  2. Morphine
  3. Fentanyl
  4. Hydromorphone
  5. Tramadol
A
  1. Answer: A ( 1, 2, & 3)
    Explanation:
    Tramadol is not commonly used in intrathecal pumps.
    Source: Trescot AM, Board Review 2004
543
Q
1281. Which of the following agents is/are useful in treating
cancer-related fatigue?
1. Megestrol acetate
2. Corticosteroids
3. Antidepresssants
4. Methylphenidate
A
  1. Answer: E (All)

Source: Reddy Etal. Pain Practice: Dec 2001, march 2002

544
Q
  1. Cauda equina tumors may present with.
  2. Acute persistent rectal pain
  3. Lower extremity weakness
  4. Patchy sensory loss
  5. Sphincter disturbances
A
  1. Answer: E (All)

Source: Nader and Candido – Pain Practice. June 2001

545
Q
  1. A 36-year old white male presents to your clinic
    with complaints of neck and upper extremity pain. on
    examination, shoulder abduction was relatively weak
    compared to the normal side. There was also weakness
    of elbow fl exion. An MRI of the neck will likely confi rm
    the presence of a disc protrusion at _____ level and
    involvement of ______ nerve root.
  2. Disc protrusion at C4/5
  3. Disc protrusion at C5/6
  4. Neurological level C5
  5. Neurological level C6
A
  1. Answer: B (1 & 3)
    Explanation:
    The deltoid and the biceps are the two most easily tested
    muscles with C5 innervation. The deltoid is almost a pure
    C5 muscle; the biceps is innervated by both C5 and C6,
    and evaluation of its C5 innervation may be slightly
    burred by this overlap.
    The deltoid is by axillary nerve or C5. It is a 3-part
    muscle. The anterior deltoid fl exors, the middle deltoid
    abducts, and the posterior deltoid extends the shoulder; of
    the three motions, the deltoid acts most powerfully on
    abduction. Since the deltoid does not work alone in any
    motion, it may be diffi cult to isolate it for evaluation.
    Therefore, note its relative strength in abduction, its
    strongest plane of motion.
    Primary shoulder abductors:
  2. Deltoid (middle portion)
    C5, C6 axillary nerve
  3. Supraspinatus
    C5, C6 suprascapular nerve
    Secondary shoulder abductors:
  4. Deltoid (anterior and posterior portions)
  5. Serratus (anterior)
    Biceps
    C5, C6, musculoskeletal nerve. The biceps is a fl exor of the
    shoulder and elbow and supinator of the forearm. To
    determine the neurologic integrity of C5, biceps should be
    tested only for elbow fl exion. Since the brachialis muscle,
    the other main fl exor of the elbow, is also innervated by
    C5, testing fl exion of the elbow should give a reasonable
    indication of C5 integrity.
    Source: Hoppenfeld S. Orthopaedic Neurology. A
    Diagnostic Guide to Neurologic Levels. Philadelphia,
    LWW, 1997
546
Q
  1. Which of the following management strategies are
    recommended for a patient with Idiopathic Adhesive
    Capsulitis
  2. Arthroscopic release
  3. Hydraulic distention of the glenohumeral joint
  4. Intra-articular injection
  5. Stretching and resistive ROM exercises
A
  1. Answer: A (1, 2, & 3)

Source: Sizer Et Al - Pain Practice March & June 2003

547
Q
  1. Cancer patients undergoing radiotherapy:
  2. May have multiple pains in addition to the cancer-related
    pain
  3. Could have pain caused by the radiotherapy itself
  4. May develop myelopathy of the spinal cord
  5. May develop acute infl ammation of the nerves or plexuses
A
  1. Answer: E (All)

Source: Nader and Candido – Pain Practice. June 2001

548
Q
1286. Which of the following drug class(es) have NO EFFECT
on acute neuropathic pain?
1. Opioids
2. Tricyclic antidepressants
3. Antiepileptics
4. Benzodiazepines
A
  1. Answer: D (4 Only)

Source: Reddy Etal. Pain Practice: Dec 2001, march 2002

549
Q
  1. Which of the following can be used to perform the
    sweating test, a special test of the function of the
    autonomic nervous system?
  2. Cobalt blue papers
  3. Iodine in oil and starch powder
  4. Ferric chloride and tannic acid
  5. Pilocarpine hydrochloride
A
  1. Answer: E (All)
    Explanation:
    Observation and physical examination of the patient
    provide substantial information about the function of the
    autonomic nervous system.
    Evaluation of endocrine status, body temperature, vital
    signs, skin and mucous membranes, perspiration, hair and
    nail growth, salivation, lacrimation, and extremities, as
    well as documentation of autonomic refl exes involving the
    cranial nerves should be performed prior to special tests of
    autonomic function.
  2. Several tests exist to supplement the information
    obtained on examination of the patient.
    The sweating test will reveal areas of autonomic
    dysfunction.
    Cobalt blue papers will turn pink when exposed to
    moisture and will remain blue in areas of anhidrosis.
  3. Iodine in oil will turn bluish black in the presence of
    starch and moisture.
  4. Ferric chloride turns black in the presence of tannic acid
    and moisture.
  5. Sweating can be elicited by application of external heat,
    ingestion of hot fl uids and aspirin, emotional stimuli,
    intellectual strain, painful cutaneous sensation, or
    subcutaneous injection of 5 mg of pilocarpine
    hydrochloride.
    Other tests of autonomic function include assessment of the pilomotor response, vasomotor response, refl ex
    erythema, histamine fl are, skin temperature, skin
    resistance, capillary microscopy, and plethysmography.
    Source: Raj, P
550
Q
  1. An elderly patient undergoes a lumber sympathetic
    block to improve blood fl ow after frostbite. Findings that
    suggest a successful lumbar sympathetic block include
    the following:
  2. Inability to dorsifl ex foot
  3. Blushing in the toes
  4. Numbness from the knee to the toes
  5. Temperature increase in the legs
A
  1. Answer: C (2 & 4)
    Explanation:
    1, 3. Numbness in the leg and inability to move it suggest
    an accidental subarachnoid or epidural injection, a rare
    but possible complication of this block.
    2, 4. The completeness of a lumber sympathetic block can
    be ascertained by skin temperature measurements and
    increases in blood fl ow. The latter can be determined by a
    number of techniques, including laser Doppler fl owmeter,
    occlusion skin plethysmography, transcutaneous oxygen
    electrodes, and mass spectrometry.
551
Q
  1. Which of the following statements about fatigue in the
    cancer patient is NOT true?
  2. Fatigue refers to a subjective sense of decreased vitality
    in physical or mental functioning.
  3. Symptoms of fatigue may be alleviated by dexamethasone.
  4. Some selective serotonin-reputake inhibitors (SSRIs)
    have been shown to be useful in treating fatigue.
  5. Correcting underlying problems such as depression,
    anxiety, or sleep disturbances is rarely useful in treating
    fatigue
A
  1. Answer: D (4 Only)

Source: Reddy Etal. Pain Practice: Dec 2001, march 2002

552
Q
  1. An elderly man has had many years of deteriorating
    kidney function due to diabetes. Dialysis was begun
    because of electrolyte abnormalities, approximately
    ten years ago. True statements about his pain problems
    include:
  2. The most common neurologic complication of chronic
    renal failure is Seizures and Delirium.
  3. The most common neurologic complication of chronic
    renal failure is Peripheral neuropathy.
  4. His symptoms of restless legs syndrome may be controlled
    with either Haloperidol or Nifedipine
  5. The most reliable treatment for the peripheral neuropathy
    of chronic renal failure is Renal transplant.
A
  1. Answer: C (2 & 4)
    Explanation:
    1, 2. The type of peripheral neuropathy most commonly
    developing with chronic renal failure is a symmetric, distal
    mixed sensorimotor neuropathy. The legs are generally
    affected fi rst and most severely. Men are more commonly
    affected than women. Most of the peripheral neuropathies
    in patients with chronic renal failure involve axonal
    degeneration.
  2. The restless legs syndrome (Ekbom syndrome) is
    characterized by a feeling of discomfort in the legs that is
    relieved by movement.The sensation is felt deep within the
    limb, and is variably describes as pulling, stretching, or
    cramping. Restless legs syndrome occurs primarily at
    night, shortly after the patient lies down. It differs from
    akathisia, which is a restlessness that occurs during the
    daytime. It may be associated with peripheral neuropathy
    and anemia and is seen in patients with chronic renal
    disease, diabetes mellitus, and many other medical
    conditions. Exercise before going to bed may alleviate
    much of the discomfort. Agents that may be effective in
    alleviating symptoms include Clonazepam, gabapentin, Ldopa,
    and opiates. Neuroleptics, calcium channel blockers,
    and caffeine may worsen symptoms.
  3. The neuropathy usually improves with dialysis or
    transplant.
    B vitamins are generally replaced when patients receive
    dialysis. Thiamine is water-soluble and so is easily lost
    during dialysis, but even replacing thiamine is not nearly
    as effective in retarding or reversing the neuropathy of
    chronic renal failure as is renal transplantation. There are
    presumed to be neurotoxins in the blood of patients with
    uremia that are not removed by routine dialysis
    Source: Anschel 2004
553
Q

1291.The presence of positive sharp waves during needle
electromyography of a patient who describes debilitating
pain and weakness of the limb while being tested is
signifi cant because
1. This waveform is only found in patients with muscular
dystrophy and never in pain syndrome
2. This type of activity is an objective sign of denervation
or reinnervation
3. This pattern is an integral component of Waddell’s signs
of nonorganic pain behavior
4. This pattern cannot be created fi ctitiously, even during
reduced voluntary motor effort

A
  1. Answer: C (2 & 4)
    Source: Raj P, Pain medicine - A comprehensive Review -
    Second Edition
554
Q
  1. Indications for prophylactic treatment in migraine are:
  2. Upcoming job interview
  3. Five migraine attacks a month
  4. Tension type headache
  5. Headaches associated with nausea
A
  1. Answer: C (2 & 4)
    Explanation:
    When deciding on the treatment of migraine, one must
    take into consideration the frequency and severity of the
    headaches. Prophylactic therapy is recommended if the
    headaches are more than 3 in a month, or are
    incapacitating requiring the patient to be hospitalized or
    miss work or the response to abortive medications is not
    satisfactory. A fi ne balance must be maintained between
    overmedicating and limiting acute attacks. It is not
    worthwhile to take prophylactic medication to prevent an
    occasional migraine once in two months. The aim of
    prophylactic therapy is to reduce the frequency and
    severity of the headaches by at least 50%. The best therapy
    for acute attacks is to use an abortive as in an upcoming
    job interview. Prophylactic therapy is not effective in
    tension type headaches. Migraines with prolonged aura
    can lead to permanent neurologic sequelae, in such cases
    prophylactic therapy maybe indicated.
    Ref: Raj, Robbins
    Source: Chopra P, 2004
555
Q
  1. Thrombosed external hemorrhoid pain:
  2. Has an abrupt onset
  3. Is of an aching or burning quality
  4. Is a localized anal pain
  5. Is associated with a tender, almond shaped mass extruding outside the canal
A
  1. Answer: E (All)

Source: Nader and Candido – Pain Practice. June 2001

556
Q
  1. The true statements regarding the N-methyl-D-aspartate
    (NMDA) receptor include the following
  2. Glutamate and aspartate act at the NMDA receptor
  3. NMDA may be involved in injury-induced wind-up
  4. Wind-up is prevented by NMDA antagonists
  5. Ketamine is an NMDA receptor agonist
A
  1. Answer: A (1, 2, & 3 )
    Explanation:
    NMDA receptors are involved in the activation of
    nociceptive neurons.
    The action of excitatory amino acids such as glutamateand
    aspartate at the NMDA receptor in the dorsal horn is
    enhanced by the neuropeptides substance P, calcitonin
    gene-related peptide (CGRP), and dynorphins.
    The corelease of excitatory amino acids and neuropeptides
    strengthens the synaptic connections in the dorsal horn
    andmay increase the development of dorsal-horn
    hyperxcitability (cord wind-up).
    There is an expansion of the receptive fi elds of the wide
    dynamic range (WDR) neurons.
    Wind-up is prevented by NMDA receptor antagonists.
    Both Mk-801 and ketamine are NMDA receptor
    antagonists.
    Source: Kahn and Desio
557
Q
1295. Trauma to the spinal accessory nerve would be expected
to cause:
1. spasm of the trapezius muscle
2. winged scapula
3. torticollis
4. hoarsenes
A
  1. Answer: B (1 & 3)
    Explanation:
  2. The spinal accessory nerve innervates the trapezius
    muscle, and trauma will cause spasm of trapezius and
    torticollis.
  3. Trauma of spinal accessory nerve cause torticollis – not
    winged scapula.
    - It is caused by pathology of long thoracic nerve.
  4. Trauma to spinal accessory nerve causes torticollis.
  5. Hoarseness might come from the recurrent laryngeal
    nerve, but, not spinal accessory nerve.
    Source: Trescot AM, Board Review 2003
558
Q
1296. During an intravenous lidocaine test, all the following
monitors are recommended
1. Electrocardiography
2. Blood pressure
3. Pulse oximetry
4. Skin temperature
A
  1. Answer: A (1, 2, & 3)
    Explanation:
    Intravenous lidocaine can be used to determine the
    effi cacy of treatment with oral antiarrhythmics such
    as mexiletine or tocainide. It may be used as a treatment
    itself if weekly infusions provide longer relief after each
    treatment.
    Some studies suggest that intravenous lidocaine may
    have predictive value as to the effi cacy of anticonvulsants
    such as phenytoin or carbamazepine.
    1, 2, 3. Because of the possibility of systemic toxicity and
    seizures, patients should be monitored by
    electrocardiography, blood pressure, and pulse oximetry.
  2. Skin temperature monitoring is not necessary.
    Source: Ramamurthy
559
Q

1297.A young female presents with a severe left-sided
throbbing headache associated with nausea, vomiting,
and photophobia. She has tried taking ibuprofen without
relief. On further questioning, she relates that she has
been having similar headaches three to four months:
1. Appropriate therapy for this patient’s present headache
includes Ergotamine tartrate.
2. Appropriate therapy for the present headache includes
Amitriptyline hydrochloride
3. Appropriate long-term management includes a prescription
for daily use of Amitriptyline hydrochloride.
4. Appropriate long-term management includes prescription
for daily use of Sumatriptan and metoclopramide.

A
  1. Answer: B (1 & 3)
    Explanation:
  2. This patient has migraine without aura. Of the agents
    listed, only Ergotamine tartrate is generally considered of
    use to abort a headache.
  3. Verapamil and amitriptyline hydrochloride may be used
    as prophylactic (preventive) therapy.
  4. Several medications are effective as prophylactic agents
    in the treatment of migraine. These include amitriptyline
    hydrochloride, propranalol, verapamil, and valproate.
    Most experts recommend initiating prophylactic therapy
    only when headaches occur at least one to two times per
    month.
  5. Metoclopramide hydrochloride, sumatriptan, and
    Ergotamine tartrate are appropriately used to treat an
    acute attack of migraine, and should not be prescribed on
    a daily basis. Daily use of these medications can establish a
    rebound syndrome that results in a daily headache. Oral
    contraceptives may be associated with either an increase or
    decrease in the frequency of migraines, but are not
    generally used as a treatment for migraine. Some experts
    recommend not prescribing OCPs for patients with
    migraine for fear of increasing the risk of a stroke,
    although OCPs are probably safe to use in most patients
    with common migraine.
560
Q
  1. True statements regarding temporal arteritis include the
    following:
  2. A swollen, tender scalp artery is present
  3. An elevated erythrocyte sedimentation rate (ESR)
  4. Typical histologic features on biopsy
  5. Polymyalgia rheumatica is frequently present
A
  1. Answer: E (All)
    Explanation:
    Diagnostic criteria for temporal arteritis (giant cell
    arteritis) include the presence of typical histopathologic
    features on temporal artery biopsy, a swollen and tender
    scalp artery, elevated ESR, and the disappearance of the
    headache with 48 h of steroid therapy.
    The headache is usually temporal, of variable severity,
    having a constant, boring quality, and is temporarily
    relieved by analgesics such as aspirin.
    Polymyalgia rheumatica as well as general malaise,
    anorexia, or mild fever frequently accompanies this
    systemic disease
561
Q
  1. The benefi cial effects of epidural administration of
    steroids have been attributed to of the following:
  2. Inhibit phospholipase A2
  3. Improve microcirculation around the nerve root
  4. Block conduction of nociceptive C nerve fi bers
  5. NMDA antagonist
A
  1. Answer: A (1, 2, & 3)
    Explanation:
    Administration of epidural steroids by interlaminar or
    transforaminal approach is one of the commonest
    approaches to treating spinal pain and radicular pain.
    Steroids decrease infl ammation by inhibiting
    phospholipase A2, thus inhibiting the formation of
    arachidonic acid, prostaglandins and leukotrienes.
    Steroids reduce infl ammatory edema around the infl amed
    nerve root and improve microcirculation. They block the
    conduction of nociceptive c fi bers. By restricting the
    formation of prostaglandins they may decrease
    sensitization of the dorsal-horn neurons.
    Source: Chopra P, 2004
562
Q
1300. True statements about complications from neurolytic
hypogastric plexus block are as follows:
1. intravascular injection
2. paralysis of lower extremity
3. injury to ureter
4. nerve injury
A
  1. Answer: E (All)

Source: Racz G. Board Review 2003

563
Q
1301.Benefi ts of continuous epidural analgesia for chest
trauma includes:
1. Shorter ICU stay
2. Improved post-injury rehabilitation
3. Avoidance of endotracheal intubation
4. Earlier post-injury extubation
A
  1. Answer: E (All)
    Explanation:
    Ref: Crews. Chapter 14, Sente Pain Syndromes. In:
    Practical Management of Pain, 3rd Edition. Raj et al,
    Mosby, 2000, page 185.
    Source: Day MR, Board Review 2003
564
Q
  1. A patient presents with lateral epicondylitis. Pain is
    noted on physical examination with which of following
    maneuvers during the examination:
  2. Resisted fl exion
  3. Resisted extension
  4. Resisted supination
  5. Extension of the wrist with a fi st
A
  1. Answer: C (2 & 4)
    Explanation:
    Lateral epicondylitis, or tennis elbow, commonly involves
    the origin of extensor-supinator muscle mass in the:
    Extensor carpi radialis brevis
    Extensor digitorum communis
    Extensor carpi radialis longus
    Extensor carpi ulnaris
    Supinator
    The extensor carpi radialis is most commonly involved.
    Probably the position of wrist fl exion, elbow extension,
    and forearm pronation stretch the tendon over the
    prominence of the radial head.The most common cause of
    lateral epicondylitis is cumulative trauma.
    Provocative testing involves elbow in extension. Further,
    in lateral tennis elbow, pain is reproduced when one asks
    the patient to make a fi st and extend the wrist. Sudden,
    severe pain is elicited at the lateral epicondyle when the
    examiner forcefully extends the patient’s wrist.
    Source: Saidoff DC, McDonough AL. Critical Pathways in
    Therapeutic Intervention. Extremities and Spine,St. Louis,
    Inc., 2002.
565
Q
1303.True statements of adverse reaction of celiac plexus block
include the following:
1. Urinary retention
2. Hypotension
3. Sexual dysfunction
4. Diarrhea
A
  1. Answer: C (2 & 4)
566
Q
1304. All of the following clavicular movements are involved in
upper extremity elevation less than 150°
1. Backward Spin
2. Elevation
3. Retraction
4. Protraction
A
  1. Answer: A (1, 2, & 3)

Source: Sizer Et Al - Pain Practice March & June 2003

567
Q
  1. A patient has intractable cancer pain with a neuropathic
    component. Ketamine is being considered as an adjuvant
    analgesic agent. Which of the following are correct
    regarding ketamine?:
  2. Sympathomimetic effects
  3. Noncompetitive NMDA antagonist
  4. Contraindicated with increased ICP
  5. May reduce the requirements for opioids
A
  1. Answer: E (All)
568
Q

1306.The following maneuvers decrease carpal tunnel
pressure
1. Forearm neutral position
2. Intermittent low tension hand exercise
3. Slight wrist palmar fl exion with ulnar deviation
4. Full wrist dorsal extension (cock-up-position)

A
  1. Answer: A (1, 2, & 3)

Source: Sizer et al - Pain Practice - March & June 2004

569
Q
  1. A 55-year old slender white female complains of back
    pain that started a week ago. This started when she
    lifted a box with both hands. There was no signifi cant
    radiation, however, it was exacerbated with any further
    activity including with lifting. She became menopausal
    approximately 2 years ago, she smoked, she does not
    exercise. The only medications she had used were
    diazepam on a long term basis. The true statements
    relating to the diagnosis and management of her
    condition are as follows:
  2. This patient suffered thoracic vertebral compression
    fracture secondary to osteoporosis.
  3. This patient suffered disc herniation
  4. Treatment includes a rigid thoracolumbar hyperextension
    orthosis, which provides external support and alleviates
    fl exion forces on the affected vertebral segments
  5. Manage with fl exion exercises
A
  1. Answer: B (1 & 3)
    Source: Saidoff DC, McDonough AL. Critical Pathways in
    Therapeutic Intervention. Extremities and Spine,St.
    Louis,Inc., 2002
570
Q
  1. Central pain arising from brain injury has been shown
    to result from which of the following structures?
  2. Brainstem
  3. Cerebral cortex
  4. Thalamus
  5. Subcortical white matter
A
  1. Answer: E (All)
    Explanation:
    Modern imaging studies have demonstrated that lesions in
    all regions of the brain can cause central pain. Bonica’s
    Management of Pain, 3rd ed, page 441.
571
Q
  1. Pulsed radiofrequency lesioning settings may be:
  2. 46 degrees Celsius and 20 volts
  3. 56 degrees Celsius and 100 volts
  4. 80 degrees Celsius and 40 volts
  5. 42 degrees Celsius and 40 volts
A
  1. Answer: D (4 Only)

Source: Racz G. Board Review 2003

572
Q
  1. FDA approved intrathecal medications include:
  2. Morphine
  3. Clonidine
  4. Baclofen
  5. Ziconitide
A
  1. Answer: B (1 & 3)
    Explanation:
    Morphine and baclofen are FDA approved for intrathecal
    use. Clonidine is approved for epidural but not intrathecal
    use. Ziconitide is still awaiting fi nal approval.
    Source: Trescot AM, Board Review 2004
573
Q
1311. As a part of a psychological evaluation, a clinical
interview includes:
1. Pain distribution and pattern
2. Financial and legal information
3. General medical status
4. Psychosocial history
A
  1. Answer: A

Source: Janata J, Board Review 2006

574
Q
  1. An interlaminar epidural steroid injection is an
    appropriate treatment choice for a patient with:
  2. A C6 disc herniation and severe cervical canal stenosis
  3. Back and leg pain due to spinal metastases
  4. Facet arthropathy producing severe back pain
  5. An L5 disc herniation without neurological fi ndings
A
  1. Answer: D (4 Only)
575
Q
  1. Which of the following are Waddell’s signs?
  2. Evoked back pain with deep palpation of the lumber
    paraspinals
  3. Evoked back pain with en-bloc trunk rotation, i.e., moving
    the shoulders and hips in unison
  4. Refuses to be examined
  5. Superfi cial tenderness
A
  1. Answer: C (2 & 4)
    Explanation:
    Waddell’s signs were developed to suggest a possible
    non-organic etiology of back pain as opposed to suggesting malingering. These tests and the clinician’s
    clinical impression may suggest a slower than expected
    recovery.
    The Waddell’s signs include (SONDSup, mnemonic):
    Simulation
    Load the spine with the weight of your hand on top of
    the patients head to reproduce low back pain
    Simulation of twisting the trunk when rotating the
    shoulders and hips in unison to reproduce the back pain
    Non-anatomic distribution of pain
    Aberrant pain drawing give away’ weakness, i.e.,
    inconsistent effort during ange of motion
    Distraction
    Sitting knee extension to test sciatic tension while
    distracting the patient with a knee, foot, or peripheral
    pedal pulse examination
    If negative, then the supine straight leg maneuver
    should be negative
    Superfi cial or subcutaneous tenderness, not deep muscle
    tenderness
  2. Evoked back pain with deep palpation of the lumber
    paraspinals is not a Waddell’s sign
  3. Evoked back pain with en-bloc trunk rotation, i.e.,
    moving the shoulders and hips in unison is a Waddell’s
    sign
  4. Refusal to examination is not a Waddell’s sign
  5. Superfi cial tenderness is a Waddell’s sign
    Source: Shah RV: 2003 (Bonica, 3rd Ed., page 1523)
576
Q
  1. The criteria for diagnosing hypochondriasis include
  2. Pseudoneurological presentation
  3. Persistent preoccupation despite medical reassurance
  4. The preoccupation is delusional
  5. The preoccupation has a duration of six months or
    more
A
  1. Answer: C

Source: Janata J, Board Review 2006

577
Q
  1. The Cremasteric Reflex best tests for what nerve root?
  2. L1
  3. L2
  4. L3
  5. L4
A
  1. Answer: A ( 1, 2, & 3)

Source: Wirght PD, Board Review 2004

578
Q

1316.The treatment of piriformis syndrome includes the
following:
1. Non-steroidal anti-infl ammatory drugs
2. Piriformis stretch exercise program
3. Piriformis injection with local anesthetic and steroids
4. Surgical section of piriformis muscle

A
  1. Answer: A (1, 2, & 3)
    Explanation:
    Surgical section of piriformis muscle is performed on
    extremely rare occasions.
579
Q
  1. Strengthening exercises:
  2. Are helpful in patients with chronic low back pain.
  3. Should be stopped if a patient complains of increase
    muscle soreness.
  4. Can result not only in improvement in strength but also
    reduction of pain.
  5. Provides the best results when performed one time per
    week at low loads.
A
  1. Answer: B (1 & 3)

Source: Malanga G, Board Review 2003

580
Q
  1. A 36-year old male presented with severe low back and
    left lower extremity pain. He reported the pain to start
    following a work related injury. Examination showed a
    positive straight leg raising on the right at 60°, and an
    absent left Achilles tendon refl ex. He was treated with
    physical therapy, improved, and returned to work after 6
    weeks. He had no pain at rest or numbness in the lower
    extremities one year after onset. He was able to perform
    all activities of daily living with only some back pain with
    heavy activity. His MRI showed left posterolateral disc
    herniation at L5/S1.
  2. His diagnosis is lumbar strain.
  3. His diagnosis is lumbar disc herniation with
    radiculopathy.
  4. His impairment rating is 20% impairment of the whole
    person.
  5. His impairment is 5% of the whole person.
A
  1. Answer: C (2 & 4)
581
Q
  1. True statements about early changes on EMG/NCV after
    L5 disc herniation include :
  2. Positive sharp waves are fi rst noticeable in paraspinal
    muscles within 7-10 days after loss of axon function
  3. By 14-18 days, positive sharp waves can appear in limb
    muscles, becoming evident throughout the involved
    myotome
  4. By 18-21 days, all muscles in the involved myotome
    have abnormalities, including positive sharp waves and
    fi brillation potentials
  5. Smaller amplitude positive sharp waves (100-150 MV)
    are indicative of acute injury
A
  1. Answer: A (1,2, & 3)
582
Q
1320. All of the following muscles are adductors while the arm
is positioned at the patient’s side
1. Latissimus dorsi
2. Pectoralis major
3. Teres Major
4. Subscapularis
A
  1. Answer: A (1, 2, & 3)

Source: Sizer Et Al - Pain Practice March & June 2003

583
Q
1321. Which of the following tests are used to evaluate the
meniscal injuries?
1. McMurray’s Test
2. Patellar Grind Test
3. Apley’s Compression Test
4. Lachman’s Maneuver
A
  1. Answer: B (1 & 3)
    Explanation:
    (Raj, Practical Mgmt of Pain, 3rd Ed.)
    Special Tests
  2. McMurray Test
    This maneuver was developed to assess for posterior
    meniscal tears and provides an excellent clinical
    evaluation. The patient lies prone. The examiner fl exes the
    symptomatic knee and rotates the tibia on the femur in
    external and internal rotation. Valgus stress is added with
    the leg in external rotation, and the knee is then slowly
    extended. An audible or palpable click suggests a meniscal
    tear.
  3. Patellar Femoral Grinding Test
    Chondromalacia patellae is a common problem in
    degenerative knees, and there are common complaints of
    increasing pain on arising from a chair or climbing stairs.
    Exacerbation of symptoms can be elicited by compressing
    the patella into the femoral groove. With the knee
    extended, pressure is placed over the patella, which is
    guided along the groove. Crepitance should also be
    palpated with fl exion and extension of the knee while the
    examiner’s hand is over the patella.
  4. Apley’s Compression or Grinding Test:
    A confi rmatory test for meniscal tears can be accomplished
    by compressing the meniscus. The patient lies prone and
    the affected knee is fl exed to 90 degrees. The examiner
    applies downward pressure against the heel as he or she
    rotates the tibia against the femur. Pain suggests a
    meniscal tear and correlates medially or laterally with the
    location of injury.
  5. Drawer Signs
    These tests were designed to examine injury or disruption
    of the cruciate ligaments. The patient may be sitting or
    lying prone with the knee fl exed at 90 degrees and the foot
    fi xed in place (the examiner may sit on the foot). The tibia
    is then drawn toward the examiner; if the tibia slides
    beneath the femur, it is a positive anterior drawer sign and
    identifi es a torn anterior cruciate ligament (ACL). If
    sliding occurs beneath the femur when the tibia is pushed
    away from the examiner, it is a positive posterior drawer
    sign and identifi es a torn posterior cruciate ligament
    (PCL).
    Source: Shah RV, Board Review 2004
584
Q
  1. The mechanism of action of low frequency acupuncture
    pain relief is explained by which of the following?
  2. Hypothalamic stimulation
  3. Reticulospinal suppression
  4. Activation of A fi ber pathways
  5. Release of endogenous enkephalins
A
  1. Answer: D (4 Only)
585
Q
  1. Complications of a single epidural steroid injection
    may include
  2. Cushing’s syndrome
  3. Elevated blood glucose
  4. Hypothalamic-pituitary-adrenal axis suppression
  5. Arachnoiditis
A
  1. Answer: A (1, 2, & 3)
586
Q
  1. The following trunk muscle groups have been identifi ed
    as targets in a restorative spine stabilization program,
  2. Diaphragm
  3. Multifi dus
  4. Transverse abdominus
  5. Rectus abdominus
A
  1. Answer: A (1, 2, & 3)

Source: Sizer et al - Pain Practice - March & June 2004

587
Q
1325. Anterior interosseous nerve syndrome would spare
which of the following muscles?
1. Flexor pollicis longus
2. Pronator quadratus
3. Index fl exor digitorum profundis
4. Pronator teres
A
  1. Answer: D (4 Only)
    Explanation:
    The anterior interosseous nerve usually innervates the
    FPL, index and long fi nger FDPs, and PQ. It does not
    mediate superfi cial sensation. Patients may present with
    loss of fl exion of the distal phalanxes of the thumb and
    index fi nger. They lose their pinching ability.
    Source: Shah RV, Board Review 2004
588
Q
  1. Which of the following can cause ulnar nerve palsy
  2. Cubitus valgus deformity
  3. Subluxation of the ulnar nerve onto or past the medial
    humeral epicondyle
  4. Occupational hazard, as a worker supports themselves
    on their elbows
  5. An aponeurotic band that extends from the medial
    epicondyle of the humerus and attaches to the medial
    border of the olecranon.
A
1326. Answer: E (All)
Explanation:
All can cause ulnar nerve palsy. Choice 4 is responsible for
cubital tunnel syndrome.
Source: Shah RV, Board Review 2004
589
Q
  1. Resisted wrist dorsal extension can provoke symptoms
    associated with following lateral elbow affl ictions
  2. Humeroradial joint chondropathy
  3. Posterior interosseus nerve entrapment
  4. Tendopathy of the extensor carpi radialis brevis
  5. Tendopathy of the fl exor carpi ulnans
A
  1. Answer: A (1, 2, & 3)

Source: Sizer et al - Pain Practice - March & June 2004

590
Q
  1. True statements about hypogastric plexus are:
  2. located in front of promontory crossing L5-S1
  3. anterior just Left side of aorta
  4. communicates with celiac ganglion
  5. contains B-C and sympathetic fi bers
A
  1. Answer: E (All)

Source: Racz G. Board Review 2003

591
Q
  1. The management of spinal stenosis includes the
    following:
  2. Medication with non-steroidal anti-infl ammatory
    drugs and calcitonin
  3. Flexibility training with slight fl exion bias in neutral
    spine position as it improves the stenosis
  4. Epidural injections are indicated in patients without
    improvement with aggressive conservative care or increased
    symptoms
  5. Surgery is indicated for intolerable pain with deterioration
    in functional status or progressive neurological
    defi cit or cauda equina symptoms
A
  1. Answer: E (All)
592
Q
  1. Shoulder impingement should be suspected in a patient
  2. that demonstrates a positive drop arm test
  3. that demonstrates a ‘Popeye’ deformity
  4. if the Yergason’s test is positive
  5. has pain with abduction
A
  1. Answer: D (4 Only)
    Explanation:
    (Raj, Practical Mgmt of Pain 3rd Ed., page 347-9)
  2. A drop arm test is consistent with a complete rotator
    cuff tear. A complete rotator cuff tear does not allow the
    arm to remain abducted, but partial tears can also be
    assessed by this test. The patient abducts the arm to 90
    degrees and is asked to lower it slowly after the examiner
    taps the extended forearm. Complete tears in the
    supraspinatus tendon cause the arm to fall immediately to
    the side, and partial tears prevent full strength or range.
  3. A ‘popeye’ deformity signifi es a tear of the long head of
    the biceps tendon
    Impingement is most pronounced with forward fl exion
    and thumbs down or abduction.Mechanical entrapmentof
    the rotator cuff occurs at the space between the humeral
    head and coracoacromial arch narrows.
  4. The Yergason Test for Biceps Tendon Stability—while,
    the patient is sitting or standing, the examined elbow is
    fl exed at the waist and a fi st is made with the hand. The
    examiner has one hand on the patient’s elbow and the
    other on the distal forearm. The patient resists shoulder
    external rotation (examiner pulls outward on distal
    forearm) and pulls downward at the elbow. If the long
    biceps tendon pops out of its groove, pain arises near the
    anterior lateral
    humeral head.
  5. Shoulder impingement produces pain with abduction.
    Source: Shah RV, Board Review 2004
593
Q
  1. Positive Waddell’s signs are indicative of:
  2. The effects of psychosocial factors
  3. A poor response to surgical intervention
  4. Need for a comprehensive evaluation of pain
  5. Lack of response to treatment
A
  1. Answer: A (1, 2, & 3)
    Explanation:
    The 4 fi ndings include superfi cial, nonanatomic tenderness; a positive simulation response; a discrepancy
    between results of examination of the same body part
    in two different positions; and non-physiologic regional
    disturbances of sensation, pain or weakness.
  2. Positive Waddell’s signs greatly increase the likelihood
    that psychosocial factors are playing a major role in the
    patient’s complaints.
  3. Patients with two or more positive test results may not
    respond favorably to surgery.
  4. Positive Waddell’s signs are indicative of the need for a
    comprehensive evaluation.
  5. Patients with positive Waddell’s signs may not respond
    well.
594
Q
  1. Which is true?
  2. Axial loading of the neck may be helpful in the evaluation
    of chronic low back pain
  3. The Adson test is used to evaluate thoracic outlet syndrome
  4. Resistance to passive neck fl exion with the hands behind
    the head, so that the chin cannot touch the chest is a
    sign that is used to evaluate meningeal irritation
  5. Increased neck pain with side bending is pathognomic
    for cervical facet pain
A
  1. Answer: A (1,2, & 3)
    Explanation:
    (Raj, Practical Mgmt of Pain, 3rd Ed., page 357, 358)
  2. Axial loading of the neck is used in evaluating Waddell’s
    signs (simulation testing). Waddell’s tests were developed
    to evaluate functional overlay in low back paincomplaints.
    Each of the following fi ndings is considered positive if
    present; a total of three positive fi ndings is considered
    signifi cant, strongly suggesting positive non-physiologic
    signs.
  3. The Adson test is used for subclavian artery
    compression. The patient’s radial pulse is continually taken
    at the wrist while the arm is abducted, extended, and
    externally rotated.The patient then takes a deep breath and
    turns the chin toward the tested arm. The examiner
    palpates a drop in pulse pressure or loss of pulse,
    suggesting compression of the artery.
  4. Kernig test. The patient lies supine with hands behind
    the head and is asked to fl ex the chin to the chest wall.
    Nerve root, meningeal, or dural infl ammation results in a
    shooting pain in the spinal canal or legs.
    Tenderness: poorly localized and does not follow
    dermatomal or documented referral patterns.
    Light touch over the low back causing widespread
    discomfort or deep touch spreading through the thoracic
    spine or to the sacrum or hips.
    Simulation testing: should not be uncomfortable or
    cause discomfort in distant sites. Axial loading of the skull
    causing lumbar pain or shoulder rotation causing lumbar
    pain.
    Distraction testing: inconsistent fi ndings with the same
    test performed in formal fashion and when
    attention is distracted. Sitting straight leg raising
    without discomfort compared with lying straight
    leg raising causing radiating pain from buttock to foot.
    Regional disturbance: nonanatomical findings on sensory and motor testing. Give-way motor
    testing (total release of motor activity without warning)
    or unexplained weakness. Glove and
    stocking dysesthesias rather than expected dermatomal
    pattern.
    Overreaction: inappropriate facial or verbal expressions,
    withdrawal of limbs from touch, or posture
    contortions. Flopping on the fl oor with twisting of the
    spine, limiting hypersensitivity to joint examination, and
    cries of pain or fear on superfi cial examination.
  5. Cervical facet joint pain is diagnosed with a certain
    degree of certainty, utilizing controlled diagnostic blocks -
    but not by physical examination.
    Source: Shah RV, Board Review 2004
595
Q
  1. Which of the following observations, after nerve injury,
    is correctly paired with the appropriate nerve?
  2. Inability to fl ex the forearm — radial nerve
  3. Numbness in the index fi nger—median nerve
  4. Inability to extend the forearm—musculocutaneous
    nerve
  5. Numbness in the little fi nger—ulnar nerve
A
  1. Answer: C (2 & 4)
    Explanation:
    To check the setup of a brachial plexus block, one can
    perform the four P’s (push, pull, pinch, pinch). Have the
    patient push or extend the forearm (triceps muscle is
    innervated by the radial nerve), pull or fl ex the forearm
    (biceps muscle is innervated by the musculocutaneous
    nerve), pinch the index or second fi nger (median nerve),
    pinch the little fi nger (ulnar nerve).
    Source: Hall and Chantigan
596
Q
  1. The Quebec Task Force on Whiplash Associated
    Disorders recommends CT or MR imaging in which
    subset of patients?
  2. Grade II (neck pain + musculoskeletal injury)
  3. Grade IV (neck pain + bony injury)
  4. Grade I (neck pain)
  5. Grade III (neck pain + neurological injury)
A
  1. Answer: C (2 & 4)
    Explanation:
    The Quebec Task Force on Whiplash Associated
    Disorders graded the severity of whiplash, as follows:
    Grade I (neck pain)
    Grade II (neck pain + musculoskeletal injury)
    Grade III (neck pain + neurological injury)
    Grade IV (neck pain + bony injury)
    1, 3. They recommended plain radiographic imaging in
    grades II, III, IV
    2, 4. They recommended CT or MRI in grades III, IV
    Source: Shah RV: 2003 (Bonica, 3rd Ed., page 1010)
597
Q
  1. A patient presents with pain in the upper extremity
    following injury to the forearm. Examination showed
    weakness in the ring and little fi ngers with numbness in
    the little fi nger. The most likely diagnosis is:
  2. C7/T1 disc herniation
  3. T1/T2 disc herniation
  4. Median nerve injury
  5. Ulnar nerve injury
A
  1. Answer: D (4 Only)
    Explanation:
    A peripheral nerve injury to the ulnar nerve causes
    weakness only in the ring and little fi ngers. However, a
    central lesion or disc herniation will cause weakness in all
    fi ngers. The fl exor digitorum superfi cialis, which fl exes
    the proximal interphalangeal joint, has only median nerve
    innervation, and is affected by root injury to C8 and
    peripheral injuries to the median nerve, but not peripheral
    injury to ulnar nerve
    Source: Hoppenfeld S. Orthopaedic Neurology. A
    Diagnostic Guide to Neurologic Levels. Philadelphia,
    LWW, 1997
598
Q
  1. Treatment for back, hip, and thigh pain in a patient
    with spondylolisthesis at L5/S1 includes which of the
    following?
  2. Pelvic tilt for trunk stabilization
  3. Flexibility training program with extension bias
  4. Strength training with fl exion bias
  5. Start exercises immediately in brace with pain
A
  1. Answer: B (1 & 3)
599
Q
  1. What is true about carpal tunnel syndrome?
  2. Patients may complain about numbness in the thumb
  3. Pain may be present in the forearm, shoulder, and elbow
  4. Hypesthesia is often present in the volar surface of the middle finger
  5. Hypothenar muscle atrophy may be present
A
  1. Answer: A (1, 2, & 3 )
    Explanation:
    CTS has variable clinical presentations, but patients often
    complain of numbness of the index fi nger, thumb, andring
    fi ngers. Pain may extend from the wrist, hand, forearm,
    elbow, and shoulder. Sensory testing may reveal loss of 2
    point discrimination in the thumb, index, and middle
    fi ngers. Hypothenar muscles are in the distribution of the
    ulnar nerve, whereas the thenar muscles are in the median
    nerve distribution
    Source: Shah RV, Board Review 2004
600
Q
  1. All of the following statements are true regarding the
    subacromiodeltoid bursa
  2. It is often the 1° source of pain with traumatic rotator
    cuff tears in patients younger than 40 years.
  3. It is the most densely innervated structure in the glenhumeral
    region
  4. It may be involved in the neurological regulation of
    shoulder movements
  5. It’s size and compartmental confi gurations are predictable
    and consistent across patients.
A
  1. Answer: A (1, 2, & 3)

Source: Sizer Et Al - Pain Practice March & June 2003

601
Q
1339. Which of the following would support the diagnosis of
an S1 nerve root lesion?
1. Absent ankle jerk
2. Weakness in toe walking
3. Atrophy of the gastrocnemius
4. Knee pain
A
  1. Answer: A (1, 2, & 3)
    Explanation:
    S1 nerve root injury may be associated with weakness of
    plantar fl exion, occasional cramping in the calf, and absent
    ankle jerk. Atrophy of the gastrocnemius, soleus, and
    hamstrings may occur. Sagging of the gluteal fold and loss
    of gluteal muscle tone also suggest S1 involvement.
    Radicular pain in the knee is suggestive of L3 injury.
    Source: Wall, p
602
Q
  1. Tadpole lesions
  2. Refers to the nerve swelling proximal to the area of the
    entrapped nerve
  3. Refers to delamination of myelin and resultant accumulation
    in the internodes
  4. Refers to appearance of ovoids, as the distal segment of
    the axon breaks down following axonal injury
  5. Are early harbingers of the process of demyelination
    and remyelination that occur with conduction block
A
  1. Answer: C (2 & 4)
    Explanation:
    Tadpole lesions occur as a consequence of myelin
    delamination and accumulation into the paranodal
    bulbous aspects of the internode. They are early signs of
    demyelination and remyelination, which occur as a
    consequence of chronic entrapment and ischemia. The
    myelin is of irregular thickness in the entire region of the
    entrapment: thinner near the area of entrapment and
    thicker away from the middle. These are polarized, such
    that it looks as if a tadpole is swimming away from the
    region of the entrapment.
    During Wallerian degeneration, the axons in the distal
    segment breakdown and begin to look like ovoids. Nerve
    swelling proximal to the entrapped nerve can be visibly
    seen during surgery and it represents fi brosis, increased
    connective tissue, and endoneurial swelling
    Source: Shah RV, Board Review 2004
603
Q
  1. Proper stretching exercises includes:
  2. Performing each stretch as quickly as possibly with multiple
    repetitions.
  3. Holding each stretch for at least 30 seconds.
  4. Avoiding placing any tension on the muscle.
  5. Stretching after a proper warm-up period to allow for a
    better stretch.
A
  1. Answer: C (2 & 4)

Source: Malanga G, Board Review 2003